You are on page 1of 397

1 Some Basic Concepts

Chemistry
of

Past Years NEET Trend

4
No. of MCQs

0
2021 2020 2019 2018 2017 2016 2015 2014 2013 2012

Investigation Report
TARGET EXAM PREDICTED NO. OF MCQs CRITICAL CONCEPTS
• Mole concept
NEET 1-2 • Stoichiometry and stoichiometric calculations

Perfect Practice Plan


Topicwise Questions Learning Plus Multiconcept MCQs NEET Past 10 Years Total MCQs
Questions
105 39 22 17 183
INTRODUCTION Material is another very common term used in chemistry. However,
Chemistry is the branch of science that studies the composition, the term material has a limited meaning, which corresponds to
matter having specific uses.
properties and interaction of matter.
Anything that has mass and occupies space is called matter. Classification of Matter
For example: book, pen, pencil, water, air, all living beings etc. Matter can be classified in two ways:
are composed of matter. You know that they have mass and they (i) Physical classification of matter
occupy space. (ii) Chemical classification of matter

MATTER

Chemical Classification Physical Classification

Based on intermolecular
Pure substance Mixture forces

Element Compound Homogeneous Heterogeneous Solid Liquid Gas

Metal Non-metal Metalloid Inorganic Organic

Physical Classification of Matter The components of such a mixture cannot be seen even under
Depending upon the physical state of matter, it can be classified a microscope. Some examples of homogeneous mixtures are air,
into three states, namely, solid, liquid and gaseous state. gasoline, sea water, brass, coloured glass, Alloys, Water + alcohol,
Water + Salt, 22 carat Gold, Water + Sugar, etc.
Properties Solid Liquid Gas Phase : A distinct portion of matter that is uniform in composition
and property is called a phase.
Shape Definite Indefinite Indefinite
In heterogeneous mixtures, the composition is not uniform
Volume Definite Definite Indefinite throughout. These consist of two or more parts (called phases)
which have different compositions.
Attraction Force Strongest Moderate Weakest
For Example : Water + Sand, Water + Mustard oil, Milk, Blood
Examples Sugar, Iron, Water, Milk, Dihydrogen, Air, plastic, smoke, petrol etc.
Gold, Wood oil, Mercury Oxygen, carbon
Pure Substances Consist of Single Type of Particles
etc. dioxide, etc
Pure substances can be further classified into elements and
heat heat compounds.


Solid  

 Liquid 
 Gas
cool cool 1. Element: An element is the simplest form of a pure
substance. It is defined as:
These three states of matter are interconvertible by changing The simplest form of a pure substance that can neither be
temperature and pressure. decomposed into nor built from simpler substances by ordinary
physical or chemical methods. For example Zn, B, Si.
Chemical Classification of Matter 2. Compound: A compound is defined as a pure substance that
contains two or more than two elements combined together
The chemical classification of matter is based upon its composition. in a fixed proportion by mass and that can be break down
At the macroscopic or bulk level, matter can be classified as into its constituent elements by suitable chemical methods.
mixture or pure substances. Compounds are further classified into two categories.
Mixture : Mixtures are defined as the substances which are made 1. Organic Compound
up of two or more pure substances. They can possess variable For Example : Sources, Oils, fats, derivative of
composition and can be separated into constituent components by hydrocarbon.
some suitable physical means/methods. 2. Inorganic Compound
For Example : HCl, H2O, H2SO4, HClO4, HNO3 etc.
For Example : Alloys (Brass, Bronze) (Brass = Copper + Zinc)
(Bronze = Copper + Tin) Water + alcohol, Water + Salt, Water +
mustard Oil, Water + Sugar, Water + Kerosene
TRAIN YOUR BRAIN
A mixture may be homogeneous or heterogeneous. Q. Which of the following mixture(s) are homogeneous?
Tap water, Air, Soil, Smoke
In a homogeneous mixture, the components completely mix
with each other and its composition is uniform throughout. Ans. Tap water, Air.

6 Dropper NEET
Q. Classify the following as pure substances or mixtures. Also Density
separate the pure substances into elements and compounds It is of two type
and divide mixture, into homogeneous and heterogeneous
categories : mass
● Absolute density =
(i) Graphite (ii) Milk volume
● Relative density or specific gravity
(iii) Air (iv) Oxygen
density of the substance
(v) 22 carat gold (vi) Iodized table salt =
(vii) Wood (viii) Cloud density of water at 4°C
Ans. Element: (i), (iv) Homogeneous Mixture : (iii), (v) Heteroge- We know that density of water at 4ºC = 1 g/ml.
neous Mixture : (ii), (vi), (vii), (viii)
For Gases
PROPERTIES OF MATTER AND THEIR Molar mass
Absolute density (mass/volume) =
MEASUREMENT Molar volume
Physical properties are those properties that can be measured
or observed without changing the identity or composition of the Relative Density or Vapour Density
substance. Example: Colour, Odour etc. Vapour density is defined as the density of the gas with respect to
Chemical properties are those in which a chemical change in that of hydrogen gas at the same temperature and pressure.
the substance takes place. Example: pH, Heat of combustion etc. d gas PM gas / RT
Vapour density = =
Expressing a Physical Quantity dH2 PM H 2 / RT
The value of a physical quantity is always expressed in two parts: Where P is pressure of gas, M = mol. wt. of gas, R is the gas
(i) Numerical value and (ii) Unit constant, T is the temperature.
M gas M gas
The International System of Units (SI Units) V.D. = =
M H2 2
The scientists have generally agreed to use the International
System of Units abbreviated as SI units. Mgas = 2 V.D.
The SI system has seven base units and are listed in table: Relative density can be calculated w.r.t. to other gases also.
4. Temperature: There are three common scales to
Physical Quantity Symbol for Name of Symbol
quantity Unit measure temperature:
1. The SI scale or Kelvin scale measured in Kelvin (K)
Length l Metre m
2. Celsius scale measured in degree Celsius (°C).
Mass m Kilogram kg
3. Fahrenheit scale measured in degrees Fahrenheit (°F)
Time t second s
(i) Conversion of celsius to Fahrenheit is
Thermodynamic Temperature T kelvin K
9
Electric current I ampere A
°F = ( °C ) + 32°
5
Amount of Substance n mole mol (ii) Conversion of Fahrenheit to celsius
Luminous Intensity Iv candela cd 5
°C = [°F − 32°]
The two temperature units are related as: 9
● Kelvin temperature (K) = °C + 273.15
o UNCERTAINTY IN MEASUREMENT
● 1A = 10−10 m Significant Figures: The uncertainty in the experimental or
1nm =10−9 m the calculated values is indicated by mentioning the number of
1pm = 10−12 m significant figures.
Significant figures are those meaningful digits which are known
Some Commonly used Quantities with certainty. The uncertainty is indicated by writing the certain
1. Mass and Weight: Mass of a substance is the amount of digits and the last uncertain digit.
matter present in it. The SI unit of mass is kilogram.
Weight is the force exerted on an object by the pull of Accuracy and Precision
gravity. Accuracy is a measure of the difference between the experimental
2. Volume: Volume is the amount of space occupied by an value or the average value of a set of measurements and the true
object. So in SI system, volume has units cubic meter, m3. value.
3. Density: Density of a substance is its amount of mass per Precision refers to closeness of two or more measurements of the
unit volume. SI unit of density is kg/m3 same quantity that agree with one another.
Some Basic Concepts of Chemistry 7
Rules for Determining the Number of Significant Figures:
 KEY NOTE
1. All non-zero digits are significant. For example, 3.132 has
Š Nuclear reactions are exception of law of conservation of mass.
four significant figures.
In nuclear reaction mass + energy is conserved.
2. Zeros between two non zero digits are significant. For Š According to the modern views, the law of conservation of
example, 3.01 has three significant figures. mass is not always valid. The law hold good only in case of
such chemical reactions where there is no evolution of heat
3. The zeros preceding to the first non-zero number (i.e. to the
or light.
left of the first non-zero number) are not significant. Such
Š During chemical processes, the loss of mass is negligible.
zeros indicate the position of decimal point. For example, But in nuclear reactions, tremendous amount of energy is
0.324 has three significant figures. evolved. Consequently, the change in mass is quite significant.
4. All zeros at the end or to the right of a number are significant Thus, it is clear that the law of conservation of mass and law of
conservation of energy are two ways of looking at the same law.
provided they are on the right side of the decimal point. For
Š Thus, combining the two we get general law known as law of
example, 0.0200 has three significant figures.
conservation of mass energy. It states that, Mass and energy
5. Exponential form: N × 10n. Where N shows the significant are inter convertible. But the total sum of mass and energy of the
figure. system remains constant.
E.g., 1.86 × 104 has three significant figure.
6. Rounding off the uncertain digit: TRAIN YOUR BRAIN
(i) If the left most digit to be rounded off is more than 5, the Q. 10 g of CaCO3 on heating gives 4.4 g of CO2 then determine
preceding number is increased by one. weight of produced CaO in quintal.
E.g., 2.16 is rounded to 2.2 Ans. Total mass of reactant = 10 g
(ii) If the left most digit to be rounded off is less than 5, the Mass of CO2 = 4.4 g
preceding number is retained. Mass of produced CaO = x
E.g., 2.14 is rounded off to 2.1 According to law of conservation of mass
(iii) If the left most digit to be rounded off is equal to 5, the 10 = 4.4 + x
preceding number is not changed if it is even and increased 10 – 4.4 = x
by one if it is odd. x = 5.6 g
E.g., 3.25 is rounded off to 3.2 ∴1 quintal = 100 kg
2.35 is rounded off to 2.4 ∴ 1 Kg = 1000 g
Kg
     = 5.6 × = 5.6 × 10–3 × Kg
TRAIN YOUR BRAIN 1000
Q. How many significant figure are there in each of the 1
following numbers:      = 5.6 × 10–3 × quintal = 5.6 × 10–5 quintal
100
(i) 1.00 × 106 (ii) 0.00010 (iii) π
Ans. (i) Three (ii) Two (iii) An infinite number 2. Law of Definite Proportions
Given by → Joseph Proust: A chemical compound always
LAWS OF CHEMICAL COMBINATIONS contains same elements combined together in same proportion by
The combination of elements to form compounds is governed by mass. i.e, chemical compound has a fixed composition & it does
not depends on the method of its preparation or the source from
the following five basic laws.
which it has been obtained.
1. Law of Conservation of Mass/Law of Indestructibility Example: Carbon dioxide can be produced by different methods
of Matter such as burning of carbon, heating lime stone etc. It has been
Given by – Lavoisier observed that each sample of CO2 contains carbon and oxygen
combined in the ratio 3:8 by mass. This means that the composition
Tested by – Landolt of a compound always remain the same irrespective of the method
According to law of conservation of mass in all physical & by which it is prepared.
chemical changes total mass of the system remains constant.
3. Law of Multiple Proportions
In a physical or chemical change mass is neither be created nor
Given by → John Dalton
destroyed.
i.e. Total mass of the reactant = Total mass of the product. According to this law, if two elements can combine to form more
than one compound, the masses of one element that combine with
This relationship holds good when reactants are completely a fixed mass of the other element, are in the ratio of smallest whole
converted into products. numbers.
If reactants are not completely consume then the relationship will be:
Example: Carbon (C) can combine with oxygen (O) to form more
Total mass of reactant than one compound, namely CO, CO2. Here ratio of masses of O
= Total mass of product + Mass of unreacted reaction that combine with fixed mass of C is 16:32 or 1:2.

8 Dropper NEET
TRAIN YOUR BRAIN 5. Gay Lussac’s Law of Gaseous Volumes
Q. On analysis it was found that the black oxide of copper and Given by → Gay Lussac: He observed that when gases combine
the red oxide of copper contain 79.9% and 88.8% metal or are produced in a chemical reaction they do so in a simple
respectively. Establish the law of multiple proportions with ratio by volume provided all gases are at same temperature and
the help of this data. pressure.
Ans. In the black oxide, 79.9 g copper combines with (100 – 79.9), Example: 2H2(g) + O2(g) → 2H2O(g)
i.e. 20.1 g oxygen 100 ml 50 ml 100 ml
∴In red oxide 88.8 g copper will combine with 100 – 88.8 = 11.2 g 2 volumes 1 volume 2 volumes
∴ According to red oxide 79.9 copper will combine with vol of H2 : vol of O2 : vol of steam
11.2 × 79.9
= = 10.08 g oxygen 2 : 1 : 2
88.8
Thus the weights of oxygen that combine with the same 79.9 TRAIN YOUR BRAIN
g copper are 20.1 g and 10.08 g respectively. These are in the Q. For the gaseous reaction H2 + Cl2 → 2HCl
ratio 20.1 : 10.08 = 2 : 1 If 40 ml of hydrogen completely reacts with chlorine then
It is a simple whole number ratio. Hence, the law of multiple find out the required volume of chlorine and volume of
proportions is established. produced HCl ?
4. Law of Reciprocal Proportion Ans. According to Gay Lussac’s Law :
Given by → Richter: The ratio of the weights of two elements H2 + Cl2 → 2HCl
A and B that combine separately with fixed weight of the third 1 ml of H2 will react will 1 ml of Cl2 and 2 ml of HCl will be
element C is either the same or some simple multiple of this ratio produced.
of the weights in which A and B combine directly with each other.
∴ 40 ml of H2 will react with 40 ml of Cl2 and 80 ml of HCl
Example: The elements C and O combine separately with the will produce.
third element H to form CH4 and H2O and they combine directly
with each other to form CO2 as shown in the below figure. Required vol. of Cl2 = 40 ml
In CH4, 12 parts by weight of carbon Produced vol. of HCl = 80 ml.
combine with 4 parts by weight of H
6. Avogadro Law
hydrogen. In H2O, 2 parts by weight of CH4
hydrogen combine with 16 parts by weights H2O Given by → Amedeo Avogadro
of oxygen. Thus the weights of C and Avogadro proposed that equal volumes of gases at the same
O which combine with fixed weight of C O temperature and pressure should contain equal number of
CO2
hydrogen (say 4 parts of weight) are 12 and molecules.
32, i.e. they are in the ratio 12 : 32 or 3 : 8.
Example: 22.4 L of every gas at STP (Standard temperature and
Now in CO2, 12 parts by weight of carbon combine directly with Pressure, ie. T = 273 K, P = 1 atm) contains equal number of
32 parts by weight of oxygen, i.e. they combine directly in the molecules, which is equal to 6.022 × 1023
ratio 12 : 32 or 3 : 8 that is the same as the first ratio.
TRAIN YOUR BRAIN
TRAIN YOUR BRAIN Q. Which of the following contains the largest number of
Q. Copper sulphide contains 66.6% Cu, copper oxide contains oxygen atoms? 1.0 g of O atoms, 1.0 g of O2, 1.0 g of ozone
79.9% copper and sulphur trioxide contains 40% sulphur. O3.
Show that these data illustrate law of reciprocal proportions. Ans. All have the same number of oxygen atoms.
Ans. In copper sulphide,
Cu : S mass ratio is 66.6 : 33.4 DALTON’S ATOMIC THEORY
In sulphur trioxide, O : S mass ratio is 60 : 40 The assumption of Dalton’s Atomic theory are:
Now in copper sulphide 1. Matter consists of indivisible atoms.
33.4 parts of sulphur combines with Cu = 66.6 parts 2. All the atoms of a given element have identical properties
40.0 parts of sulphur combines with Cu. including identical mass. Atoms of different elements differ
66.6 × 40 in mass.
= = 79.8 parts 3. Compounds are formed when atoms of different elements
33.4
combine in a fixed ratio.
Now ratio of the masses of Cu and O which combines with 4. Chemical reactions involve reorganisation of atoms. These
same mass (40 parts) of sulphur separately is 79.8 : 60 are neither created nor destroyed in a chemical reaction.
Cu : O ratio by mass in CuO is 79.9 : 20.1 Dalton’s theory could explain the laws of chemical combination.
79.8 20.1 The main failures of Dalton’s atomic theory are:
Ratio I : Ratio II = × = 3 :1
60 79.9 1. It failed to explain how atoms of different elements differ
Which is simple whole number ratio. from each other i.e., did not tell anything about structure of
Hence, law of reciprocal proportion is proved. the atom.
Some Basic Concepts of Chemistry 9
2. It does not explain how and why atoms of different element
combine with each other to form compound.  KEY NOTE
3. It failed to explain the nature of forces present between Equivalent Mass (E.M)
different atoms in a molecule. Atomic mass
4. It fails to explain Gay Lussac’s law of combining volumes. Š E.M. of an element =
Valency
5. It did not make any difference between ultimate particle of
an element that takes part in reaction (atoms) and ultimate Molecular mass
Š E.M. of an acid =
particle that has independent existence (molecules). Basicity
ATOMIC MASS AND MOLECULAR MASS Molecular mass
Š E.M. of a base =
Atomic Mass Unit Acidity
1
It is defined as exactly th of the mass of a carbon-12 atom. It MOLE CONCEPT AND MOLAR MASSES
12
is represented as amu. [Now a new symbol ‘u’ called unified mass ‘Mole’ was introduced as the seventh base quantity for the amount
is used]. of substance in SI system.
12 1 One mole of a substance contains as many particles and
Mass=
of 1 amu × = 1.67 × 10–24 g
6.022 × 10 23
12 their number is equal to the number of particles in 12 g of
Today, ‘amu’ has been replaced by ‘u’ which is known as unified the 12C isotope. This number is known as avogardo constant
mass. (NA = 6.022 × 1023).

Average Atomic Mass Mole Concept in Gaseous Reaction


When we take into account the existence of the isotopes and their Molar volume is the mole related to volume of gaseous substance.
relative abundance (Percent occurrence), the average atomic mass The volume occupied by 1 mol of a gaseous substance is called
of that element is calculated. molar volume. 1 mole occupies 22.414 L or 22414 ml at STP ie.
Average atomic mass of an element is the sum of the masses of 273 K and 1 atm.
its isotopes each multiplied by its natural abundance. Volume
Number of moles =
Mathematically, average atomic mass of X (Ax) Molar volume
a1 x1 + a2 x2 + ..... + an xn Molar Mass
=
100
The mass of 1 mol of a substance in grams is called its molar
a1 = atomic mass ; x1 % occurence in nature
mass.
 KEY NOTE Mass-Mole-Number Relationship
Relative atomic mass is nothing but the number of nucleons Mass
present in the atom. Number of moles =
Molar massin g mol−1

TRAIN YOUR BRAIN TRAIN YOUR BRAIN


Q. Naturally occuring chlorine is 75% Cl35 which has an atomic Q. The molecular mass of H2SO4 is 98 amu. Calculate the
mass of 35 amu and 25% Cl37 which has a mass of 37 amu. number of moles of each element in 294 g of H2SO4.
Calculate the average atomic mass of chlorine -
(a) 35.5 amu (b) 36.5 amu Ans. Gram molecular mass of H2SO4 = 98 gm
(c) 71 amu (d) 72 amu 294
moles of H2SO4 = = 3 moles
Ans. (a) Average atomic mass = 98
% of Ι isotope × its atoms mass + % of ΙΙ isotope H2SO4 H S O
× its atomic mass
One molecule 2 atom one atom 4 atom
 100
1 × NA 2 × NA atoms 1 × NA atoms 4 × NA atoms
75 × 35 + 25 × 37
= 35.5 amu ∴ One mole 2 mole one mole 4 mole
100
∴ 3 mole 6 mole 3 mole 12 mole
Molecular Mass
It is the sum of atomic masses of the elements present in a PERCENTAGE COMPOSITION
molecule. It is obtained by multiplying the atomic mass of each We know that according to law of definite proportions any sample
element by the number of its atoms and adding them together. of a pure compound always possess constant ratio with their
combining elements.
Formula Mass Mass percentage of an element
In ionic compounds we use formula mass instead of molecular Mass of that element in the compound
mass. Formula mass of an ionic compound
= is the sum of the × 100
atomic masses of all atoms in a formula unit of compound. Molar mass of the compound

10 Dropper NEET
TRAIN YOUR BRAIN Element Sym- % of A.mu Relative Simplest Simple
bol ele- of ele- no. of atoms ratio whole no.
Q. Every molecule of ammonia always has formula NH3 ment ment atomic
irrespective of method of preparation or sources. i.e. 1 mole of ratio
ammonia always contains 1 mole of N and 3 mole of H. In other
words 17 gm of NH3 always contains 14 gm of N and 3 gm of 12.1 1.01
Carbon C 12.1 12 = 1.01 =1 1
H. Now find out % of each element in the compound. 12 1.01

Mass of N in 1 mol NH 3 16.2 1.01


Ans. Mass % of N in NH3 = × 100 Oxygen O 16.2 16 = 1.01 =1 1
Mass of 1 mol of NH 3 16 1.01
14 gm
= × 100 = 82.35 % 71.7 2.02
17 gm Chlorine Cl 71.7 35.5 = 2.02 =2 2
35.5 1.01
Mass of H in 1 mol NH 3
Mass % of H in NH3 = × 100
Mass of 1 mol e of NH 3 Then empirical formula = COCl2
3 Q. 1.615 g of anhydrous ZnSO4 was left in moist air. After a
= × 100 = 17.65 % few days its weight was found to be 2.875 g. What is the
17
molecular formula of hydrated salt?
Chemical Formula (AT. masses: Zn = 65.5, S = 32, O = 16, H = 1)
It is of two types: Ans. Molecular mass of anhydrous ZnSO4
= 65.5 + 32 + 4 × 16 = 161.5 g
(a) Empirical formula: It represent the simplest whole number
So, 1.615 g of anhydrous ZnSO4 combine with water
ratio of various atoms present in a compound. eg. EF of = 2.875 – 1.615 = 1.260 g
benzene (C6H6) is CH. 1.615 g of anhydrous ZnSO4 combine with water = 1.260 g
(b) Molecular formula: It shows the exact number of different 161.5 g of anhydrous ZnSO4 combine with
types of atoms present in a molecule of a compound. 1.260
= × 161.5 =
126g
eg., MF of benzene is C6H6 1.615
126
Determination of Chemical Formula No. of moles of water = =7
18
(a) Determination of empirical formula:
Hence, Formula is ZnSO4. 7H2O.
Step (I): Determination of percentage of each element
Step (II): Determination of mole ratio
STOICHIOMETRY AND STOICHIOMETRIC
CALCULATIONS
Step (III): Making it whole number ratio
Chemical Equation and Balanced Chemical Equation
Step (IV) : Simplest whole number ratio
Chemical Reaction: It is a process in which two or more than two
(b) Determination of Molecular Formula
substances interact with each other where old bonds are broken and
MF = (EF) × n; new bonds are formed.
Where n is a simple whole number. Chemical equation is a scientific method of representing a
Molecular formula = n × Empirical formula chemical change in terms of symbols and formula of reactants
and products involved in it.
= 2 × (C5H4) = C10H8
e.g., Zn + H2SO4 → ZnSO4 + H2

 KEY NOTE However, a balanced chemical equation tells us a lot of quantitative


information. Mainly the molar ratio in which reactants combine
Molecular weight and the molar ratio in which products are formed.
n=
Empirical weight
Features of a Balanced Chemical equation
(a) It contains a same number of atoms of each element on
TRAIN YOUR BRAIN both sides of equation.(POAC)
(b) It should follow law of charge conservation on both side.
Q. Phosgene, a poisonous gas used during World war-I, contains (c) Physical states of all the reagents/reactants should be
12.1% C, 16.2% O and 71.7% Cl by mass. What is the included in brackets.
empirical formula of phosgene. (d) All reagents/reactants should be written in thier standard
(a) COCl2  (b) COCl (c) CHCl3   (d) C2O2Cl4 forms (Molecular, Atomic, Solid etc.)
(e) The coefficients give the relative molar ratios of each
Ans. (a) reagent/reactant.
Some Basic Concepts of Chemistry 11
Stoichiometry deals with the calculation of masses and Mole-mole
sometimes volumes also of the reactants and products in a Mole relationship of Mole
reaction. The coefficients of reactants and products in a balanced equation

×2
chemical equation is called the stoichiometric coefficients.

ss
s
22
as

ma

2
.4
rm

.4
Lt
Steps:

Lt
ola
o la

M
÷M
1. Write the balanced chemical equation. Volume at STP Volume at STP
Mass Mass
2. See the number of moles of various reactants that take part
in the reaction and products formed.
3. Calculate the number of moles or amount of substance formed. TRAIN YOUR BRAIN
Interpretation of balanced chemical equations: Q. 367.5 gram KClO3 (M = 122.5) when heated how many
Once we get a balanced chemical equation then we can interpret gram of KCl and oxygen is produced.
a chemical equation by following ways: Ans. Balance chemical equation for heating of KClO3 is
● Mass - mass analysis 2KClO3 → 2KCl + 3O2
● Mass - volume analysis mass-mass ratio : 2 × 122.5 g : 2 × 74.5 g : 3 × 32 g
● Mole - mole analysis
mass of KClO3 2 × 122.5 367.5 122.5
Mass-mass Analysis = ⇒ =
   mass of KCl 2 × 74.5 W 74.5
In the following reaction W = 3 × 74.5 = 223.5 g
 According to stoichiometry  Mass of KClO3 2 × 122.5 367.5 2 × 122.5
Mass − mass ratio : 2 KClO3 → 2 KCl + 3O2  of the reaction 
2 ×122.5 2 × 74.5 3× 32
= ⇒ =
Mass of O 2 3 × 32 W 3 × 32
Mass of KClO3 2 × 122.5 Mass of KClO3 2 × 122.5 W = 144 g
or = = =
Mass of KCl 2 × 74.5 Mass of O2 3 × 32 Q. 367.5 g KClO3 (M = 122.5) when heated, how many litre of
oxygen gas is produced at STP
Mass–Volume Analysis mass of KClO3 2 × 122.5 367.5 2 × 122.5
Considering decomposition of KClO3 Ans. = ⇒ =
volume of O2 at STP 3 × 22.4 lt V 3 × 22.4 lt
2KClO3 → 2KCl + 3O2 V = 3 × 3 × 11.2 ⇒ V = 100.8 lt
mass volume ratio : 2 × 122.5 g : 2 × 74.5 g : 3 × 22.4 litre at STP
We can use two relation for volume of oxygen Limiting Reagent
Mass of KClO3 2 × 122.5 ● The reactant that gets consumed during the reaction &
= ...(i) limits the amount of product formed is known as the limiting
volume of O2 at STP 3 × 22.4 lt
reagent.
Mass of KCl 2 × 74.5 ● Limiting reagent is present in least stoichiometric amount
= ...(ii)
volume of O2 at STP 3 × 22.4 lt and therefore, controls amount of product.
● The remaining or leftout reactant is called the excess reagent.
Mole-mole Analysis ● If we are dealing with balance chemical equation then
This analysis is very much important for quantative analysis if number of moles of reactants are not in the ratio of
point of view. Consider again the decomposition of KClO3. stoichiometric coefficient of balanced chemical equation,
2KClO3 → 2KCl+ 3O2 then there should be one reactant which should be limiting
reactant.
In very first step of mole-mole analysis you should read
the balanced chemical equation like 2 moles KClO3 on TRAIN YOUR BRAIN
decomposition gives us 2 moles KCl and 3 moles O2 . and
from the stoichiometry of reaction we can write Q. Three moles of Na2CO3 are reacted with 6 moles of HCl
Moles of KClO3 Moles of KCl Moles of O2 solution. Find the volume of CO2 gas produced at STP. The
= = reaction is Na2 CO3 + 2HCl → 2 NaCl + CO2 + H2O
  2 2 3
Ans. From the reaction : Na2 CO3 + 2HCl → 2 NaCl + CO2 + H2O
Now for any general balance chemical equation like
aA+bB→cC+ dD given moles 3 mol 6 mol
You can write. given mole ratio 1 : 2
Moles of A reacted moles of B reacted Stoichiometric 1 : 2
= = coefficient ratio
a b
moles of C formed moles of D formed Given moles of reactant are in stoichiometric coefficient
= ratio therefore no reactant is left over.
c d
Mole-mole analysis to calculate V of CO2 produced at STP
 KEY NOTE Moles of Na2 CO3 Mole of CO2 Pr oduced
=
In fact mass-mass and mass-vol analysis are also interpreted in 1 1
terms of mole-mole analysis you can see in the following chart Moles of CO2 produced = 3
also. Volume of CO2 produced at STP = 3 × 22.4 L = 67.2 Lt
12 Dropper NEET
How to Find Limiting Reagent Mathematically : Molarity decreases as temperature increases.
Step: I Divide the given moles of reactant by the respective 1 1
Molarity ∝ ∝
stoichiometric coefficient of that reactant. temperature volume
Step: II See that for which reactant this division come out to If a particular solution having volume V1 and molarity M1 is
be minimum. The reactant having minimum value is diluted to V2 mL then
limiting reagent for you. M1V1 = M2V2
M2 : Resultant molarity
TRAIN YOUR BRAIN If a solution having volume V1 and molarity M1 is mixed with
another solution of same solute having volume V2 & molarity M2
Q. 6 moles of Na2CO3 and 4 moles of HCl are made to react.
then M1V1 + M2V2 = MR (V1 + V2)
Find the volume of CO2 gas produced at STP. The reaction
is M1V1 + M 2V2
MR = Resultant molarity =
Na2 CO3 + 2HCl → 2 NaCl + CO2 + H2O V1 + V2
Ans. From Step I & II Na2 CO3 HCl
TRAIN YOUR BRAIN
6 4
= 6= 2 ( division is minimum ) Q. 149 gm of potassium chloride (KCl) is dissolved in 10 Lt of
1 2 an aqueous solution. Determine the molarity of the solution
∴ HCl is limiting reagent (K = 39, Cl = 35.5)
From Step III
Ans. Molecular mass of KCl = 39 + 35.5 = 74.5 gm
Mole of HCl Moles of CO2 produced 149 gm
= =2
2 1 ∴ Moles of KCl =
74.5 gm
4 2
∴ mole of CO2 produced = = 2 ∴ Molarity of the solution = = 0.2 M
2 10
∴ volume of CO2 produced at S.T.P. = 2 × 22.4 = 44.8 lt. Molality
Reactions in Solutions It is defined as the number of moles of solute present in 1 kg of
solvent. it is denoted by m.
The concentration of a solution or the amount of substance present
No. of moles of solute
in its given volume can be expressed in any of the following ways. Thus, Molality(m) =
Mass of solvent in kg
1. Mass percent or weight percent (w/w%)
Molality is independent of temperature changes.
2. Mole fraction
There are other terms also used to express concentration of
3. Molarity solution
4. Molality
5. Normality Normality (N)
It is the number of gram equivalent of a solute dissolved per liter
Mass Percent
of the solution.
It is obtained by using the following relation:
No. of gram equivalents of solute
Mass of solute Normality ( N ) =
Mass percent = × 100 Vol. of solution in litres
Mass of solution
Mass of solute in gram
Mole Fraction =
Equivalent weight in gram × vol.of soltution in litres
It is no. of moles of a certain component to the total no. of moles
of the solution. Normality equation: N1V1 = N2 V2
No.of molesof A nA Wt. of ionic solute
Mole fraction of A = = Formality =
No.of molesof solution nA + nB Formula Wt. of solute × Vol. in lit.
Mole fraction is a pure number. It will remain independent of
temperature changes. TRAIN YOUR BRAIN
Molarity Q. 255 g of an aqueous solution contains 5 g of urea. What
It is defined as the number of moles of the solute in 1 liter of the is the concentration of the solution in terms of molality.
solution. It is denoted by M (Mol. wt. of urea = 60)
No.of molesof solute Ans. Mass of urea = 5 g
Molarity(M) =
Volume of solution in liters Molecular mass of urea = 60 g
Molarity is an unit that depends upon temperature. It varies Number of moles of urea = 0.083
inversely with temperature. Mass of solvent = (255 – 5) = 250 g
Some Basic Concepts of Chemistry 13
∴ Molality of the solution Q. 20 cm3 of an alcohol is dissolved in 80 cm3 of water.
Number of moles of solute Calculate the percentage of alcohol in solution.
× 1000 Ans. Volume of alcohol = 20 cm3
Mass of solvent in gram
Volume of water = 80 cm3
0.083 20
=
× 1000 = 0.332 m × 100 = 20.
250 ∴ percentage of alcohol =
20 + 80
Q. What is the concentration of sugar (C12 H22 O11) in mole
TRAIN YOUR BRAIN L–1 if its 20g are dissolved in enough water to make a final
volume upto 2L?
Q. 0.5 g of a substance is dissolved in 25 g of a solvent. Calculate Ans. Molarity of solution (mol L–1)
the percentage amount of the substance in the solution.
mass of solute (g) 1000
Ans. Mass of substance = 0.5 g = ×
M. Mass V in mL
Mass of solvent = 25 g
20 1000
∴ Percentage of the substance (w/w) =
0.5 conc. of sugar = × = 0.0292 mol L−1
× 100 342 2000
0.5 + 25
               = 1.96

14 Dropper NEET
Topicwise Questions

UNCERTAINTY IN MEASUREMENT 12. 6 g of carbon combines with 32 g of sulphur to form CS2,


1. The number of significant figures in 0.0045 are 12 g of C also combine with 32 g oxygen to form CO2. 10
g of sulphur combines with 10 g of oxygen to form Sulphur
(a) Two (b) Three (c) Four (d) Five
dioxide. Which law is illustrated by this?
2. Light travels with a speed of 3 × 108 m/sec. The distance (a) Law of multiple proportions
travelled by light in 1 Femto sec is:
(b) Law of constant composition
(a) 0.03 mm (c) Law of reciprocal proportions
(b) 0.003 mm (d) Gay Lussac’s law
(c) 3 mm
13. Which of the following data illustrates the law of conservation
(d) 0.0003 mm of mass?
3. Area of nuclear cross-section is measured in “Barn”. It is (a) 56 g of C reacts with 32 g of Oxygen to produce 44 g of
equal to: CO2
(a) 10–20 m2 (b) 10–30 m2 (b) 1.70 g of AgNO3 reacts with 100 ml of 0.1M HCl to
(c) 10–28 m2 (d) 10–14 m2 produce 1.435 g of AgCl and 0.63 g of HNO3
4. Two students X and Y report the mass of the same substance (c) 12 g of C is heated in vacuum and on cooling, there is no
as 7.0 g and 7.00 g respectively, which of the following change in mass
statement is correct? (d) 36 g of S reacts with 16 g of O2 to produce 48 g of SO2
(a) Both are equally accurate 14. One part of an element A combines with two parts of another
(b) X is more accurate than Y element B, 6 parts of element C combines with 4 parts of B.
(c) Y is more accurate than X If A and C combine together the ratio of their weights, will
be governed by
(d) Both are inaccurate scientifically
(a) law of definite proportion
5. The number of significant figures in value of π are:
(b) law of multiple proportion
(a) 1 (b) 2 (c) 3 (d) ∞
(c) law of reciprocal proportion
6. 5.041 has how many significant figures. (d) law of conservation of mass
(a) 1 (b) 2 (c) 3 (d) 4
15. The law of conservation of mass holds good for all of the
7. The correctly reported answer of the addition of 29.4406, 3.2 following except.
and 2.25 will have significant figures: (a) All chemical reactions (b) Nuclear reaction
(a) 3 (b) 4 (c) 2 (d) 5 (c) Endothermic reactions (d) Exothermic reactions
8. What is the area of rectangle which is 12.34 cm wide and 16. The % of copper and oxygen in samples of CuO obtained by
1.23 cm long? different methods were found to be the same. This proves
(a) 15.2 m2 (b) 15.18 cm2 the law of:
(c) 16.2 cm­2 (d) 16.2 m2 (a) Constant Proportion (b) Reciprocal Proportion
9. If an object has a mass of 0.2876 g, then find the mass of (c) Multiple Proportion (d) Conservation of mass.
nine such objects: 17. Two elements X and Y combine in gaseous state to form
(a) 2.5884 g (b) 2.5886 g (c) 2.588 g (d) 2.5 g XY in the ratio 1:35.5 by mass. The mass of Y that will be
10. The value of Plank’s constant is 6.62618 × 10–34 Js. The required to react with 2 g of X is:
number of significant figures in it is (a) 7.1 g (b) 3.55 g
(a) Six (b) Five (c) 71 g (d) 35.5 g
(c) Three (d) Thirty four 18. 4.4 g of an oxide of nitrogen gives 2.24 L of nitrogen and
60 g of another oxide of nitrogen gives 22.4 L of nitrogen at
LAW OF CHEMICAL COMBINATIONS S.T.P. The data illustrates:
11. In Habers process, the volume at S.T.P of ammonia relative (a) Law of conservation of mass
to the total volume of reactants at STP is : (b) Law of constant proportions
(a) One fourth (b) One half (c) Law of multiple proportions
(c) Same (d) Three fourth (d) Law of reciprocal proportions

Some Basic Concepts of Chemistry 15


19. The law of multiple proportions is illustrated by the two 30. 10 grams of each O2, N2 and Cl2 are kept in three bottles. The
compounds correct order of arrangment of bottles containing decreasing
(a) Sodium chloride and sodium bromide number of Molecules.
(b) Ordinary water and heavy water (a) O2, N2, Cl2 (b) Cl2, N2, O2
(c) Caustic soda and caustic potash (c) Cl2, O2, N2 (d) N2, O2, Cl2
(d) Sulphur dioxide and sulphur trioxide. 31. Avogadro’s number is the number of molecules present in
20. How many grams of H3PO4 is required to completely (a) 1 g of molecule (b) 1atom of molecule
neutralize 120g of NaOH (c) gram molecular mass (d) 1 litre of molecule
(a) 49 (b) 98 (c) 196 (d) 9.8 32. Maximum number of atoms are present in
21. The weight of oxygen required to completely react with 27 gm (a) 14 gms. of carbon monoxide
of ‘Al’ is (b) 2 gms. of hydrogen
(a) 8 gm (b) 16 gm (c) 32 m (d) 24 gm (c) 11.2 lit. of nitrogen at STP
22. If law of conservation of mass was to hold true, then 20.8 g (d) 1.5 gm atoms of helium
of BaCl2 on reaction with 9.8 g of H2SO4 will produce 7.3 g
33. One amu is equal to
of HCl and BaSO4 equal to:
(a) 1.66 × 10–8 g (b) 1.66 × 10–4 g
(a) 11.65 g (b) 23.3 g (c) 25.5 g (d) 30.6 g
(c) 1.66 × 10–16 g (d) 1.66 × 10–24 g
23. One of the following combinations which illustrates the law
of reciprocal proportions is: 34. The number of molecules present in one milli litre of a gas at
(a) N2O3, N2O4, N2O5 (b) NaCl, NaBr, NaI STP is known as
(c) CS2, CO2, SO2 (d) PH3, P2O3, P2O5 (a) Avogadro number
(b) Boltzman number
24. 23g of sodium will react with ethyl alcohol to give
(c) Loschmidt number
(a) 1 mole of H2 (b) 1/2 mole of H2
(d) Universal gas constant
(c) 1 mole of O (d) 1 mole of NaOH
35. Which of the following gases contain the same number of
25. Hydrogen and oxygen combine to form H2O2 and H2O
containing 5.93% and 11.2% hydrogen respectively, the data molecules as that of 16 grams of oxygen?
illustrates: (a) 16gm of O3 (b) 32 grams of SO2
(a) Law of conservation of mass (c) 16gm of SO2 (d) All
th
(b) Law of Constant proportions 1
36. If the atomic mass unit ‘u’ were defined to be of the
(c) Law of reciprocal proportions 5
(d) Law of multiple proportions mass of an atom of C-12, what would be the atomic weight
of nitrogen in amu or ‘u’ in this state? Atomic weight of N
26. Two elements X (of mass 16) and Y (of mass 14) combine to
on conventional scale is 14:
form compounds A, B and C. The ratio of different masses of
Y which combine with a fixed mass of X in A, B and C is 1 : 3 (a) 6.77 u (b) 5.834 u
: 5, if 32 parts by mass of X combines with 84 parts by mass of (c) 14 u (d) 23 u
Y in B, then in C, 16 parts by mass of X will combine with; 37. A 100 g sample of Haemoglobin on analysis was found to
(a) 14 parts by mass of Y (b) 42 parts by mass of Y contain 0.34% Fe by mass. If each haemoglobin molecule
(c) 70 parts by mass of Y (d) 84 parts by mass of Y has four Fe2+ ions, the molecular mass of haemoglobin is
(Fe = 56 amu)
ATOMIC AND MOLECULAR MASSES (a) 77099.9 g (b) 12735 g
27. Insulin contains 3.4% sulphur by mass. What will be the (c) 65882 g (d) 96359.9 g
minimum molecular weight of insulin?
(a) 94.117 u (b) 1884 u MOLE CONCEPT AND MOLAR MASSES
(c) 941 u (d) 976 u 38. 1 g-atom of nitrogen represents:
28. If we assume 1/24 th part of mass of carbon instead of 1/12 (a) 6.02 × 1023 N2 molecules
th part of it as 1 amu., mass of 1 mole of a substance will (b) 22.4 L of N2 at S.T.P
(a) Remain unchanged (b) get doubled (c) 11.2 L of N2 at S.T.P
(c) Get halved (d) can’t be predicted (d) 28 g of nitrogen
29. Boron has two isotopes 10B and 11B whose relative 39. Which is correct for 10 g of CaCO3?
abundances are 20% and 80% respectively. Atomic weight (a) It contains 1 g atom of carbon
of Boron is (b) It contains 0.3 g atoms of oxygen
(a) 10 (b) 11 (c) It contains 12 g of calcium
(c) 10.5 (d) 10.8 (d) It refers to 0.1 g equivalent of CaCO3

16 Dropper NEET
40. The number of oxygen atoms present in 14.6 g of magnesium 51. Out of 1.0 g dioxygen, 1.0 g (atomic) oxygen and 1.0 g
bicarbonate is: ozone, the maximum number of oxygen atoms are contained
(a) 6 NA (b) 0.6 NA (c) NA (d) N A in:
2 (a) 1.0 g of atomic oxygen
41. Which of the following has the highest mass? (b) 1.0 g of ozone
(a) 20 g of sulphur (c) 1.0 g of oxygen gas
(b) 4 mol of carbon dioxide (d) All contain same number of atoms
(c) 12 × 1024 atoms of hydrogen
52. The maximum volume at S.T.P. is occupied by:
(d) 11.2 L of helium at N.T.P.
(a) 12.8 g of SO2 (b) 6.02 × 1022 molecules of CH4
42. If isotopic distribution of C-12 ad C-14 is 98% and 2%
(c) 0.5 mL of NO2 (d) 1g molecule of CO2
respectively, then the number of C-14 atoms in 12 g of carbon
is: 53. If NA is Avogadro’s number, then the number of oxygen
(a) 1.032 × 1022 (b) 3.01 × 1022 atoms in one g-equivalent of oxygen is:
(c) 5.88 × 1023 (d) 6.02 × 1023 (a) NA (b) NA/2
43. 5.6 L of a gas at S.T.P. weights equal to 8 g. The vapour (c) NA/4 (d) 2NA
density of gas is: 54. If 224 ml. of a triatomic gas has a mass of 1g at 273 K and 1
(a) 32 (b) 16 (c) 8 (d) 40 atm pressure, then the mass of one atom is:
44. One atom of an element weighs 1.8 × 10–22 g, its atomic mass (a) 8.30 × 10–23 g (b) 6.24 × 10–23
is: (c) 2.08 × 10–23 g (d) 5.54 × 10–23 g
(a) 29.9 g (b) 18 g 55. The rest mass of an electron is 9.11 × 10–31 kg. Molar mass
(c) 108.36 g (d) 154 g of the electron is:
45. If H2SO4 ionises as H2SO4 + 2H2O →2H3O+ + SO24– . Then (a) 1.5 × 10–31 kg mol–1 (b) 9.11 × 10–31 kg mol–1
total number of ions produced by 0.1 mol H2SO4 will be: (c) 5.5 × 10–7 kg mol–1 (d) 6.02 × 1023 kg mol–1
(a) 9.03 × 1021
56. A sample of ammonium phosphate, (NH4)3 PO4, contains
(b) 3.01 × 1022 3.18 moles of hydrogen atoms. The number of moles of
(c) 6.02 × 1022 oxygen atoms in the sample is:
(d) 1.8 × 1023 (a) 0.265 (b) 0.795
46. Which of the following will not have a mass of 10 g? (c) 1.06 (d) 3.18
(a) 0.1 mol CaCO3. (b) 1.51 × 1023 Ca2+ ions 57. What is the total number of atoms present in 25.0 mg of
(c) 0.16 mol of CO32- ions (d) 7.525 × 1022 Br atom camphor, C10H16O?
47. xL of N2 at S.T.P. contains 3 × 1022 molecules. The number (a) 9.89 × 1019 (b) 6.02 × 1020
of molecules in x/2 L of ozone at S.T.P. will be: (c) 9.89 × 10 20 (d) 2.67 × 1021
(a) 3 × 1022 (b) 1.5 × 1022 58. 4.0 g of caustic soda (NaOH) (mol mass 40) contains same
(c) 1.5 × 1021 (d) 1.5 × 1011 number of sodium ions as are present in-
48. A person adds 1.71 gram of sugar (C12H22O11) in order to (a) 10.6 g of Na2CO3 (mol. mass 106)
sweeten his tea. The number of carbon atoms added are: (b) 58.5 g of NaCl (Formula mass 58.5)
(mol mass of sugar = 342)
(c) 100 ml of 0.5 M Na2SO4 (Formula mass 142)
(a) 3.6 × 1022 (b) 7.2 × 1021
(d) 1mol of NaNO3 (mol. mass 85)
(c) 0.05 (d) 6.6 × 1022
59. Total number of atoms present in 64 gm of SO2 is -
49. The number of atoms present in 0.1 mole of P4 (At. mass =
31) are: (a) 2 × 6.02 × 1023 (b) 6.02 × 1023
(a) 2.4 × 1024 atoms (c) 4 × 6.02 × 1023 (d) 3 × 6.02 × 1023
(b) Same as in 0.05 mol of S8 60. The total number of protons, electrons and neutrons in 12gm
(c) 6.02 × 1022 atoms of 6C12 is -
(d) Same as in 3.1g of phosphorus (a) 1.084 × 1025 (b) 6.022 × 1023
50. Which one contains maximum number of molecules? (c) 6.022 × 1022 (d) 18
(a) 2.5 g molecule of N2 61. Number of Ca+2 and Cl– ion in 111 g of anhydrous CaCl2
(b) 4 g atom of nitrogen are -
(c) 3.01 × 1024 atoms (a) NA, 2NA (b) 2NA, NA
(d) 82 g of dinitrogen (c) NA, NA (d) None

Some Basic Concepts of Chemistry 17


62. The maximum volume at N.T.P. is occupied by 72. A compound having the empirical formula (C3H4O) has
(a) 12.8 gm of SO2 a molecular mass of 170 ± 5. The molecular formula of it
(b) 6.02 × 1022 molecules of CH4 compound is:
(c) 0.5 mol of NO2 (a) C3H4O (b) C6H8O2
(d) 1 gm-molecule of CO2 (c) C6H12O3 (d) C9H12O3
63. Number of moles of water in 488 g of BaCl2.2H2O are - (Ba 73. Two oxides of a metal contains 50% and 40% metal (M)
= 137) respectively. If the formula of first oxide is MO2, the formula
(a) 2 moles (b) 4 moles (c) 3 moles (d) 5 moles of second oxide will be:
(a) MO2 (b) MO3
64. 4.4 g of CO2 and 2.24 litre of H2 at STP are mixed in a
container. The total number of molecules present in the (c) M2O (d) M2O5
container will be - 74. The vapour density of gas A is four times that of B. If
(a) 6.022 × 1023 (b) 1.2044 × 1023 molecular mass of B is M, then molecular mass of A is:
(c) 2 moles (d) 6.023 × 1024 (a) M (b) 4 M
65. One mole of nitrogen gas has volume equal to M
(c) (d) 2 M
(a) 1 litre of nitrogen at S.T.P. 4
(b) 32 litre of nitrogen at S.T.P. 75. A metal nitride M3N2 contains 28% of nitrogen. The atomic
(c) 22.4 litre of nitrogen at S.T.P. mass of metal M is:
(d) 11.2 litre of nitrogen at S.T.P. (a) 24 (b) 54

PERCENTAGE COMPOSITION (c) 9 (d) 87.62

66. The percentage of C, H and N in an organic compound are 76. A container of volume V, contains 0.28 g of N2 gas. If same
40 %, 13.3% and 46.7% respectively then empirical volume of an unknown gas under similar conditions of
formula is temperature and pressure weights 0.44 g, the molecular mass
of gas is:
(a) C3H13N3 (b) CH2N
(c) CH4N (d) CH6N (a) 22 (b) 44
(c) 66 (d) 88
67. B1 g of an element gives B2 g of its chloride, the equivalent
mass of the element is: 77. A gaseous hydrocarbon on complete combustion gives
3.38 g of CO2 and 0.690 g of H2O and no other products. The
(a) B1 (b) B2
´35.5 ´35.5 empirical formula of hydrocarbon is:
B2 - B1 B2 - B1
(a) CH (b) CH2
(c) B2 - B1 ´35.5 (d) B2 - B1 ´35.5 (c) CH3 (d) The data is not complete
B1 B2
78. The percentage of Carbon in CO2 is
68. 60 g of a compound on analysis gave 24 g C, 4 g H and 32 g
O. The empirical formula of the compound is: (a) 27.27% (b) 29.27%
(a) C2H4O2 (b) C2H2O2 (c) 30.27% (d) 26.97%
(c) CH2O2 (d) CH2O 79. The haemoglobin form red blood corpuscles of most
69. A compound made of two elements A and B are found to mammals contain approximately 0.33% of iron by mass.
contain 25% A (at mass 12.5) and 75% B (at mass 37.5). The The molecular mass of haemoglobin is 67200. The number
simplest formula of the compound is: of iron atoms in each molecule of haemoglobin is:
(a) AB (b) AB2 (a) 3 (b) 4
(c) AB3 (d) A3B (c) 2 (d) 6
70. 400 mg of capsule contains 100 mg of ferrous fumarate. The 80. On analysis a certain compound was found to contain iodine
percentage of Fe present in the capsule is approximately: and oxygen in the ratio of 254 g of iodine (at mass 127) and
(formula of ferrous fumarate is (CHCOO)2 Fe). 80 g oxygen (at mass 16). What is the formula of compound?
(a) 8.2% (b) 25% (a) IO (b) I2O
(c) 16% (d) Unpredictable (c) I5O3 (d) I2O5
71. Simplest formula of compound containing 50% of element 81. 0.5 mol of potassium ferrocyanide contains carbon equal to:
X (at mass 10) and 50% of element Y (at mass 20) is: (Formula of potassium ferrocyanide is K4[Fe(CN)6].
(a) XY (b) X2Y (a) 1.5 mol (b) 36 g
(c) XY2 (d) X2Y3 (c) 18 g (d) 3.6 g

18 Dropper NEET
82. 14 g of element X combine with 16g of oxygen. On the basis of 90. The molarity of pure water is
this information, which of the following is a correct statement : (a) 100 M (b) 55.6M
(a) The element X could have an atomic weight of 7 and its (c) 50 M (d) 18 M
oxide formula XO
91. The mass of 70% H2SO4 by mass is required for neutralisation
(b) The element X could have an atomic weight of 14 and its of 1 mole of NaOH is:
oxide formula X2O
(a) 65 (b) 98 (c) 70 (d) 54
(c) The element X could have an atomic weight of 7 and its
oxide is X2O 92. If potassium chlorate is 80% pure then 48 g of oxygen would
be produced from:
(d) The element X could have an atomic weight of 14 and its
oxide is XO2 (a) 153.12 g of KClO3
(b) 120 g of KClO3
83. A compound has 20% of nitrogen by weight. If one molecule
of the compound contains two nitrogen atoms, the molecular (c) 20 g of KClO3
weight of the compound is (d) 90 g of KClO3
(a) 35 (b) 70 (c) 140 (d) 280 93. Density of a solution containing x% by mass of H2SO4 is y.
The normality is
STOICHIOMETRY & STOICHIOMETRIC
xy × 10 xy × 10
CALCULATION (a) (b) ×2
98 98 y
84. ‘X’ litres of carbon monoxide is present at STP. It is
xy × 10 x × 10
completely oxidized to CO2. The volume of CO2 formed is (c) ×2 (d)
11.207 litres at STP. What is the value of ‘X’ in litres? 98 98 y

(a) 22.414 94. Mass percentage (w/w) of ethylene glycol (HOCH2 -


(b) 11.207 CH2OH) in a aqueous solution is 20, then mole fraction of
(c) 5.6035 solute is
(d) 44.828 (a) 0.5 (b) 0.067

85. The moles of O2 required for reacting with 6.8 g ammonia. (c) 0.1 (d) 0.4
(.....NH3+....O2→.....NO+.....H2O) is: 95. Number of gram equivalents of solute in 100 ml of 5 N HCl
(a) 5 (b) 2.5 solution is
(c) 1 (d) 0.5 (a) 50 (b) 500 (c) 5 (d) 0.5
86. What mass of CaCl2 in grams would be enough to produce 96. If 1.26 grams of oxalic acid is dissolved in 250 ml of solution
14.35 gm of AgCl? then its normality is
(a) 5.55 g (b) 8.29 g (a) 0.05 (b) 0.04 (c) 0.02 (d) 0.08
(c) 16.59 g (d) 10 g 97. 100ml of ethyl alcohol is made upto a litre with distilled
87. What weight of sodium hydroxide is required to neutralise water. If the density of C2H5OH is 0.46 gm/ml. Then its
100 ml of 0.1N HCl ? molality is
(a) 4 g (b) 0.4 g (a) 0.55 m (b) 1.11m
(c) 0.04 g (d) 40 g (c) 2.22 m (d) 3.33m
88. H2O2 is sold as a solution of approximately 5.0 g H2O2 98. A solution of 0.1 mole of a metal chloride MClx required 500
per 100 ml of the solution. The molarity of this solution is mL of 0.6 molal AgNO3 solution for complete ppt. The value
approximately of x is:
(a) 0.15 M (a) 5 (b) 4
(b) 1.5 M (c) 3 (d) 1
(c) 3.0 M 99. If 20 g of CaCO3 is treated with 100 ml 20% HCl solution.
(d) 3.4 M The amount of CO2 produced is:
89. The amount of oxalic acid (eq.wt.63) required to prepare (a) 22.4l g (b) 8.8 g
500 ml of its 0.10 N solution is (c) 2.2 g (d) 8 l
(a) 0.315 g 100. The mass of CaCO3 required to react with 25 mL of 0.75
(b) 3.150 g molar HCl is:
(c) 6.300 g (a) 0.94 g (b) 0.68 g
(d) 63.00 g (c) 0.76 g (d) 0.52 g

Some Basic Concepts of Chemistry 19


101. 2 moles of H2S and 11.2 L of SO2 at N.T.P. reacts to form x 103. A sample of pure compound contains 1.15 g of sodium,
moles of sulphur. The value of x is: 3.01 × 1022 atoms of carbon and 0.1 mol of oxygen atom. Its
(a) 1.5 (b) 3.5 empirical formula is:
(a) Na2CO3 (b) NaCO2 (c) Na2CO (d) Na2CO2
(c) 7.8 (d) 12.7
102. Sulphuryl chloride (SO2Cl2) reacts with H2O to give a 104. If 0.30 mol of zinc are added to 0.52 mol of HCl, the moles
mixture of H2SO4 & HCl. Aqueous solution of 1 mole of H2 formed are:
SO2Cl2 will be neutralised by: (a) 0.52 (b) 0.30 (c) 0.26 (d) 0.60
(a) 3 moles of NaOH 105. The specific gravity of 98% H2SO4 is 1.8 g/cc. 50 ml of this
(b) 2 moles of Ca(OH)2 solution is mixed with 1750 ml of pure water. Molarity of
(c) Both (a) & (b) resulting solution is
(d) None of these (a) 0.2 M (b) 0.5 M (c) 0.1 M (d) 1 M

Learning Plus

1. Which of the following is/are not affected by temperature? 5. What will be the molarity of a solution, which contains 5.85
(a) Molarity g of NaCl (s) per 500 mL?
(b) Molality (a) 4 mol L–1
(c) Normality (b) 20 mol L–1
(d) None of these (c) 0.2 mol L–1
2. Ferric sulphate on heating gives sulphur trioxide. The ratio (d) 2 mol L–1
between the weights of oxygen and sulphur present in SO3
6. Number of atoms in 55.85 gram Fe (at. wt. of Fe = 55.85 g mol–1)
obtained by heating 1 kg of ferric sulphate is
is
(a) 2 : 3 (b) 1 : 3
(a) Twice that 60 g carbon
(c) 3 : 1 (d) 3 : 2
(b) 6.023 × 1022
3. The number of atoms present in 4.25 grams of NH3 is
approximately (c) Half that in 8g He
(d) 5558.5 × 6.023 × 1023
(a) 1 × 1023
7. Neon has two isotopes Ne20 and Ne22. If atomic weight of
(b) 8 × 1020 Neon is 20.2, the ratio of the relative abundances of the
(c) 2 × 1023 isotopes is
(d) 6.02 × 1023 (a) 1 : 9 (b) 9 : 1
4. Two students performed the same experiment separately and (c) 70 % (d) 80 %
each one of them recorded two readings of mass which are 8. The total weight of 1022 molecular units of CuSO4 . 5H2O is
given below. Correct reading of mass is 3.0 g. On the basis
nearly
of given data, mark the correct option out of the following
statements: (a) 4.144 g (b) 5.5 g
(c) 24.95 g (d) 41.45 g
Students Readings
9. The number of Cl– and Ca+2 ions in 222g. of CaCl2 are
(i) (ii)
A 3.01 2.99 (a) 4N, 2N (b) 2N, 4N
B 3.05 2.95 (c) 1N, 2N (d) 2N, 1N
10. The empirical formula of a gaseous compound is ‘CH2’.
(a) Results of both the students are neither accurate nor
The density of the compound is 1.25 gm/lit. at S.T.P. The
precise
molecular formula of the compound is ‘X’
(b) Results of student A are both precise and accurate
(a) C2H4 (b) C3H6
(c) Results of student B are neither precise nor accurate
(d) Results of student B are both precise and accurate (c) C6H12 (d) C4H8

20 Dropper NEET
11. If 500 mL of a 5 M solution is diluted to 1500 mL, what will 22. From 320 mg. of O2, 6.023 ×1020 molecules are removed,
be the molarity of the solution obtained? the no. of moles remained are
(a) 1.5 M (b) 1.66 M (a) 9 × 10–3 moles (b) 9 × 10–2 moles
(c) 0.017 M (d) 1.59 M (c) Zero (d) 3 × 10–3 moles
12. The number of atoms present in one mole of an element is 23. An oxide of nitrogen has a molecular weight 92. Find the
equal to Avogadro number. Which of the following element total number of electrons in one gram mole of that oxide.
contains the greatest number of atoms? (a) 4.6 N (b) 46 N
(a) 4 g He (b) 46 g Na (c) 23 N (d) 2.3 N
(c) 0.40 g Ca (d) 12 g He 24. No. of moles of water in 488.6 gms of BaCl2.2H2O are
(molecular weight of BaCl2.2H2O=244.33)
13. The empirical formula of an organic compound is CH2O. Its
(a) 2 moles (b) 4 moles (c) 3 moles (d) 5 moles
vapour density is 45. The molecular formula of the compound
25. One mole of any substance contains 6.022 × 1023 atoms/
is
molecules. Number of molecules of H2SO4 present in 100 mL
(a) CH2O (b) C2H4O2 of 0.02 M H2SO4 solution is:
(c) C3H6O3 (d) C6H12O6 (a) 12.044 × 1020 molecules (b) 6.022 × 1023 molecules
14. 0.132 g of an organic compound gave 50 ml of N2 at NTP. The (c) 1 × 1023 molecules (d) 12.044 × 1023 molecules
weight percentage of nitrogen in the compound is close to 26. Given the numbers, 161 cm, 0.161 cm, 0.0161 cm. The
(a) 15 (b) 20 number of significant figures for the three numbers is:
(c) 48.9 (d) 47.34 (a) 3, 4 and 5, respectively (b) 3, 4 and 4, respectively
15. 0.7 moles of potassium sulphate is allowed to react with 0.9 (c) 3, 3 and 4, respectively (d) 3, 3 and 3, respectively
moles of barium chloride in aqueous solutions. The number 27. A certain compound contains magnesium, carbon and Nitrogen
of moles of the substance precipitated in the reaction is in the mass ratio 12 : 12 : 14. The formula of the compound is
(a) 1.4 moles of potassium chloride (a) MgCN (b) Mg2CN
(b) 0.7 moles of barium sulphate (c) MgCN2 (d) Mg(CN)2
(c) 1.6 moles of potassium chloride 28. An oxide of nitrogen contains 36.8% by weight of nitrogen.
The formula of the compound is
(d) 1.6 moles of barium sulphate
(a) N2O (b) N2O3 (c) NO (d) NO2
16. The number of moles of Fe2O3 formed when 0.5 moles of O2
29. 40 ml. of a hydrocarbon undergoes combustion in 260 ml
and 0.5 moles of Fe are allowed to react are
of oxygen and gives 160 ml of carbon dioxide. If all gases
(a) 0.25 (b) 0.5 (c) 1/3 (d) 0.125 are measured under similar conditions of temperature and
17. Amount of oxalic acid required to prepare 250ml of N/10 pressure, the formula of hydrocarbon is
solution (MW of oxalic acid = 126) is (a) C3H8 (b) C4H8 (c) C6H14 (d) C4H10
(a) 1.5759 g (b) 3.15 g (c) 15.75 g (d) 63.0 g 30. The mass of Hydrogen at S.T.P. that is present in a vessel
which can hold 4 grams of oxygen under similar conditions
18. If the concentration of glucose (C6H12O6) in blood is 0.9 g
is
L–1, what will be the molarity of glucose in blood?
(a) 1 gram (b) 0.5 grams
(a) 5 M (b) 50 M
(c) 0.25 gms. (d) 0.125 gm
(c) 0.005 M (d) 0.5 M
31. Which of the following solutions has the highest normality?
19. What will be the molality of the solution containing 18.25 g (a) 172 milli equivalents in 200 ml
of HCl gas in 500 g of water?
(b) 84 milli equivalents in 100 ml
(a) 0.1 m (b) 10 m (c) 0.5 m (d) 1 m (c) 275 milli equivalents in 250 ml
20. Increasing order of number of moles of the species (d) 43 milli equivalents in 60 ml
(i) 3 grams of NO 32. What volume of 75 % H2SO4 by mass is required to prepare
(ii) 8.5 grams of PH3 and 1.5 litres of 0.2 M H2SO4? (Density of the sample is 1.8 g/
(iii) 8 grams of methane is cc)
(a) (i) < (ii) < (iii) (b) (iii) < (ii) < (i) (a) 14.2cc (b) 28.4cc
(c) 21.7cc (d) 7.1 cc
(c) (i) < (iii) < (ii) (d) (ii) < (iii) < (i)
33. The empirical formula and molecular mass of a compound
21. The number of molecules present in 1.12 × 10–7 cc of a gas
are CH2O and 180 g respectively. What will be the molecular
at STP is formula of the compound?
(a) 6.02 × 1023 (b) 3.01 × 1012 (a) C9H18O9 (b) CH2O
(c) 6.02 × 1012 (d) 3.01 × 1023 (c) C6H12O6 (d) C2 H4O2
Some Basic Concepts of Chemistry 21
34. If the density of a solution is 3.12 g mL–1, the mass of 1.5 mL (b) Total mass of reactants = total mass of product, therefore,
solution in significant figures is: law of multiple proportions is followed
(a) 4.7 g (b) 4680 × 10–3 g (c) Amount of Fe2O3 can be increased by taking any one of
(c) 4.680 g (d) 46.80 g the reactants (iron or oxygen) in excess
35. 4.9 grams of H2SO4 is present is 100 ml of the solution, then (d) Amount of Fe2O3 produced will decrease if the amount
its molarity and normality are
of any one of the reactants (iron or oxygen) is taken in
(a) 1, 0.5 (b) 1, 1
excess
(c) 0.5, 1 (d) 0.5, 2
36. In order to prepare one litre normal solution of KMnO4 , 39. Which of the following statements indicates that law of
how many grams of KMnO4 required if the solution is to be multiple proportion is being followed?
used in acidic medium for oxidation (a) Sample of carbon dioxide taken from any source will
(a) 158 (b) 79 (c) 31.6 (d) 790 always have carbon and oxygen in the ratio 1 : 2
37. 50 gm of sample of sodium hydroxide required for complete (b) Carbon forms two oxides namely CO2 and CO, where
neutralisation, 1L 1N HCl. What is the percentage purity of masses of oxygen which combine with fixed mass of
NaOH is
carbon are in the simple ratio 2 :1
(a) 50    (b) 60 (c) 70   (d) 80
(c) When magnesium burns in oxygen, the amount of
38. Which of the following statements is correct about the
magnesium taken for the reaction is equal to the amount
reaction given below?
of magnesium in magnesium oxide formed
4Fe(s) + 3O2(g) → 2Fe2O3(g)
(d) At constant temperature and pressure, 200 mL of
(a) Total mass of iron and oxygen in reactants = total mass
of iron and oxygen in product therefore it follows law of hydrogen will combine with 100 mL oxygen to produce
conservation of mass 200 mL of water vapour

Multiconcept MCQs

1. In HABER’s process, 30 L of H2 and 30 L of N2 were taken for 4. In an organic compound of molar mass 108 g mol–1 C, H
a reaction which yielded only 50% of the expected product. and N atoms are present in 9 : 1 : 3.5 by weight. Molecular
What will be the composition of the gaseous mixture under formula can be –
these conditions? (a) C6H8N2 (b) C7H10N
(a) 10 L NH3, 25 L N2, 15 L H2 (c) C5H6N3 (d) C4H18N3
(b) 20 L NH3, 10 L N2, 3 L H2 5. 6.02 × 1020 molecules of urea are present in 100 mL of its
(c) 20 L NH3, 25 L N2, 15 L H2 solution. The concentration of urea solution is -
(d) None of these (a) 0.001 M (b) 0.01 M
2. Diborane (B2H6) can be prepared by the following reaction- (c) 0.02 M (d) 0.1 M

3NaBH 4 + 4BF3 
→ 3NaBF4 + 2B2 H 6 6. What is the empirical formula of a compound composed of
O & Mn in equal weight ratio? (At. wt of Mn = 55)
If the reaction has a 70% yield, how many moles of NaBH4 (a) MnO (b) MnO2
should be used with excess BF3 in order to obtain 0.200 mol
(c) Mn2O3 (d) Mn2O7
of B2H6?
(a) 0.21 moles (b) 0.429 moles 7. 1 g sample of alkaline earth metal react completely with 4.08
g H2SO4 and yields an ionic product MSO4. Then find out
(c) 0.300 mol (d) 0.175 moles
the atomic mass of alkaline earth metal (M)?
3. An ore contains 1.24% of mineral argentate, Ag2S by mass. (a) 9 (b) 24
How many grams of this ore would have to be processed in
(c) 40 (d) 87
order to obtain 1g of pure solid silver?
8. A compound contains 36% C by mass. If each molecule
(a) 92.6 g
contains two C atoms the number of moles of the compound
(b) 88.1 g in its 10 g is/are-
(c) 101.11 g (a) 0.15 (b) 1.5
(d) 107.25 g (c) 150 (d) 1500

22 Dropper NEET
9. When a certain amount of octane is burnt completely, 16. 3.68 g of mixture of CaCO3 and MgCO3 is heated to liberate
7.04 g of CO2 is formed. What mass of H2O is formed 0.04 mole of CO2. The mole % of CaCO3 and MgCO3 in the
simultaneously? mixture is respectively:
(a) 3.24 g (b) 6.68 g (a) 50%, 50% (b) 60%, 40%
(c) 6.48 g (d) 6.16 g (c) 40%, 60% (d) 30%, 70%
10. A 1.50 g sample of KHCO3 having 80% purity is strongly
17. An element is found in nature in two isotopic forms with
heated. Assuming the impurity to be thermally stable, the
mass numbers (A–1) and (A + 3). If the average atomic mass
loss in weight of the sample, on heating is:
of the element is found to be A, then the relative abundance
(a) 26.4 g (b) 2.64 g of the heavier isotope in the nature will be.
(c) 0.264 g (d) 0.0264 g
(a) 66.6% (b) 75%
11. Chlorine can be prepared by reacting HCl with MnO2. The
(c) 25% (d) 33.3%
reaction is represented by the equation
18. Which of the following molarity values of ions in a
MnO 2 (g) + 4HCl(aq) → MnCl2 (aq) + Cl2 (g) + 2H 2 O()
aqueous solution of 5.85% w/v NaCl, 5.55% w/v CaCl2
Assuming that the reaction goes to completion. What mass
and 6% w/v NaOH are correct [Na = 23, Cl = 35.5,
of conc. HCl solution (36% by mass) is needed to produce
Ca = 40, O = 16]
2.5g Cl2?
(a) [Cl–] = 2M (b) [OH–] = 1.5 M
(a) 14.27 g (b) 25 g
(c) 25.21 g (d) 30 g (c) [Ca2+] = 0.5 M (d) All of these

12. 29.2% (w/w) HCl stock solution has a density of 1.25 g mL– 19. How many moles of ferric alum, (NH4)2SO4 Fe2(SO4)3.24H2O
1. The molecular weight of HCl is 36.5 g mol–1. The volume can be made from the sample of Fe containing 0.0056 g of
(in mL) of stock solution required to prepare a 200 mL it?
solution of 0.4 M HCl is- (a) 10–4 mol (b) 0.5 × 10–4 mol
(a) 16 mL (b) 61 mL (c) 0.33 × 10–4 mol (d) 2 × 10–4 mol
(c) 80 mL (d) 8 mL 20. The following substances are present in different containers
13. The composition of residual mixture will be, if 30 g of Mg (i) one gram atom of nitrogen
combines with 30 g of O2 - (ii) one mole of calcium
(a) 40 g MgO + 20 g O2 (iii) One atom of silver
(b) 45 g MgO + 15 g O2 (iv) One mole of oxygen molecules
(c) 50 g MgO + 10 g O2 (v) 1023 atoms of carbon
(d) 60 g MgO only (vi) One gram of iron.
14. If the yield of given reaction is 33.33% what volume of The correct order of increasing masses (in grams) is/are
O2 gas will be produced if 4 moles of K2Cr2O7 are taken (a) (iii) < (iv) < (i) < (v) (b) (iii) < (vi) < (iv) < (ii)
initially; (c) (vi) < (v) < (i) < (iv) (d) (b) and (c) both are correct
21. How many molecules of HCl gas will be produced by
(a) 11.2 L (b) 22.4 L reacting 112 L of H2 (0 °C, 1 atm) with 213 g of Cl2?

(c) 33.6 L (d) 67.2 L (a) 3.61 × 1024 (b) 6.13 × 1023
(c) 6.13 × 1024 (d) 1.63 × 1024
15. 0.8 mole of a mixture of CO and CO2 requires exactly 40
gram of NaOH in solution for complete conversion of all the 22. 85 g CaCO3 (limestone sample), on heating produces exactly
CO2 into Na2CO3, if the mixture (0.8 mole) is completely the same amount of CO2 which converts 30 g of MgO to
oxidised to CO2, find further required moles of NaOH. MgCO3. The percentage purity of limestone sample is
(a) 0.2 (b) 0.6 (a) 80% (b) 82.4%
(c) 1 (d) 1.5 (c) 88.24% (d) 84.8%

Some Basic Concepts of Chemistry 23


NEET Past 10 Years Questions

1. An organic compound contains 78% (by wt.) carbon and 10. If Avogadro number NA, is changed from 6.022 × 1023 mol–1 to
remaining percentage of hydrogen. The right option for the 6.022 × 1020 mol–1, this would change: (2015 Re)
empirical formula of this compound is: [Atomic wt. of C is (a) The ratio of elements to each other in a compound
12, H is 1] (2021)
(b) The definition of mass in units of grams
(a) CH2 (b) CH3 (c) CH4 (d) CH (c) The mass of one mole of carbon
2. Which one of the followings has maximum number of (d) The ratio of chemical species to each other in a balanced
atoms? (2020) equation
(a) 1 g of Mg(s) [Atomic mass of Mg = 24]
11. What is the mass of the precipitate formed when 50 mL of
(b) 1 g of O2(g) [Atomic mass of O = 16] 16.9% solution of AgNO3 is mixed with 50 mL of 5.8% NaCl
(c) 1 g of Li(s) [Atomic mass of Li = 7] solution?
(d) 1 g of Ag(s) [Atomic mass of Ag = 108] (Ag = 107.8, N = 14, O = 16, Na = 23, Cl = 35.5) (2015 Re)

3. One mole of carbon atom weighs 12g, the number of atoms (a) 3.5 g (b) 7 g (c) 14 g (d) 28 g
in it is equal to.  (2020 Covid Re-NEET)
12. 20.0 g of a magnesium carbonate sample decomposes on heating
(Mass of carbon- 12 is 1.9926 × 10–23 g) to give carbon dioxide and 8.0 g magnesium oxide. What will be
(a) 6.022 × 1022 (b) 12 × 1022 the percentage purity of magnesium carbonate in the sample?
(c) 6.022 × 1023 (d) 12 × 1023 (Atomic weight of Mg = 24) (2015 Re)
4. The number of moles of hydrogen molecules required to (a) 96    (b) 60 (c) 84    (d) 75
produce 20 moles of ammonia through Haber’s process is :
13. When 22.4 litres of H2(g) is mixed with 11.2 litres of Cl2(g),
 (2019)
each at STP, the moles of HCl(g) formed is equal to: (2014)
(a) 10 (b) 20 (c) 30 (d) 40
(a) 2 mol of HCl(g) (b) 0.5 mol of HCl(g)
5. A mixture of 2.3 g formic acid and 4.5 g oxalic acid is treated (c) 1.5 mol of HCl(g) (d) 1 mol of HCl(g)
with conc. H2SO4. The evolved gaseous mixture is passed
through KOH pellets. Weight (in g) of the remaining product 14. 1.0 g of magnesium is burnt with 0.56 g O2 in a closed vessel.
at STP will be:  (2018) Which reactant is left in excess and how much? (2014)

(a) 1.4 (b) 3.0 (c) 4.4 (d) 2.8 (Atomic weight Mg = 24; O = 16)
(a) O2, 0.16 g (b) Mg, 0.44 g
6. In which case is number of molecules of water maximum? (2018)
(c) O2, 0.28 g (d) Mg, 0.16 g
(a) 18 mL of water (b) 0.18 g of water
–3
(c) 10 mol of water 15. Equal masses of H2, O2 and methane have been taken in
(d) 0.00224 L of water vapours at 1 atm and 273 K a container of volume V at temperature 27°C in identical
conditions. The ratio of the volumes of gases H2 : O2 : methane
7. A hydrocarbon contains 85.7% of Carbon and 14.3% of would be:  (2014)
Hydrogen. If 42 mg of the compound contains 3.01 × 1020
molecules, the molecular formula of the compound will be: (a) 8 : 16 : 1 (b) 16 : 8 : 1
 (2017-Gujarat) (c) 16 : 1 : 2 (d) 8 : 1 : 2
(a) C2H4 (b) C3H6 (c) C6H12 (d) C12H24 16. 6.02 × 1020 molecules of urea are present in 100 mL of its
8. Suppose the elements X and Y combine to form two solution. The concentration of solution is: (2013)
compounds XY2 and X3Y2. When 0.1 mole of XY2 weighs (a) 0.02 M (b) 0.01 M
10 g and 0.05 mole of X3Y2 weighs 9 g, the atomic weights (c) 0.001 M (d) 0.1 M
of X and Y are:  (2016–II)
17. An excess of AgNO3 is added to 100 mL of a
(a) 20, 30 (b) 30, 20 (c) 40, 30 (d) 60, 40
0.01 M solution of dichlorotetraaquachromium(III) chloride.
9. The number of water molecules is maximum in: (2015 Re) The number of moles of AgCl precipitated would be: (2013)
(a) 18 moles of water (b) 18 molecules of water (a) 0.001 (b) 0.002
(c) 1.8 gram of water (d) 18 gram of water (c) 0.003 (d) 0.01

24 Dropper NEET
ANSWER KEY

Topicwise Questions
1. (a) 2. (d) 3. (c) 4. (c) 5. (d) 6. (d) 7. (a) 8. (b) 9. (c) 10. (a)
11. (b) 12. (c) 13. (b) 14. (c) 15. (b) 16. (a) 17. (c) 18. (c) 19. (d) 20. (b)
21. (d) 22. (b) 23. (c) 24. (b) 25. (d) 26. (c) 27. (c) 28. (a) 29. (d) 30. (d)
31. (c) 32. (b) 33. (d) 34. (c) 35. (b) 36. (b) 37. (c) 38. (c) 39. (b) 40. (b)
41. (b) 42. (a) 43. (b) 44. (c) 45. (d) 46. (c) 47. (b) 48. (a) 49. (b) 50. (d)
51. (d) 52. (d) 53. (c) 54. (d) 55. (c) 56. (c) 57. (d) 58. (c) 59. (d) 60. (a)
61. (a) 62. (d) 63. (b) 64. (b) 65. (c) 66. (c) 67. (a) 68. (d) 69. (a) 70. (a)
71. (b) 72. (d) 73. (b) 74. (b) 75. (a) 76. (b) 77. (a) 78. (a) 79. (b) 80. (d)
81. (b) 82. (c) 83. (c) 84. (b) 85. (d) 86. (a) 87. (b) 88. (b) 89. (b) 90. (b)
91. (c) 92. (a) 93. (c) 94. (b) 95. (d) 96. (d) 97. (b) 98. (c) 99. (b) 100. (a)
101. (a) 102. (b) 103. (b) 104. (c) 105. (b)

Learning Plus

1. (b) 2. (d) 3. (d) 4. (b) 5. (c) 6. (a) 7. (b) 8. (a) 9. (a) 10. (a)
11. (b) 12. (d) 13. (c) 14. (d) 15. (b) 16. (a) 17. (a) 18. (c) 19. (d) 20. (a)
21. (b) 22. (a) 23. (b) 24. (b) 25. (a) 26. (d) 27. (d) 28. (b) 29. (d) 30. (c)
31. (c) 32. (c) 33. (c) 34. (a) 35. (c) 36. (c) 37. (d) 38. (a) 39. (b)

Multiconcept MCQs

1. (a) 2. (b) 3. (a) 4. (a) 5. (b) 6. (d) 7. (b) 8. (a) 9. (a) 10. (c)
11. (a) 12. (d) 13. (c) 14. (b) 15. (b) 16. (a) 17. (c) 18. (d) 19. (b) 20. (d)
21. (a) 22. (c)

NEET Past 10 Years Questions

1. (b) 2. (c) 3. (c) 4. (c) 5. (d) 6. (a) 7. (c) 8. (c) 9. (a) 10. (c)
11. (b) 12. (c) 13. (d) 14. (d) 15. (c) 16. (b) 17. (a)

Some Basic Concepts of Chemistry 25


1 Some Basic Concepts of
Chemistry

Topicwise Questions
1. (a) The number of significant figures in 0.0045 are two In CS2, C : S = 6 : 32 = 3 : 16
because zeros to the left of the first non-zero digit are not
In CO2, C : O = 12 : 32 = 3 : 8
significant.
From CS2 and CO2, S : O = 16 : 8 = 2 : 1
2. (d) d=s×t
1 femto = 10–15 sec = (3 × 108 m/s) (10–15 s) Also in SO2, S : O = 10 : 10 = 1 : 1
= 3 × 10–7 m 2 : 1 is multiple of 1 : 1
= 3 × 10–4 mm = 0.0003 mm So law of reciprocal proportion is satisfied.
3. (c) 1 Barn = 10–28 m2 13. (b) Hint: Total mass of reactants = Mass of products.
4. (c) 7.00 is more accurate than 7.0 because the former has
14. (c) Law of reciprocal proportion
three significant figures while the later has two.
5. (d) π is irrational number (please note that π ≠ 22/7 as 22/7 is 15. (b) Law of conservation of mass not applicable to nuclear
a rational number) and it means it has infinite number of reactions
significant figures. 16. (a) According to law of constant proportion
6. (d) According to rule (2) zero between ‘5’ and ‘4’ is also
significant 17. (c) 1g of X combines with Y = 35.5 g
7. (a) 29.4406 + 3.2 + 2.25 = 34.8906. As 3.2 has least number ∴ 2 g of X combines with Y = 2 × 35.5 g = 71 g
of decimal places, i.e., one, therefore sum should contain 18. (c) In First Oxide N2O: Mass of 2.24 L of nitrogen at STP =
one decimal place only. After round off, reported sum is 2.8g
34.9, which has three significant figures.
∴ Mass of Oxygen = 4.4 – 2.8 = 1.6 g
8. (b) Area = length × width In Second Oxide N2O2: Mass of 22.4 L of nitrogen at
= (12.34cm) × (1.23cm) = 15.1782 cm2
STP = 28 g
~ 15.18 cm2
∴ Mass of oxygen = 60 – 28 = 32 g
9. (c) 0.2876 g × 9 = 2.5884 g ∴ In second oxide 2.8 g of nitrogen combines with 3.2 g
= 2.588 g
of oxygen.
10. (a) According to rule (1) all non zero digits are significant Keeping the mass of nitrogen same in both the oxides,
11. (b) Haber’s process: the different masses of oxygen which combines with
N2 + 3H2 → 2NH3 2.8 g of nitrogen are 1.6 g : 3.2 g or 1 : 2, This is a
1L 3L   2L simple whole number ratio. This illustrates the law of
Total volume of reactant at STP = 4L multiple proportions.
Volume of NH3 = 2L
19. (d) SO2 , SO3
Volume of NH3 = 1/2 volume of Reactant
20. (b) H3PO4 + 3NaOH → Na3PO4 + 3H2O
Hence (b) one half.
As 1 mole H3PO4 neutralize 3 mole of NaOH. 120 g
12. (c) 6g of carbon combines with 32g sulphur to give CS2. of NaOH is 3 moles. So, 98 g of H3PO4 will neutralize
12g of carbon combine with 32g of O to give CO2. 120 g of NaOH.
10g of S combine with 10g of O to give SO2. 21. (d) 4Al + 3O2 → 2Al2O3
22. (b) BaCl2 + H2SO4 → BaSO4 + 2HCl 34. (c) The number of molecules present in 1 ml of a gas at STP
20.8 g 9.8 g x 7.3 g is known as loschmidt number.

mass of BaSO4 produced 35. (b) Verify options by calculating no. of moles

x = 20.8 + 9.8 – 7.3 = 23.3 g 36. (b) Apply,


absolute at.wt.
23. (c) In CO2, 12 parts by mass of C combine with 32 parts weight =
1amu
by mass of oxygen while in SO2, 32 parts by mass of
absolute at.wt.
S combine with 32 parts by mass of oxygen. The ratio 14 = …(1)
of masses of carbon and sulphur which combine with a 1 12
× g
fixed mass of oxygen is 12 : 32 or 3 : 8. In CS2, 12 parts 12 N A
of carbon combines with 64 parts by mass of sulphur absolute at.wt.
x= …(2)
therefore 12 : 64, i.e., 3 : 16. 1 12
× g
3 3 5 NA
∴ The ratios are : or 2 :1
8 16 x = 5.83 amu
24. (b) C2 H5 OH+Na → C2 H5 ONa+ 12 H 2 37. (c) 100 g Haemoglobin contains Fe = 0.34 g
1 mole haemoglobin contains Fe = 4 mole Fe2+
25. (d) H2O H2O2 = 4 × 56 = 224 g
H:O H:O 0.34 g Fe is present in Haemoglobin = 100 g
5.93 : 94.07 11.2 : 88.8 or 100
224 g Fe is present in Haemoglobin= × 224
= 65882 g 0.34
5.93 : 47.0
Ratio of different masses of O which combines with fixed 38. (c) 1 g – atom of nitrogen = 6.02 × 1023 N atoms
mass of H is 94.07 : 47.0 or 2 : 1 1 mol of N = 1/2 mole of N2 = 11.2 L at S.T.P
26. (c) In B, 32 parts of X combines with Y = 84 parts
∴ 16 parts of X combine with Y = 42 parts 39. (b) 10 g of CaCO3 = 0.1 mol of CaCO3
Now, number of parts of X in both B and C is equal. = 0.1 × 3 g – atom of Oxygen
Different masses of Y which combine with same mass of = 0.3 g atoms of oxygen
X in B and C are in the ratio 3:5 40. (b) Magnesium bicarbonate is Mg(HCO3)2. So, 146 g of
Mass of Y in B 3 Mg(HCO3)2 contains O atom = 0.6 NA
∴ =
Mass of Y in C 5 41. (b) Molecular mass of CO2 = 44g mol–1
∴ 4 mol of CO2 = 44 × 4 = 176 g
5
∴Mass of Y in C = × 42 (c) 6.02 × 1023 atoms of hydrogen has mass = 1.008 g
3
= 12 × 1024 atoms of hydrogen has mass = 20.1 g
= 70 parts (d) 22.4 L of helium at N.T.P. has mass = 4 g 11.2 L of
27. (c) 3.4 amu S → 100 amu insulin helium at N.T.P. has mass = 2 g
100 Thus; 4 moles of CO2 has maximum mass
32 amu S → × 32 =
941 amu
3.4 42. (a) In 12 g of carbon, the amount of C – 14
12 × 2 = 0.24
28. (a) Mass of one mole of a substance remains same =
100
29. (d) Average relative at Atm.wt
0.24 × 6.02 × 1023
20 × 10 + 80 × 11 ∴ C-14 atoms in 0.24 g =
. =
AW 14
100 22
= 1.03 × 10 atoms
200 + 880 1080 8 × 22.4
= = = 10.8 g 43. (b) Mass of 22.4 L gas at S.T.P. = = 32 g (it is also
100 100 5.6
equal to mol. mass)
W Mol. mass = 32
30. (d) No. of molecules = ×N 32 × 1
M .w ∴ V.D. = =16
2
31. (c) Gram molecular weight of any substance contains
Avagadro number of molecules. 44. (c) Mass of 1 atom = 1.8 × 10–22 g
Mass of 6.022 × 1023 atoms
Wt = 6.02 × 1023 × 1.8 ×10–22
32. (b) No. of atoms = × N × atomicity
MW = 108.36 g
33. (d) One amu = 1.66 × 10–24g ∴ Atomic mass of element = 108.36 g

6 Dropper NEET
45. (d) 1 M produces ions = 3 moles 55. (c) Mass of 1 mole of electrons
∴ 0.1 M H2 SO4 produce ions = 0.3 mol = 9.11 × 10–31 kg × 6.02 × 1023 = 5.5 × 10–7 kg mol–1
Number of ions = 0.3 × 6.02 × 1023
= 1.8 × 1023 56. (c) If 3.18 mol of hydrogen atom are there, then number of
moles of oxygen atoms
46. (c) 0.1 Mol CaCO3 = 0.1 × 100 = 10 g 3.18
= mol = 1.06 mol
1.51×10 23
3
Ca2+ = × 40 = 10 g
6.023 × 1023 57. (d) Molar mass of C10 H16 O = 120 + 16 +16
CO32– = 0.16 × 60 = 9.6 g = 152 g mol–1
25.0mg =25.0mg×  1g  ×  1mol  ×  27 × 6.02 × 10 atoms 
23
7.525 × 1022
Br = × 80 = 10 g 3
10 mg 152g 1mol
6.02 × 1023      
25 × 27 × 6.02
47. (b) Equal volume under similar conditions of temperature = ×1020 atom
152
and pressure have fixed number of molecules.
= 2.67 × 1021 atoms
1.71
48. (a) Moles of sugar added = = 5×10–3 4 NA
342 58. (c) × NA = atom
40 10
Carbon atoms added = 12 × 5 × 10–3 × 6.02 × 1023
= 3.61 × 1022 0.5 × 2 NA
0.5 M Na2SO=
4 × N A=
49. (b) 0.1 mole P4 contains = 4 × 0.1 × 6.02 × 1023 100 10
0.05 mole of S8 contains = 8× 0.05 × 6.02 ×1023 atoms 59. (d) SO2 → Contain 3 atom
82 wt. 64
50. (d) 82 g of N2 = g-molecule = 2.92 moles n
= = × 3N A
28 M.wt. 64
51. (d) Since; mass of oxygen is fixed. Therefore, the number of = 3 × 6.02 × 1023
g-atom and also the number of atoms will be fixed. 60. (a) Total e– + P + nº = 18
1× 2 18 × NA = 1.084 × 1025
i.e, Atoms of Oxygen = in O2
32
61. (a) 111 g anhydrous CaCl2 = 1 mole CaCl2
1 1 mole CaCl2 contain 1 mole Ca+2 and 2 mole Cl–
= in atomic Oxygen
16
62. (d) 1 g molecule of CO2 = 1 mole CO2
1× 3 1 mole CO2 occupy → 22.4 L
= in O3
48 63. (b) 224 g contain 36 g of water
52. (d) 12.8 g of SO2 = 4.48 L 488 g contain 2 × 36 g of water
6.02 × 1022 molecules of CH4= 2.24 L
18 g of water = 1 mole
0.5 mol of NO2 = 11.2 L
1 g molecule of CO2 = 22.4 L 72 g of water = 4 mole

53. (c) G.M.M of O2 = 32 g 64. (b) 4.4 g CO2 = 0.1 mole CO2 molecule
2.24 L H2 at STP = 0.1 mole H2 molecule
At.mass of O 16 1 NA
eq mass = = = 8= = = 0.2 mole molecules
valency 2 4 4
= 0.2 × 6.023 × 1023 = 1.20 × 1023
54. (d) No of mol of triatomic gas
65. (c) 22.4 L Vol. of gas at STP = 1 mole
224 ml
= = 10–2 mol 40 13.3 46.7
22400 ml mole −1 66. (c) C % : H %: N % = : :
12 1 14
No. of molecules = 10–2 mol × ( 6.02 × 1023 molecules
67. (a) Mass of chloride = (B2 – B1) g
mol–1)
(B2 – B1)g combines with metal = B1 g
no. of atoms of gas = 6.02 × 1021
= 3 × 6.02 × 1021 = 18.06 ×1021 35.5 × B1
∴ 35.5 g combines with metal = g
1.806 ×1022 atoms has mass = 1 g B2 − B1
68. (d)
1
1 atom has mass = Mass in % age H. Ratio S.R
1.806 ×1022 g
"g"
10−22 g C 24 40 40/12 = 3.33 1
=
1.806
H 4 6.66 6.66/1=6.66 2
= 0.554 × 10–22 g O 32 53.33 53.33/16=3.33 1
= 5.54 × 10–23 g Empirical formula : CH2O
Some Basic Concepts of Chemistry 7
69. (a) 74. (b) Molecular mass of B = M
Element % At. mass Simplest V.D. of B = M/2
%
ratio V.D. of A = 4 × V.D. of B
At.mass
M
A 25 12.5 25/12.5 = 2 1 4× =2M
2
B 75 37.5 75/37.5 = 2 1 Molecular mass of A = 2 × 2 M = 4M
The formula of the compound is AB
28
75. (a) Moles of N = =2
70. (a) Mol. mass of Fe (CHCOO)2 = 170 g 14
56 × 100 100 − 28 72
Fe in 100 gms of Fe (CHCOO)2 = Moles of metal, M = =
170 a a
= 32.9 mg 72
Total Fe in 400mg of capsule = 32.9 mg Mole ratio, M:N = :2 = 3:2
a
32.9 × 100 72
percentage of Fe = = 8.2% =3
400 a
71. (b) 72
Element % At. mass Mass Ratio S.R a= = 24
3
X 50 10 5 2
M (G ) 0.44 (Given)
Y 50 20 2.5 1 76. (b) =
M ( N2 ) 0.28
E.F. = X2Y
72. (d) E.F. = C3H4O 0.44
∴ M(G) = × 28 = 44 g
0.28
E.F. Mass = 12 × 3+ 4 × 1 + 16 = 56
170 ± 5 77. (a) 44 CO2 = 1 mol carbon
n= =3 3.38 g CO2 = 0.0768 moles carbon
56
∴ M.F. = (C3H4O)3 = C9H12O3 18 g H2O = 2 mol of hydrogen atoms
0.690 g H2O = 0.0767 mol hydrogen atoms
73. (b)
Molar ratio of C : H = 1:1
Oxide (I) Oxide (II)
∴ Empirical formula of hydrocarbon is CH
Metal; M 50% 40%
Oxygen; O 50% 60% Wt 12
78. (a) % of C = × 100 = × 100 =27.27
M.Wt(CO 2 ) 44
As first Oxide is MO2
let atomic mass of M = x 79. (b) 100 g haemoglobin has = 0.33 g Fe
67200 g haemoglobin has Fe
∴ % of O = 32 × 100
0.33
x + 32 = × 67200 = 221.76 g
100
50 32 1 mole of haemoglobin
or =
100 x + 32 221.76 g atom of Fe
=
x = 32 56
At. mass of metal M, x = 32 = 3.96 g atom of Fe ≈ 4.0 g atoms
254
let formula of second Oxide M2On 80. (d) Moles of Iodine = =2
127
2x 64
% of M = ×100
= × 100 80
2x + 16n 64 + 16n Moles of Oxygen= = 5
16
40 64 formula : I2O5
=
100 64 + 16n
81. (b) 0.5 mol of K4 [Fe (CN)6]
0.25 n = 2.5 – 1 = 1.5
1
1.5 has C = × 6 mol = 3 mol or 3 ×12 = 36 g
n= =6 2
0.25 82. (c) X2O
Now, formula of second oxide X=7
= M2O6 or MO3 X = 7 × 2 = 14 g
8 Dropper NEET
83. (c) 100 → 20 98. (c) MClx + x AgNO3 → M (NO3)x + xAgCl
Molar ratio: 1 x 1 x
? → 28
1 0.6
84. (b) CO + O2 → CO2 Molecules of AgNO3 given = × 500 =
3
2 100
Moles of MClx = 0.1
85. (d) The balanced equation is: ∴ The value of x = 3
4 NH3 + 5O2 → 4NO + 6H2O ...(1)
99. (b) CaCO3 + 2HCl → CaCl2 + CO2 + H2O
4 × 17 g of NH3 (eq-1)
100 g 73 g 44 g
Required O2 = 5 mol 100 ml of 20% HCl solution = 20 g HCl
5 × 6.8
6.8 of NH3 require O2 = = 0.5 mol Here, CaCO3 is a limiting reagent
4 × 17
Hence,
44
86. (a) CaCl2 + 2AgNO3 → Ca (NO3)2 + 2 AgCl 20 g of CaCO3 = × 20 g of CO2 = 8.8 g of CO2
100
111 g 2 × 143.5 g
CaCl2 require to produce 2 × 143.5 of AgCl = 111 g 100. (a) CaCO3 + 2HCl → CaCl2 + CO2 + H2O
CaCl2 required to produce 14.35 g of AgCl 25
25 ml of 0.75 molar HCl = 1000 × 1× 0.75 = 0.01875 moles
111× 14.35 moles of HCl 0.01875
= = 5.55 g Moles of CaCO3 required = =
2 × 143.5 2 2
87. (b) NaOH + HCl → NaCl + H2O = 9.375 ×10–3 moles
w 1000 Mass of CaCO3 required = 9.375 × 10–3 moles × 100 g/moles
88. (b) M = ×
GM .w V in ml = 0.94 g

N×E.wt×V(in ml) 101. (a) 22.4 L of SO2 at N.T.P = 1 moles


89. (b) w =
1000 11.2 L of SO2 at N.T.P = 1/2 moles
w 1 SO2 + 2H2S → 3S + 2H2O
90. (b) M = ×
Molar mass V in litres Here SO2 is a limiting reactant
∴ moles of sulphur produced = 3 × moles of SO2
91. (c) 2 NaOH + H2SO4 → Na2SO4 + H2O
Pure H2SO4 is required for 1 mole of = 1.5 moles
NaOH = 1/2 mole = 49 g 102. (b) SO2Cl2 + 2H2O → H2SO4 + 2HCl
70% H2SO4 required for 1 mole of NaOH H2SO4 + Ca(OH)2 → CaSO4
49 × 100 ∴ 1 mol of SO2Cl2 in aqueous solution required is 2
= = 70
70 moles Ca(OH)2
92. (a) 2KClO3 → 2 KCl + 3O2 1.15g
2 × 122.5 g 3 × 32 g 103. (b) Moles of Na = = 0.05 mol
23g mol−1
3 × 32 g of O2 is produced from KClO3 = 245 g
3.01× 1022
48g of O2 is produced from KClO3 = 245 g Moles of C = = 0.05 mol
6.02 × 1023
122.5 ×100
∴ 80% KClO3 needed = = 153.12 g Moles of O = 0.1
80
Moles ratio, Na : C: O = 1 : 1 :2
percentage×d×10
93. (c) N = ∴ Empirical formula – NaCO2
E.wt
ns 104. (c) Zn + 2HCl → ZnCl2 + H2
94. (b) Mass %(s) =
ns + no Here: HCl is a limiting reagent

N×V (in ml ) moles of HCl


95. (d) No.gram equivalents = ∴ Moles of H2 produced =
1000 2
0.52
w 1000 = = 0.26
96. (d) N = × 2
E.wt V (in ml)
105. (b) M = Percentage×specific gravity×10
w 1000 GMW
97. (b) m = ×
E.wt b (in gms) M1V1 = M2V2

Some Basic Concepts of Chemistry 9


Learning Plus
1. (b) Molality is not effected by temperature as molality Therefore, 60 g of carbon means 5 moles of carbon.
is not involved with volume (volume changes with Thus contains 5 × NA atoms, twice of this equals 10 NA
temperature). atoms of carbon.
Molality = [(Moles of solute / Weight of solvent (kg)] Total number of atom
= 10 × 6.022 × 1023 = 6.022 × 1024
2. (d) We have
6.022 × 1024 no. of atoms found in twice that in 60 g carbon.
Fe2(SO4)3 → Fe2O3 + 3SO3
7. (b) Let W% of Ne20 = x
So, 400 g of ferric sulphate is producing 240 g of SO3.
W % of Ne22 = 100 – x
So, 1 kg (1000 g) of ferric sulphate produce
x × 20 + (100 − x ) 22
SO3 = (240/400) × 1000 = 600 g 20.2 =
100
In 80 g of SO3, mass of oxygen = 48 g
w % of Ne20 = 90 %
Mass of sulphur = 32 g,
w% of Ne22 = 100 – 90 = 10%
So, mass of oxygen in 600 g of SO3 = (48/80) × 600 = 360g
Ne20 : Ne22 = 90 : 10 = 9 : 1
Mass of sulphur in 600 g of SO3 = (32/80) × 600 = 240 g
8. (a) Molecular mass of CuSO4, 5H2O = 249.5
(Mass of oxygen/Mass of sulphur) = (360/240) = (3/2)
= 63.5 + 32 + 4 × 16 + 5 × 18 = 249.5
3. (d) Molar mass = 14 + 3 = 17
Thus 249.5 g of CuSO4.5H2O contains Avogadro no. of
No. of moles = (4.25/17) = 0.25 moles
molecules
Total atoms = 4[1 + 3] NH3
So mass of 6.023 × 1023 molecules = 249.5
1 mole = 4 × 6.022 × 1027 atoms
249.5
0.25 mole = 0.25 × 4× 6.022 × 1023 = 6.022 × 1023 atoms. So mass of 1022 molecules = × 1022 = 4.144 g
6.023×10 23
4. (b) Average of readings of student,
9. (a) 111 g CaCl2 has = N ions of Ca2+
3.01 + 2.99
= A = 3.00 222
2 No. of molecules = ×N = 2 N ions of Ca2+
Average of readings of student, 111
3.05 + 2.95 111 g CaCl2 = 2 N ions of Cl–
=B = 3.00
2 2 N × 222
Correct reading = 3.00 222 g of CaCl2 = ions of Cl–
111
For both the students, average value is close to the correct = 4 N ions of Cl–
value. Hence, readings of both are accurate.
But the readings recorded by student A are more precise 10. (a) E.F. Wt = 14 gm
as they differ only by ± 0.01, whereas readings recorded MW = d × 22.4
by the student B are differed by +0.05. Thus, the results MW
of student A are both precise and accurate. M.F = E.F ×
EF Wt.
5. (c) Since, molarity (M) is calculated by following equation 11. (b) Given that,
weight × 1000 M1 = 5 M
Molarity =
molecular weight × volume (mL) V1 = 500 mL
5.85 × 1000 V2 =1500 mL
= = 0.2 molL−1
58.5 × 500 M2 = M
Note: Molarity of solution depends upon temperature For dilution, a general formula is
because volume of a solution depends upon temperature. M1V1 = M2V2
(Before dilution) (After dilution)
6. (a) No of atoms = (558.5/55.85) = 10 mole
500 × 5 M =1500 × M
Total number of atom
5
= 10 × 6.022 × 1023 = 6.022 × 1024 M= = 1.66 M
12 g of carbon means Avogadro’s number. 3

10 Dropper NEET
12. (d) For comparing number of atoms, first we calculate the W
moles as all are monoatomic and hence, moles × NA = 20. (a) No. of moles =
MW
number of atoms.
4 21. (b) 22400 cc of a gas at STP = 6.023 × 1023 molecules
Moles of 4 g He= = 1mol
4 So, 1.12 × 10–7cc of a gas at STP = ?
46 1.12 ×10−7 × 6.023 ×1023
46 g Na
= = 2 mol = = 3.01 × 1012 molecules
23 22400
0.4 22. (a) 32 g of O2 = 1 mol. = 6.023 × 1023 molecules.
0.40 g Ca
= = 0.01mol
40 ∴ 320 mg = 0.32 g = 0.01 mol = 6.3023 × 1021
12 From this 6.023 × 1020 moles are removed = 0.001 moles
12 g He
= = 3 mol
4 Remaining = 0.01 – 0.001 = 0.009 moles = 9 × 10–3 moles
Hence, 12 g He contains greatest number of atoms as it
23. (b) The at. no. of N and O are 1/2 of their Mass no. So, one
possesses maximum number of moles.
molecule of 92 g molecular weight will have 46 electrons
13. (c) E.F. Wt = 30 gm so 1 gm of molecule contain 46 N e–s.
MW = 2 × V.D W
MW 24. (b) No. of moles =
M.F = E.F × MW
EF Wt. 1 mole contain 2 mole of water
28 V of N 2 at S .T .P 25. (a) One mole of any substance contains 6.022 × 1023 atoms/
14. (d) W % of N 2 = × × 100 molecules.
22400 Wt. of compound
Hence, Number of millimoles of H2SO4
15. (b) The balanced chemical reaction is,
= molarity × volume in mL
K2SO4 + BaCl2 → BaSO4 + 2KCl
= 0.02 × 100 = 2 millimoles
1 mole of potassium sulphate reacts with 1 mole of
= 2 × 10–3 mol
barium chloride to precipitate 1 mole of barium sulphate
Number of molecules = number of moles × 6.022 × 1023
and 2 moles of potassium chloride is left in the solution.
Hence, in the given situation, sulphate ions are the = 2 × 10–3 × 6.022 × 1023
limiting agents. = 12.044 × 1020 molecules
So, 0.7 moles of potassium sulphate reacts with 0.7 moles 26. (d) 3, 3, 3
of barium chloride to precipitate 0.7 moles of barium
sulphate. Wt
27. (d) = no.of gram atoms
16. (a) The reaction is given below: At.wt
4Fe + 3O2 → 2Fe2O3 12 12 14
Mg : C : N = : :
Fe is the limiting reagent in this reaction. Limiting 24 12 14
reagent decides the amount of product formed. 28. (b) Let us assume the weight of the compound = 100 gm
Therefore, 4 moles of Fe produce 2 moles of Fe2O3. Weight of nitrogen = 36.8 gm
Thus, 0.5 moles of Fe will produce 0.25 moles of Fe2O3. Moles of nitrogen = (36.8/14)
∴ Weight of oxygen = 100 – 36.8 = 63.2 gm
N× Eq.wt ×V(in ml) 0.1× 63 × 250
17. (a) w = = = 1.575 g Moles of oxygen = (63.2/16)
1000 1000
The ratio of number of moles of N and O is,
18. (c) In the given question, 0.9 g L–1 means that 1000 mL
N : O = (36.8/14) : (63.2/16)
(or 1 L) solution contains 0.9 g of glucose.
= 2.628 : 3.95 = 1 : 1.5 = 2 : 3
conc in gL−1 0.90 So, formula is N2O3.
=M = = 0.005 M
Molar mass 180
 y y
Moles of solute 29. (d) C x H y +  x +  O2 → xCO2 + H 2 O
19. (d) Molality (m) = …(i)  4  2
Mass of solvent (in kg)
160 26
Given that, Mass of solvent (H2O) = 500 g = 0.5 kg x= = 4 and x + y=
40 4
Weight of HCl = 18.25 g
Molecular weight of HCl = 36.5 g Y 10
=
18.25 0.5 4 4
Moles of HCl = = 0.5 = = 1m ∴ Y = 10
36.5 0.5

Some Basic Concepts of Chemistry 11


30. (c) Equal volumes of different gases at STP contains an = 1.5 mL × 3.12 gm L–1 = 4.68 g
equal number of moles. Hence, the answer is reported as 4.7 g.
4 g oxygen corresponds to (4/32) = 0.125 mole. This is
2 significant figures.
also equal to the number of moles of hydrogen. 1 mole of
hydrogen corresponds to 2 gm. w 1000
35. (c) M = ×
Hence, 0.125 mole will correspond to 2 × 0.125 = GMW V in ml
0.25 gm.
Hence, the correct option is C. N = M × Basicity of an acid
No.of milli equivalents N×GEW×V (in ml)
31. (c) N = 36. (c) w =
V (in ml ) 1000
37. (d) W = N × G.EW × V (In L)
32. (c) M = percentage×10×d
GMW 40
=
% Purity of NaOH × 100
33. (c) Empirical formula mass = CH2O 50
= 12 + 2 × 1+ 16 = 30 = 80 %
Molecular mass = 180 38. (a) According to the law of conservation of mass,
Molecular mass Total mass of reactants = Total mass of products
n=
Empirical formula mass Amount of Fe2O3 is decided by limiting reagent.

180 39. (b) The element, carbon, combines with oxygen to form two
= = 6 compounds, i.e., carbon dioxide and carbon monoxide in
30
CO2, 12 parts by mass of carbon combine with 32 parts by
∴ Molecular formula = n × empirical formula
mass of oxygen while in CO, 12 parts by mass of carbon
= 6 × CH2O combine with 16 parts by mass of oxygen.
= C6H12O6 Therefore, the masses of oxygen combine with a fixed
34. (a) Given that density of solution = 3.12 g mL–1 mass of carbon (12 parts) in CO2 and CO are 32 and 16
Volume of solution = 1.5 mL respectively. These masses of oxygen have ratio of 32 :
16 or 2 : 1 to each other.
For a solution,
Mass = volume × density This is an example of law of multiple proportion.

Multiconcept MCQs
1. (a) N 2(g) + 3H 2(g) 
 2NH 3(g) 3
To obtain 0.200 mole B2H6, NaBH4 required = × 0.200
2
1 L of N2 reacts with 3 L of H2 to form 2 L of NH3.
= 0.300 mole
So, N2 is the limiting reagent here because when 30L of
Because, the yield is 70%, Hence
H2 will be consumed, the volume of N2 consumed will be 70
rd x× = 0.300
1 1 100
i.e., 10L .
× 30 =
3 3 0.300 100 3
x= × == 0.429 moles
1000 70 7
Since actual yield is 50% of the expected value, NH3
formed = 10 L , N2 reacted = 5 L and H2 reacted 15 L. 3. (a) Ag2S → 2Ag + S
To obtain 2 mole Ag, mole of Ag2S required = 1
So, the final mixture contains 10 L NH3, 25 L N2 and 15
1
L H2. To obtain mole Ag, mole of Ag2S required
108
2. (b) 3NaBH 4 + 4BF3 
→ 3NaBF4 + 2B2 H 6 1 1 1
=× = mole
2 108 216
Since BF3 is in excess, the limiting reagent is NaBH4.
1
To obtain 2 mole B2H6, NaBH4 required = 3 mole =
grams of Ag2S required × 248 g
216
12 Dropper NEET
1.24 g Ag2S is obtained from ore = 100 g 7.04
Moles of H 2 O formed when moles of CO 2 react
248 100 248 44
=
g Ag2S is obtained from ore ×
216 1.24 216 9 7.04 63.36
= ×= = 0.18 moles
24800 8 44 8 × 44
= = 92.6 g
267.84 wt. of H2O formed = 0.18 × 18 = 3.24 g
4. (a) C : H : N = 9 : 1 : 3.5
9 1 3.5 10. (c) 2KHCO3  D
→ K 2 CO3 + H 2 O + CO 2 ↑
∴ mole ratio =C:H:N = : : =3:4:1 Loss of wt. will be because of CO2 escaped
12 1 14
(C3H4N)x ⇒ x = 2 & C6H8N2 Total KHCO3 chosen = 1.50 g
percentage purity = 80%
6.02 × 1020 80
5. (b)
= [urea] = 10−3 mol ∴ Pure KHCO3 = 1.50 × = 1.2 g
6.02 × 1023 100
M. mass of KHCO3 = 39 + 1 + 12 + 3 × 16 = 100 g
10−3
∴ [urea] = = 0.01 M 1.2
0.1 =
moles of pure KHCO = 0.012 moles
3 100
x
6. (d) Mn = x gm = = 0.018x From balanced equation-
55
2 moles of KHCO3 yield moles of CO2 = 1
x
O = x gm = = 0.0625 x 1
16 0.012 moles of KHCO3 yield moles of CO2= × 0.012
2
0.018 x 0.0625 x = 0.006 moles
=1 ⇒ ≈ 3.5
0.018 x 0.018 x wt. of CO2 formed = 0.264 g
Mn1O3.5 ⇒ Mn2O7
11. (a) MnO2 + 4HCl → MnCl2 + Cl2 + 2H2O
7. (b) M + H2SO4 → MSO4 + H2 From balanced equation,
71 gm Cl2 is produced from HCl = 4 × 36.5 g
Mole of M Mole of H 2SO 4
= 4 × 36.4 × 2.5g
1 1 2.5 gm Cl2 is produced from HCl =
71
= 5.14 g
1 4.08
= Now, HCl is 36% by mass, which means-
a 98
36 g HCl is obtained from HCl solution = 100 g
98
=a = 24.01 100 × 5.14
4.08 5.14 g HCl is obtained from HCl solution =
36
Thus atomic weight of M = 24 g.
= 14.27 g
8. (a) Let molar mass of compound = y g
12. (d) Density of solution = 1.25 g/mL
% by wt of C = 36
29.2% (w/w) means that 29.2 g of HCl is present in
So, 24 × 100 =
36 100 gms of solution
y
wt. of soln.
So, ρ ( density of solution ) =
2400 vol. of soln.
=y
36 100
1.25 g/mL =
y = 66.6 g vol. of soln.
So, 66.6 g has moles = 1
100
1 v= mL
10 g has moles= × 10= 0.15 1.25
66.6
no. of moles of solute
molarity of solution =
25 vol. of solution (in ltrs)
9. (a) C8 H18 + O 2 → 8CO 2 + 9H 2 O
2
29.2 / 36.5
M
= × 1000
= 10 M
7.04 100 / 1.25
Moles of CO 2 formed = moles = 0.16
44 Apply, M1V1 = M2V2
now, from equation 0.4 × 200 = 10 × V
Moles of H2O formed when 8 moles of CO2 react = 9
V = 8 mL
Some Basic Concepts of Chemistry 13
13. (c) 2 Mg + O2 → 2 MgO 18. (d) Only single solution have all these
2 × 24 g Mg reacts with O2 = 32 g Means 100 ml solution have 5.85 gm NaCl = 0.1 mole
32 and 5.55 gm CaCl2 = 0.05 mole
30 g Mg reacts with O= × 30
= 20 g
2 2 × 24 ( 0.1 + 0.05 × 2 ) ×1000
=[Cl− ] = 2M
So, 10 g O2 will be left unreacted 100
Mg is the limiting reagent ( 0.1 + 0.15 ) × 1000
⇒ [Na + ]
= = 2.5 M
100
2 × 24 g Mg forms MgO = 2 × 40 g
0.5
2 × 40 [Ca 2 + ] = × 1000 =0.5 M ⇒ [OH − ] =1.5 M
=
30 g Mg forms MgO × 30
= 50 g 100
2 × 24

14. (b) 0.0056


19. (b) Moles=
of Fe = 10−4
56
mole 4 3 (100% yield)
1 mol of alum = 2 mol of Fe
= 1 (for 33.33%)
2 mol of Fe = 1 mol of alum
Volume of O2 gas produced = 1 × 22.4 = 22.4 L
1
10–4 mol of Fe= × 10−4 mol of alum
15. (b) CO2 + 2NaOH → Na2CO3 + H2O 2
nNaOH = 1 = 0.5 × 10–4 mol of alum
∴ CO2 present in mixture = 0.5 and CO present = 0.3 20. (d) (i) 14 g (ii) 40 g
mole 108
(iii) = 23
1.79 × 10−22 g
6.022 × 10
When more CO2 produced = 0.3, more NaOH required
(iv) 32 g (v) 1.99 g (vi) 1 g
= 0.3 × 2 = 0.6 mole
Hence, the correct order of increasing masses is
D
16. (a) CaCO3 + MgCO3  → CaO + MgO + 2CO 2 (iii) < (vi) < (v) < (i) < (iv) < (ii)
Let the mass of CaCO3 = x g
21. (a) H2(g) + Cl2(g) → 2HCl(g)
Then, mass of MgCO2 = (3.68 – x)g
x 112 213
moles of CaCO3 = =112L
= moles 213g
100 22.4 71
3.68 − x = 5=
moles 3moles
moles of MgCO3 =
84 Here, Cl2(g) is limiting reagent,
Applying POAC for C-atoms 1 Mole Cl2(g) produces HCl(g)
x 3.68 − x = 2 × 6.022 × 1023 molecules
+ 0.04
=
100 84 3 Moles Cl2(g) produces HCl(g)
x=2 = 3 × 2 × 6.022 × 1023 molecules
2 1.68 = 3.61 × 1024 molecules
∴ n CaCO3 = = 0.02 and n MgCO3 = = 0.02
100 84 22. (c) MgO + CO2→ MgCO3
0.02 40 g 44 g
mole% of CaCO3 = × 100 = 50%
0.04 40g MgO needs CO2 = 44 g
0.02 44 × 30
mole% of MgCO3 = × 100 = 50% 30g MgO needs CO2 = = 33g
0.04 40
CaCO3 → CaO + CO2
17. (c) Let relative abundance of heavier isotope be x %
100 g 44 g
(100 − x )( A − 1) + x ( A + 3) 44 g CO2 is obtained from CaCO3 = 100 g
A= 100 × 33
100 33 g CO2 is obtained from CaCO3 = = 75g
44
100 A = 100A – xA – 100 + x + 3x + xA Percentage purity of CaCO3 sample
4x = 100 75
= × 100 =88.24%
∴ x = 25. 85

14 Dropper NEET
NEET Past 10 Years Questions
1. (b) Gaseous mixture formed is CO and CO2. When it is
passed through KOH, only CO2 is absorbed. So the
At %
__________ Simplest remaining gas is CO. KOH pellets absorbs all CO2, H2O
Element %
Weight At weight Ratio is absorbed by H2SO4 thus CO is remaining product.
C 78 12 6.5 1 So, weight of remaining gaseous product CO is
2
H 22 1 22 3 ´ 28 = 2.8g
20
Empirical formula of this compound is CH3 So, the correct option is (d)
2. (c) 6. (a)
1
(a) Number of Mg atoms
= × NA (a) Mass of water = 18 × 1 = 18 g
24
1 18
= × 6.022 × 1023 atom Molecules of water = mole × NA = NA
24 18
= 1 NA
1
(b) Number of O atoms
= × NA 0.18
32 (b) Molecules of water = mole × NA = NA
1 18
= × 2 × 6.022 × 1023 atom
32 = 10–2 NA
1 (c) Molecules of water = mole × NA = 10–3 NA
(c) Number of Li atoms= × NA
0.00224
7 (d) Moles of water = = 10-4
1 22.4
= × 6.022 × 1023 atom
7 Molecules of water = mole × NA = 10–4 NA
1 7. (c)
(d) Number of Ag atoms
= × NA
108 % Moles Relative moles
1 85.7
= × 6.022 × 1023 atom C 85.7 = 7.14 1
108 12
Hence, 1g lithium has the largest number of atoms.
14.3
3. (c) H 14.3 = 14.3 2
1
No of atom in 12 g carbon = 12 ÷ (1.9926 × 10–23) Hence, empirical formula = CH2.
= 6.022 × 1023 atoms empirical weight = 14
Thus Number of atoms in 1 mole carbon 3.01 × 1020 42 × 10−3
= 6.022 × 1023 atoms = No.
= of moles
6.022 × 1023 M
4. (c) 1 42 × 10 −3

Haber’s process × 10−3 =


2 M
 2NH3 (g)
N 2 (g) + 3H 2 (g)  M = 84
84
20 moles need to be produced \ Atomicity
= = 6
14
2 moles of NH3 → 3 moles of H2 Molecular formula = C6H12.
3 × 20 8. (c)
Hence 20 moles of NH3 → = 30 moles to H2
2 For XY2, let atomic weight of X = Ax
5. (d)
and of Y = Ay
HCOOH Conc
 H 2SO4
→ CO ( g ) + H 2O ( l )
10
 1  1 So, n XY2= 0.1
=
2.3g or  mol  mol A x + 2A y
 20  20 Ax + 2Ay = 100 ...(1)
Similarly for X3Y2,
3Ax + 2Ay = 180 ...(2)
On solving (1) and (2)
Ax = 40 and Ay = 30

Some Basic Concepts of Chemistry 15


9. (a) 14. (d)
(a) 18 moles of water will contain 1
= 18 × 6.022 × 1023 molecules of H2O n Mg
= = 0.0416 moles
24
(b) 18 molecules 0.56
n O 2 = 0.0175 moles
=
(c) 1.8 = 0.1 mole will contain 32
18
The balanced chemical equation:
= 0.1 × 6.022 × 1023 molecules of H2O
18 1
(d) g= 1 × 6.022 × 1023 molecules of H2O
1mole = Mg + O 2 → MgO
18 2
So, maximum number of molecules is present in 18 Initial 0.0416 moles 0.0175 moles 0
moles of H2O.
Final (0.0416 – 2 × 0.0175) 0 2 × 0.0175
10. (c) Avogadro’s number 6.022 × 1023 is ideally the mass of
number of atoms present in 1 mole that is 12 grams of = 0.0066 moles (O2 is limiting reagent)
C. If we change the Avogadro’s number it will directly ∴ Mass of Mg left in excess = 0.0066 × 24
change the mass of 1 mole that is 12 g of C.
= 0.16 g
11. (b) Molecular weight of AgNO3 = 170
Molecular weight of NaCl = 58.5 15. (c)
1. 16.9% solution of AgNO3 means 16.9 g of AgNO3 in 100 According to Avogadro’s principle, ratio of volume of
mL of solution gases will be equal to the ratio of their number of moles
so, 8.45 g of AgNO3 in 50 mL of solution.
w
2. 5.8% solution of NaCl means 5.8 g of NaCl is in 100 mL mole =
Mw
solution. So, in 50 mL = 2.9 g NaCl
AgNO3 + NaCl → AgCl + NaNO3 n H 2 : n O 2 : n CH
4

8.45 2.9 w w w
Initial Mole: 00 0 0 : : ⇒ 16 : 1 : 2
170 58.5 2 32 16
= 0.049 = 0.049 0 0 16. (b)
Final mole: 0 0 0.049 0.049 6.02 × 1023 number of molecules = 1 mole
Mass of AgCl precipitated = 0.049 mole = 0.049 × 143.3 6.02 × 1020 = 0.001 mole
= 7.02 gm  7 gm 0.001
mole
12. (c) Concentration
= =
× 1000 × 1000
V ( mL ) 100
MgCO3 → MgO + CO2 (g)
↓ ↓ ⇒ 0.01 M
Mw → 84 g 40 g 17. (a) Molarity of solution of dichlorotetraaquachromium(III)
According to question chloride = 0.01 M.
84 g MgCO3 gives = 40 g MgO
Volume of solution of dichlorotetraaquachromium(III)
40
1 g MgCO3 gives = chloride = 100 ml.
84
The formula of dichlorotetraaquachromium(III) chloride
40
20 g MgCO3 gives = × 20 = 9.52 g of MgO is [Cr(H2O)Cl2]Cl.
84
But according to question On ionisation,
yield of MgO is = 8 g
[Cr(H2O)Cl2]Cl → [Cr(H2O)Cl2]+ + Cl–
8
% purity = × 100 =
84%
9.52 Initial   100 × 0.01 0 0
13. (d) 1 mole = 22.4 litres at S.T.P. Final    0       1 mol   1 mol
22.4 11.2
n H2
= = 1 mol ; n=
Cl 2 = 0.5 mol So 1 mol of Cl– ions will react with 1 mol of AgNO3 mole
22.4 22.4
of [Cr(H2O)Cl2]Cl 0.1 M 100 ml solution is,
Reaction is as,
H2 (g) + Cl2 (g) → 2HCl (g) No. of moles = Molarity × Volume
Initial 1 mol 0.5 mol 0       = 0.01 × 0.1 = 0.001 mol
Final (1 – 0.5) (0.5 – 0.5) 2 × 0.5 Hence, 0.001 mol of Cl– ions will react with 0.001 mole
= 0.5 mol = 0 mol 1 mol
Here, Cl2 is limiting reagent. So, 1 mole of HCl (g) is of AgNO3.
formed. So number of moles of AgCl formed is 0.001 mol.

16 Dropper NEET
2 Structure of Atom

Past Years NEET Trend

4
No. of MCQs

0
2021 2020 2019 2018 2017 2016 2015 2014 2013 2012

Investigation Report
TARGET EXAM PREDICTED NO. OF MCQs CRITICAL CONCEPTS

• Bohr’s model for hydrogen atom, Calculation of E


NEET 1-2 and r, Quantum numbers

Perfect Practice Plan


Topicwise Questions Learning Plus Multiconcept MCQs NEET Past 10 Years Total MCQs
Questions
118 35 24 22 199
INTRODUCTION Proton: The smallest and highest positive ion obtained from
hydrogen.
The atomic theory of matter was first proposed by John Dalton,
Neutron: Neutron are the particles in an atom that have neutral
known as Daltons atomic theory.
charge
Dalton regarded that atom is the ultimate particle of matter. 9 4 12 1
4 Be + 2 He → 6 C + 0 n
According to Modern atomic theory atoms can be further divided Beryllium α− particle Carbon Neutron
into subatomic particles i.e. electrons, protons and neutrons.
SUB-ATOMIC PARTICLES ATOMIC MODELS
Dalton’s Theory is able to explain law of conservation of mass, Thomson Model
law of constant composition and law of multiple proportions. J.J. Thomson, in 1898, proposed that an atom possesses a spherical
Same salient feature’s of this theory are: shape radius in which the positive charge is uniformly distributed.
(i) Atoms of a specific element are like but it differ from atoms According to Thomson atom is like water melon and electron’s
of other element are embedded like seeds in water melon.
(ii) Atom of each element is an ultimate particle and it has a Positive Sphere
characteristic mass but is structureless
(iii) Atoms are indestructible, as they can neither be created nor
be destroyed.

Fundamental Particles
Electron
Fundamental Particle
Fig: Thomson model of atom
Electron Proton Neutron
An important feature of this model is that the mass of the atom is
Charge –1.6022 × 10–19 C +1.6022 × 10–19 C Neutral assumed to be uniformly distributed over the atom.
This model can not explain electrical neutrality of the atom.
9.10939 × 10–31 kg 1.67262 × 10–27 kg 1.67493 × 10–27 kg
Mass This model also called plum pudding, raisin pudding or
0.00054 a.m.u 1.00727 a.m.u 1.00867 a.m.u
watermelon model of atom.
Charge to Mass Ratio of Electron Rutherford’s Model of Atom
The charge to mass ratio is known as specific charge. Larger the Rutherford suggested atomic model based on a-ray scattering
magnitude of the charge on the particle larger is the deflection experiment.
when electric and magnetic field is applied.
Scattering of a narrow beam of a-particles as they passed from a
Lighter the mass of the particle larger will be the deflection.
thin gold foil and it is covered with fluorescent ZnS screen. When
Thomson was able to find the value of charge to mass ratio as
a-particles strike the screen then flash of light was produced at
1.758820 × 1011 ckg–1
that point.
As the velocity of the electron increases, the specific charge of
Gold foil
the electron decreases due to the increase of relative mass of the
electron. *
* *
The mass of a moving electron may be calculated by applying *
the formula
m0
m= * *
2 *
 v * * *
1−  
 c
Source of Lead plate Photographic plate
alpha particles
Where m0 = Rest mass of electron
Fig.: Rutherford’s scattering experiment
v = Velocity of the electron
c = Velocity of light
If v = c, mass of the moving electron becomes infinity.
Charge on the Electron
Mullikan determined the charge of the electron by an oil drop
experiment. Millikan concluded that the magnitude of electrical
charge, q, on the droplets is always an integral multiple of the
electrical charge, e, that is q = ne, where n = 1,2,3......
Charge on the oil drops was always an integral multiple of 1.60
× 10–19 C.
e 1.60 × 10−19 C
me
= = = 9.1094 × 10−31 kg
e / me 1.758820 × 1011 Ckg −1 Fig.: Schematic molecular view of the gold foil
Structure of Atom 27
Observation ● Cosmic rays, g-rays, X-rays, UV light, visible light, Infrared
1. Most of the a-particles passes through the foil undeflected. light, micro waves and radio waves are known electromagnetic
2. A small fraction of a-particles were deflected by small radiation as they are made up of electric and magnetic fields
angles. that are radiates in perpendicular directions in one another.
3. A very few a-particles bounced back were deflected by 180o ● Electromagnetic radiations have wave characteristics and no
medium is required for their propagation. They can travel
Conclusion through the vacuum.
1. Most of the space in the atom is empty.
2. A few positively charged were deflected. The deflection Crest  Crest

Vibrating source
must be due to enormous repulsive forces shows that the a
positive charge of the atom is not spread out the atom.
a Direction
3. Volume occupied by the nucleus is very small as compare to of propogation
total volume of atom Trough  Trough
Assumption of this Model
The Electromagnetic Spectrum
(i) All the positive charge and mass of the atom is present in
a very small region at the centre of the atom. It is called In Å 10–4
nucleus.
0.01 0.1 150 3800 7600 6 ×106 3×109 3×1014
(ii) The nucleus is surrounded by electrons that move around the
nucleus with a very high speed in circular paths called orbits.
Cosmic g-rays X-rays Ultra Visible Infra red Micro Radio
(iii) The centrifugal force is present due to fast moving electrons rays violet wave waves
balances the coulombic force of attraction present between
the nucleus and the electrons.
(iv) Rutherford’s atomic model is comparable with the solar
system. So it is known as planetary model.
Drawback of Rutherford’s Model
1. It fails to explain the atomic spectrum or line spectrum.
2. It fails to explain stability of atom. V I B G Y O R
Violet Indigo Blue Green Yellow Orange Red
Atomic Number and Mass number
3800 4300 4500 4900 5500 5900 6500
● The number of electrons or protons present in an atom of an 7600 In Å
element is called its atomic number(Z).
● The sum of protons and neutrons in the atom of an element is A wave is characterized by following characteristics:
called its mass number(A). Wave Length (l): The distance between two neighbouring
● Number of neutrons = A – Z. troughs or crests in wave is known as wave length.
Isotopes and Isobars The units of wave length are m, cm, Å , nm, mm or pm.
Atoms of the same element having same atomic number but 1A0 = 10–8 cm = 10–10 m
different mass numbers are known as isotopes. Frequency (n): The number of waves which pass through a given
Example: (1) Isotopes of hydrogen: point in one second is called frequency.
Protium (1H1), Deuterium (1H2) or (1D2) or Tritium 1T3 The units of frequency are sec–1, cycles per second (cps) or Hertz
(2) Isotopes of chlorine : (17Cl35) and (17Cl37) (Hz).
1cps = 1 Hz = sec–1
Isobars
The relation between frequency(n) and wavelength (l) is
Atoms of different elements having same mass number but
different atomic numbers are known as isobars. c
ν=
λ
Eg: 14 14
6 C, 7 N
Where c = Velocity of light
WAVE NATURE OF ELECTROMAGNETIC Wave Number ( ν ) : The number of wave lengths per centimetre
RADIATION or the reciprocal wave length is called wave number.
● Electromagnetic radiation is the energy transmitted from one The unit of wave number is cm–1 or m–1
body to another in the form of waves and these waves travel 1
Wave number ν =
in the space with the same speed as light ( 3 × 108 m/s) and λ
these waves are called as Electromagnetic waves or radiant
energy. The relation between ν and ν is ν = cν

28 Dropper NEET
Amplitude (A): The height of the crest or depth of the trough of TRAIN YOUR BRAIN
a wave is called amplitude.
Q. Calculate number of photon coming out per sec. from the
Amplitude is a measure of the intensity or brightness of a beam bulb of 100 watt. If it is 50% efficient and wavelength
of light. coming out is 600 nm.
Velocity (C): The distance travelled by a wave in one second is Ans. Energy = 100J
called its velocity. Energy of one photon
The units of velocity are m/sec or cm/sec. hc 6.625 × 10−34 × 3 × 108 6.625
= = −9
= × 10−19
λ 600 × 10 2
Particle Nature of Electromagnetic Radiation
Some of the experimental phenomenon such as diffraction 100
No. of photon = × 1019 = 15.09 × 1019
and interference can be explained by the wave nature of the 6.625
electromagnetic radiation.
Photo Electric Effect
A body that is capable of absorbing all types of radiations and In 1887, H. Hertz performed a experiment that is photo electric effect.
emits completely the absorbed radiations is called a black body.
When light is exposed to clean metallic surface, electrons are ejected
When a black body is heated it emits thermal radiation of different from the surface. This phenomenon is called photo electric effect.
wave lengths or frequencies.
At a given temperature the intensity of radiation increases with The Observation of Photoelectric Effect
wavelength reaches a maximum and then decreases. (i) The electrons are ejected from the metal surface as soon as
When solids are heated they emit radiation over a wide range of the beam of light strikes the surface, i.e., there is no time
lag between the striking of light beam and the ejection of
wavelength.
electrons from surface.
T2 > T1 (ii) Ejection of electrons from the surface of a metal by
T2
Intensity

irradiating it with light of suitable frequency.


(iii) A part of the energy of photon which is used to escape the
T1 electron from the attractive forces and the remaining energy
is used to increase the kinetic energy of electron.
E = W + KE
Wavelength
hn = hn0 + KE
Fig.: Wavelength intensity relationship Where W = Work function or Threshold energy
W = hv0
Planck’s Quantum Theory
Where v0 = Threshold frequency
● Substances that absorb or emit light discontinuously in the
form of small packets or bundles of energy known as quanta. (iv) In photo electric effect the number of photo electrons that is
emitted is proportional to intensity of incident light.
● The radiation is propagated in the form of waves.
The minimum energy that is required for emission of photo
● The energy of a quanta is directly proportional to the electrons is called threshold energy or work function.
frequency of the radiation. For each metal, there is a characteristic minimum frequency (also
E∝n known as threshold frequency) below which photoelectric effect is
E = hv not observed. At a frequency ν > ν 0 then photoelectric effect is
observed.
hc
∴E = = hcν
λ Spectra
The spectra that is obtained by the emission of energy by the
Where E = Energy of radiation
excited atoms are called emission spectra.
h = Planck’s constant
When white light is passed through a gas and the emergent beam
= 6.625 × 10–27 erg – sec of light that is allowed to fall on a photographic plate, the spectrum
= 6.625 × 10–34 Joule – sec obtained is called absorption spectrum.
● A body that can absorb or emit energy in whole number
Hydrogen Spectrum
multiple of quantum.
The source of radiation here is a hydrogen discharge tube.
E = nhn
The discharge tube that contains hydrogen gas at low pressure and
Where n = 1, 2, 3, 4, ........
high potential difference
● The frequency of emitted radiation goes from lower to higher
The bright light emitted from the discharge tube is passed through
as the temperature increases. a prism to cause dispersion.
Structure of Atom 29
The emergent beam of light falls on a photographic plate and is When an electron jumps from a lower state to a higher state,
recorded as the atomic spectrum of hydrogen. energy is absorbed.
The wavelength or wave number of various lines in the visible The absorbed or evolved energy is equal to the difference in
region can be expressed by an equation. [Rydberg’s equation] energies of two orbits, which is equal to quanta.
1 1 1  DE = E2 – E1 = hn
ν= = RH  2 − 2 
λ  n1 n 2  where E2= Energy of higher state
E1= Energy of lower state
where RH = Rydberg’s constant for H-atom
The frequency of radiation absorbed (or) emitted when transitions
= 1,09,677 occurs between two stationary states that differ in energy by DE,
n1 = Lower energy level is given by
n2 = Higher energy level ∆E E 2 − E1

=v =
Rydberg constant value is not same for all the elements. h h
For hydrogen like species He+, Li2+, Be3+, This expression is known Bohr’s frequency
R = 1,09,677 x Z2 cm-1 The line spectrum is obtained due to the electronic transition from
= RH × Z2 one state to another state
Maximum no of spectral lines produced from n2 to n1 state of a
n ( n + 1)  KEY NOTE
simple atom = where n = n2 – n1 Š The force of attraction between the nucleus and then electron
2
They can be classified into various series. − Ze2
= 2
r
Series of Hydrogen of Spectrum 0.529 × n 2 0
Š Radius of orbits in H atom like ions r = A
Name of the series n1 n2 Region Z
1 Ze2
Lyman series 1 2, 3, 4, 5.. Ultraviolet Š Kinetic energy of electron= mν 2=
2 2r
2
Balmer series 2 3, 4, 5, 6.. Visible − Ze
Š Potential energy of electron =
r
Paschen series 3 4, 5, 6. 7 Near I.R
Š As we go to higher orbits, kinetic energy decreases, potential
Brackett series 4 5, 6, 7 8.. I.R energy increases and the total energy increases.
Pfund series 5 6, 7, 8. 9 Far I.R Š Total energy of electron
Ze2 Ze2 Ze2
BOHR’S MODEL E= KE + PE = − = −
2r r 2r
Postulates of Bohr’s Atomic Model Š Expression for the energy of Bohr’s orbit
The electrons in an atom that revolve round the nucleus in fixed −2π2 mZ2e4
circular orbits or shells or energy levels. E=
n 2h 2
Electrons that can revolve only in those stationary orbits in which Š Velocity of electron in bohr orbit
h
their angular momentum is equal to integral multiple of . nh
2π mvr =

nh
∴ mvr =
2π Energy of orbits in hydrogen atom (Z = 1)
where m = mass of electron
v = velocity of electron −2.179 × 10−11
E= ergs/atom
r = radius of orbit n2
n = Principal quantum number −2.179 × 10−18
= 1, 2, 3, 4, ....... = Joules/atom
n2
h = Planck’s constant
−1312
= kJ/mole
 KEY NOTE n2
Angular momentum is the product of moment of inertia(I) and 13.6
= 2 eV
angular velocity(w). n
leV = 1.602 × 10–19 J
When an electron drops from a higher state to a lower state, The energy of the electron in a hydrogen atom has a negative sign
energy is released. for all possible otbitals because the energy of the electrons in the
atom is lower than the energy of a free electron at rest.
30 Dropper NEET
For Hydrogen atom like ions. Q. The ionization energy of a hydrogen like Bohr atom is 4
2 Rydberg. What is the radius of the first orbit of this atom ?
Z
E=
n × E1 (Bohr radius of hydrogen = 5 × 10−11 m; 1 Rydberg = 2.2 ×
n2 10−18 J)
where En = Energy of nth orbit in other ions like H-atom Z2 −18
Ans. −13.6 2 =4 R =4 × 2.2 × 10 J
Z = Atomic number n
E1 = Energy of first orbit in hydrogen atom
Difference of energy between two Bohr orbits of hydrogen atom 4 × 2.2 × 10−18 J
2 2 4 = Z2 = −19
4;
= Z 2
2π mZ e 13.6 × 1.6 × 10
=R = 1, 09, 680 x Z2 cm −1
h 3C 1
=r 0.529 × 10−10 × = 2.645 × 10−11 m.
1 1  2
= ∆E Z2 R H  2 − 2  2.65 × 10–11 m.
 n1 n 2 
Q. Which state of the triply ionized Beryllium (Be3+) has the
where Z = atomic number. same orbit radius as that of the ground state of hydrogen
Velocity of electron in hydrogen atom atom?
Ans. Radius of ground state of hydrogen atom = 0.529 Å
2πZe 2 Z
V= = 2.188 × 108 × cm/ sec n2
nh n 0.529
= 0.529 ×
Z
Magnitude of velocity of electron increases with increase of
positive charge on the nucleus and decreases with increases of n2
0.529
= 0.529 ×
principle Quantum Number. 4
⸫n=2
 KEY NOTE
Limitations of Bohr’s Model
Š Number of revolutions per second (or) orbital frequency by
an electron in a shell (i) It failed to explain the line spectra of atoms or ions having
more than one electron.
Velocity v z2
= = = 6.66 × 1015 × 3 (ii) It fails to account the fine spectra details of the hydrogen atom.
Circumference 2πr n (iii) It fail to explain Zeeman’s effect and stark effect.
−E
Š For hydrogen atom, ionization potential = 21 .
n  KEY NOTE
−E1 × Z2 Š The splitting of spectral lines of an atom into a group of fine lines
Š For H atom, like ions, Ionisation potential =
n2 under the influence of a magnetic field is called Zeeman effect.
 Z2  Š The splitting of spectral lines of an atom into group of fine lines
Š Ionisation potential of an atom or ion =13.6  2  eV under the influence of an electric field is called Stark effect.
 n 

De Broglie’s Wave Theory


TRAIN YOUR BRAIN The wave nature of electron was first proposed by de Broglie.
Q. The ratio of (E2 – E1) to (E4 – E3) for He+ ion is approximately According to de Broglie theory all moving particles have wave
equal to (where En is the energy of nth orbit) properties.
(a) 10     (b) 15    (c) 17     (d) 12 Wave properties are important only for particles of small mass
 1 and high velocity.
2 1 
13.6 ( 2 )  2 −  h h
( 2 )2  de Broglie’s equation is=
l =
 (1) = 15 mv p
Ans. (b) where l = wave length
2
 1 1 
13.6 ( 2 )  2 −  h = Planck’s constant
 ( 3) ( 4 )2  v = Velocity of the particle
Q. What is the orbit number of H atom if electron having energy mv = p = Momentum of the particle
is – 3.4 eV? Also report the angular momentum of electron. According to de Broglie’s theory, electrons revolve around the
Ans. E1 for H = – 13.6 eV nucleus in atomic orbits with stationary waves.
E1 −13.6
Now = En 3.4
∴ −= ∴n 2
= Electrons revolves in those orbits, whose circumference must be
n2 n2 equal to integral multiple of wave length.
Now, Angular momentum (mvr)
\ 2pr = nl
h 2 × 6.626 × 10−34
n. = 2.1 10−34 J − sec −1
=× where r = radius of the orbit
2π 2 × 3.14 n = 1, 2, 3, 4 .....
Structure of Atom 31
HEISENBERG’S UNCERTAINTY PRINCIPLE For a system (such as an atom or a moleule whose energy does not
It is impossible to determine simultaneously, the exact position change with time) the Schrodinger equation is written as Ĥ ψ = Eψ
and exact momentum (or velocity) of an electron” It is called where Ĥ is a mathematical operator called Hamiltonian operator.
Heisenberg’s uncertainty principle. h2  ∂2 ∂2 ∂2 
Ĥ =V−  + + 
The uncertainty principle equation is 8π2 m  ∂x 2 ∂y 2 ∂z 2 
h
 x . p  Important Features of Quantum Mechanical Model of

Atom
h h The energy of electrons in atoms is quantized (i.e., can only have
  x .m  v  ⇒ ∆x.∆v ≥
4π 4 πm certain specific values), for example when electrons are bound to
where Dx = uncertainty in position the nucleus in atoms.
Dp = uncertainty in momentum Exact position and exact velocoity of an electron in an atom
Dv = uncertainty in velocity cannot be determined simultaneously (Heisenberg uncertanity
m = mass of the particle principle). the path of an electron in an atom thus, can never be
h = Planck’s constant determined or known accurately.
If Dx = 0 i.e., the position of the electron determined accurately An atomic orbital is the wave function y for an electron in an
then Dv = ∞ atom.
If Dv = 0 i.e., the velocity of the electron determined accurately The probability of finding an electron at a point within an atom is
2
then Dx = ∞ proportional to the square of the orbital wave function i.e., ψ
2
TRAIN YOUR BRAIN at that point. ψ is known as probability density and is always
Q. Alveoli are the tiny sacs of air in the lungs whose average positive.
2
diameter is 50 pm. Consider an oxygen molecule trapped From the value of ψ at different point within an atom, it is
within a sac. Calculate uncertainty in the velocity of oxygen possible to predict the region around the nucleus where electron
molecule? will most probability be found.
(a) 1.98 × 10–2 ms–1 (b) 19.8 ms–1 The probability of finding an electron at a certain distance from
(c) 198 × 10–4 ms–1 (d) 19.8 × 10–6ms–1 the nucleus is called radial probability.
Ans. (b) Δx = 50 pm Shapes of Atomic Orbitals
h 6.625 × 10−34 × 6.022 × 1023 The space around the nucleus of an atom in which there is a
= ∆v = −3 −12
m / sec
4π.m∆x 4 × 3.14 × 32 × 10 × 50 × 10 maximum probability of finding an electron is called an orbital.
= 0.0019853 × 104 m/sec The variation of y(r) as function of r for 1s and 2s orbitals is
= 19.853 m/sec given as follows
(1s) (2s)
QUANTUM MECHANICAL MODEL OF ATOM & (z) 160 50
40
ITS IMPORTANT FEATURES 120 ψ(r) 30
The branch of science that takes into account this dual behaviour (r) 80 20
of matter is called quantum mechanics. 10
40 0
The fundamental equation of quantum mechanics was developed
0 –10
by Schrodinger. 0.4 0.8
0 0.4 0.8 0
Schrodinger’s wave equation is r(nm) r(nm)

∂2ψ ∂2ψ ∂2ψ 8π 2 m The variation of y2(r) as function of r for 1s and 2s orbitals is
2
+ 2
+ 2
+ 2
(E − V) ψ =
0
given as follows
∂x ∂y ∂z h
where y = Wave function 5000 (1s) 300
(2s)
m = Mass of electron 4000 240
h = Planck’s constant 2
 (r)
3000  (r)
2 180

E = Total energy of the electron 2000 120


1000 60
V = Potential energy of electron 0
0
x, y, z are co-ordinates. 0 0.4 0.8 0 0.4 0.8
r(nm)
The solution of Schrodinger wave equation gives principal, r(nm)

azimuthal and magnetic quantum number but not the spin quantum The region where probability density (y2) is zero called nodes
number y indicates the amplitude of the wave. (or) nodal surface.
32 Dropper NEET
s-orbital dyz d x 2  y2 d z2
For ns-orbital, (n-1) nodes are present. z z z
Eg: For 2s-one, 3s-two
The shape of s orbital ( l = 0 ) is spherical.
y
x x
xy
y
(c) (d) (e)

Fig: Shape of d-orbital


These are degenerate orbitals.
Fig: Shape of s-orbital The shape of a d – orbital ( l = 2 ) is double dumb bell.
Size of s orbital increases with increase in n, that is 4s > 3s > 2s > ls. d xy, d yz and d zx orbitals are oriented in between the axes.
s – orbital is a non directional orbital.
d x 2 − y2 and d z 2 orbitals are oriented along the axes.
p-orbital Orbital : d xy d yz d zx d x 2 − y2 dz 2
In a p-sub shell, the three orbitals are represented as px, py, and pz
These are degenerate orbitals. m:   ±2 ±1 ±1 ±2    0
The shape of a p - orbital ( l = 1 ) is dumb bell.
 KEY NOTE
2px
Š Number of nodal planes (or) angular nodes for an orbital = l.
2py
z z Š When the number of nodal planes increases, the energy of
the orbital increases. So the energy order of the orbitals is s
x x <p<d<f
Š Number of radial nodes = n – l – 1
y y where n = principal quantum number
2pz l = Azimuthal quantum number
z Š Total numbers of nodes = (n–1) i.e., sum of angular nodes
and radial nodes.
x
Quantum Numbers
y A set of numbers used to provide a complete description of an
electron in an atom are called quantum numbers.
Fig: Shape of p-orbital There are four quantum numbers required for a complete
explanation of electrons in an atom.
p – orbitals are oriented along the axes. So they are directional
orbitals. The quantum numbers are
1. Principal quantum number
Orbital: Px Py Pz 2. Azimuthal quantum number
m: ±1 ±1 0 3. Magnetic quantum number
4. Spin quantum number
With the increase of principle quantum number size, and energy
Principal quantum number : Represented by “n”
of ‘p’ orbitals increases 4p > 3p > 2p.
It represents the name, size and energy of the shell to which
d-orbital electron belongs.
Greater the value of n greater is the distance from the nucleus.
● In a d - sub shell, the five orbitals are represented as
Greater the value of n greater is the energy of shell
d xy, d yz, d zx, d x 2 − y2 and d z 2 .
E1 < E2 < E3 < E4 – – – – – – – –
dxy dxz The number of electrons in a particular shell is equal to 2n2
z z
Azimuthal quantum number / Angular quantum number:
Represented by “”
● It represents the shape of the subshell and orbital.
x ● It is used for calculating orbital angular momentum.
x
● Value of  is between 0 to (n – 1)
y y i.e.  = 0,1,2 – – – – – – – – – (n–1)
(a) (b)
 = 0(s Subshell)
Structure of Atom 33
 = 1(p Subshell) RULES FOR FILLING OF ORBITALS
 = 2(d Subshell)
Aufbau Principle
 = 3(f Subshell)
Principle: The electrons are filled up in increasing order of the
If the value of n is same then the order of energy of the various
energy of the subshells. The subshell with minimum energy is
subshells will be s < p < d < f
filled up first and when this subshell obtains maximum quota of
If Value of  is same but value of n is different then the order of electrons then the next subshell of higher energy starts filling.
energy will be. 1s < 2s < 3s < 4s < 5s < 6s
The sequence in which various subshell are filled is the following.
The orbital angular momentum is equal to
h  h 
l ( l + 1) or l ( l + 1)    = 
1s 2s 3s 4s 5s 6s 7s
2π  2π  Starting
point 2p 3p 4p 5p 6p 7p
The number of electron in a particular subshell is equal to
2(2 +1)
Magnetic quantum number /Orientation quantum number (m) 3d 4d 5d 6d

● It represents the orientation of electron cloud (orbital) i.e.


direction of electron density. 4f 5f

● Value of m = All integral values from –  to + , including zero.


Spin Quantam number (s) (n + ) rule
● It represents the direction of electron spin around its own
Principle of ( n + ) rule
axis.
1 The subshell with lowest(n + ) value is filled up first, but when
● Value of s = ± two or more subshells have same (n + ) value then the subshell
2
● Spin angular momentum of an with lowest value of n is filled up first.
h
e− = s ( s + 1) . or s ( s + 1) Subshell n l n+l

● Total spin = no. of unpaired electron × +1/2 1s 1 0 1

 KEY NOTE 2s 2 0 2

Magnetic moment ( µ ) is given by= µ n(n + 2) 2p 2 1 3


where n is number of unpaired electron
3s 3 0 3

TRAIN YOUR BRAIN 3p 3 1 4


Q. Calculate total spin, magnetic moment for the atoms having
4s 4 0 4
at. no. 7, 24 and 36.
Ans. The electronic configuration are 3d 3 2 5

7N : 1s2, 2s2 2p3 unpaired electron = 3 4p 4 1 5

24Cr : 1s2, 2s2 2p6, 3s2 3p6 3d5, 4s1 unpaired electron = 6 5s 5 0 5
2 2 6 2 6 10 2 6
36Kr : 1s , 2s 2p , 3s 3p 3d , 4s 4p unpaired electron = 0
4d 4 2 6
⸫ Total spin for an atom = ± 1/2 × no. of unpaired electron
5p 5 1 6
For 7N, it is = ± 3/2 ; For 24Cr, it is = ± 3 ;
For 36Kr, it is = 0 6s 6 0 6

Also magnetic moment = n ( n + 2) Pauli’s Exclusion principle


In 1925, Pauli stated that no two electrons in an atom can have
For 7N, it is = 15 ; For 24Cr, it is = 48 ;
same values of all four quantum numbers.
For 36Kr, it is = 0
Energy of orbitals × ×
The energy of an electron in a hydrogen atom is determined by
the principal quantum number. Thus the energy of the orbitals
increases as follows: An orbital can accomodate maximum 2 electrons and that too
1s < 2s = 2p > 3s = 3p = 3d < 4s = 4p = 4d = 4f with opposite spin.

34 Dropper NEET
Hund’s Rule of Maximum Multiplicity Stability of Completely Filled and
According to Hund’s rule electrons are distributed among the Half Filled Subshells
Orbitals of subshell in such a way as to give maximum number The completely filled and exactly half filled sub shells are stable
of unpaired electron with parallel spin. i.e. in a subshell pairing due to the following reason.
of electron will not start until and unless all the orbitals of that 1. Symmetrical Distribution of electrons: It as that symmetry
subshell will get one electron each with same spin. leads to stability, the completely filled or half filled subshells
have symmetrical distribution of electrons in them and they
Exceptions of Aufbau Principle
are therefore more stable.
In some cases it is observed that the actual electornic configuration
2. Exchange Energy: The number of exchanges which can
of an element is slightly different from the arrangement given by
take place is maximum when the subshell is either half
aufbau principal. A simple reason behind this is that the half filled
filled or completely filled. As a result the exchange energy is
and full filled subshell have got extra stability due to symmetry
maximum and so is the stability.
and exchange energy.

Structure of Atom 35
Topicwise Questions

SUB-ATOMIC PARTICLES (c) The emitted electrons have zero energy.


(d) The emitted electrons have energy equal to energy of
1. The charge of an electron was discovered by:
photons of incident light.
(a) J.J. Thomson (b) Neil Bohr 11. In photoelectric effect, the kinetic energy of photoelectrons
(c) Chadwick (d) Millikan increases linearly with
2. Which has highest specific charge? (a) Wavelength of incident light
(a) Na+ (A = 23) (b) Mg2+ (A = 24) (b) Frequency of incident light
3+ (c) Velocity of incident light
(c) Al (A = 27) (d) Si4+ (A = 28)
(d) Atomic mass of the element
3. Which of the following properties of atom could be explained 12. The value of Rydberg constant is:
correctly by Thomson model of atom? (a) 109, 677 cm–1 (b) 19876 cm–1
(a) Overall neutrality of atom (c) 108769 cm –1 (d) 108976 cm–1
(b) Spectra of hydrogen atom 13. What is the lowest energy of the spectral line emitted by the
(c) Position of electrons, protons and neutrons in atom. hydrogen atom in the Lyman series?
(d) Stability of atom 5hR H c 4hR H c 3hR H c 7hR H c
(a) (b) (c) (d)
36 3 4 144
ATOMIC MODELS
14. The scientist who proposed the atomic model based on the
4. The ratio of charge and mass would be greater for:
quantisation of energy for the first time is:
(a) Proton (b) Electron
(a) Max Planck (b) Neil Bohr
(c) Neutron (d) α-particle
(c) De-Broglie (d) Heisenberg
5. (32Ge76, 34Se76) and (14Si30, 16S32) are the examples of
15. The kinetic energy of electron is 3.0 × 10–25 J. The wave
(a) Isotopes and isobars length of the electron is
(b) Isobars and isotones (a) 7965Å (b) 4625Å (c) 91Å (d) 8967Å
(c) Isotones and isotopes 16. A 100 watt bulb emits monochromatic light of wavelength
(d) Isobars and isotopes 400nm. Calculate the number of photons emitted per second
6. The lightest particle is: by the bulb.
(a) α-particle (b) Positron (c) Proton (d) Neutron (a) 20.12 × 1020 s–1 (b) 2.012 × 1020 s–1
(c) 4.969 × 10–19 s–1 (d) 49.69 × 10–19 s–1
DEVELOPMENTS LEADING TO THE BOHR’S
MODEL OF ATOM BOHR’S MODEL FOR HYDROGEN ATOM
7. Which pair is of isoelectronic species? 17. The wave number for the longest wavelength transition in
(a) K+, Cl–, Na+ (b) K­+, Cl–, Ca2+ the Balmer series of atomic hydrogen is
(c) Fe–, Al3+, K+ (d) Fe2+, Cu2+, V3+ (a) 15.2 × 106m–1 (b) 13.6 × 106m–1
6
(c) 1.5 × 10 m–1 (d) 1.3 × 106m–1
8. The Wavelength (in Å) of an emission line obtained for
Li2+ during a electronic transition from n2 = 2 to n1 = 1 is: 18. The ratio of the energies of two different radiations whose
(R = Rydberg constant) frequencies are 3 × 1014 Hz and 5 × 1014 Hz is
(a) 3R/4 (b) 27R/4 (c) 4/3R (d) 4/27R (a) 3 : 5 (b) 5 : 3 (c) 3 : 1 (d) 5 : 1
9. Splitting of spectral lines under the influence of magnetic 19. If the wavelength of green light is about 5000Å, then the
field is called: frequency of its wave is
(a) Zeeman effect (b) Stark effect (a) 16 × 1014 sec–1 (b) 16 × 10−14 sec–1
14
(c) 6 × 10 sec –1 (d) 6 × 10−14 sec–1
(c) Photoelectric effect (d) None of these
10. A metal surface is exposed to solar radiations 20. What is the energy of photons that corresponds to a wave
(a) The emitted electrons have energy less than a maximum number of 2.5 × 10–5cm–1
value of energy depending upon the frequency of the (a) 2.5 × 10–20erg
incident radiation.
(b) 5.1 × 10–23erg
(b) The emitted electrons have energy less than the
maximum value of energy depending upon intensity of (c) 4.96 × 10–21erg
incident radiations. (d) 8.5 × 10–2erg

36 Dropper NEET
21. The frequency of a wave light is 1.0 × 106 sec−1. The wave 32. A species having only one electron has ionization energy of
length for this wave is 11810 kJ mol–1. The number of protons in its nucleus will
(a) 3 × 104 cm (b) 3 × 10−4 cm be:
4
(c) 6 × 10 cm (d) 6 × 106 cm (a) 1 (b) 2 (c) 3 (d) 4
22. Suppose 10–17 J of energy is needed by the interior of human 33. When the electron of a hydrogen atom jump from n = 4 to
eye to see an object. How many photons of green light n = 1 state, the number of different spectral line emitted are:
(λ = 550nm) are needed to generate this minimum amount (a) 15 (b) 9 (c) 6 (d) 3
of energy? 34. The wave number of the spectral line in the emission
(a) 14 (b) 28 (c) 39 (d) 42 spectrum of hydrogen will be equal to 8/9 times the rydberg
23. The wavelength of a spectral line emitted by hydrogen atom constant if the electron jumps from:
16 (a) n = 3 to n = 1 (b) n = 10 to n = 1
in the Lyman Series is cm. What is the value of n2?
15R (c) n = 9 to n = 1 (d) n = 2 to n = 1
(R = Rydberg constant) 35. The radius of the first Bohr orbit of the H-atom is r. Then, the
(a) 2 (b) 3 (c) 4 (d) 1 radius of the first orbit of Li2+ will be:
24. The energy of an electron present in Bohr’s second orbit of (a) r/9 (b) r/3
hydrogen atom is_______ (c) 3r (d) 9r
(a) –1312J atom–1 (b) –328kJ mol–1 36. In which one of the following, the number of protons is
(c) –328J mol–1 (d) –164J mol–1 greater than the number of neutrons but number of protons
is less than the number of electrons?
25. The emission spectrum of hydrogen discovered first and the
region of the electromagnetic spectrum to which it belongs, (a) D3O+ (b) SO2
respectively are- (c) H2O (d) OH–
(a) Lyman, Ultraviolet (b) Lyman, Visible 37. The energy ratio of a photon of wavelength 3000Å and
(c) Balmer, Ultravoilet (d) Balmer, Visible 6000Å is:
26. The velocity of electron in second shell of hydrogen atom is: (a) 1 : 1 (b) 2 : 1 (c) 1 : 2 (d) 1 : 4
(a) 10.94 × 106 m/sec (b) 18.88 × 106 m/sec 38. Bohr’s model can explain:
(c) 1.88 × 106 m/sec (d) 1.094 × 106 m/sec (a) The solar spectrum
27. For which of the following species, Bohr’s theory is not (b) Spectrum of hydrogen molecule
applicable? (c) Spectrum of any atom or ion containing one electron
(a) Be3+ (b) Li2+ (c) He2+ (d) H only
(d) Spectrum of hydrogen atom only
28. If the first ionisation energy of H– atom is 13.6 eV, then the
second ionisation energy of He– atom is: 39. The first line emission of hydrogen atom spectrum in the
(a) 27.2 eV (b) 40.8 eV Balmer species appears at:
(c) 54.4 eV (d) 108.8 eV (a) 5R/36 cm–1 (b) 3R/4 cm–1
(c) 7R/144 cm –1 (d) 9R/400 cm–1
29. When the electrons of hydrogen atom return to L-shell
from shell of higher energy, we get a series of lines in the 40. The radius of 2nd Bohr’s orbit of hydrogen atom is:
spectrum. This series is called: (a) 0.053 nm (b) 0.106 nm
(a) Balmer series (c) 0.2116 nm (d) 0.4256 nm
(b) Lyman series 41. The maximum energy possessed by an electron in when it is
(c) Brackett series present:
(d) Pascher series (a) In nucleus
30. The nitride ion in lithium nitride is composed of: (b) In ground energy state
(a) 7-protons + 10 electrons (c) In 1st excited state
(b) 10-protons + 10 electrons (d) At infinite distance from the nucleus
(c) 7-protons + 7 electrons 42. Which one of the following transition have minimum
(d) 10-protons + 7 electrons wavelength?
31. In which one of the following pairs, the two species are (a) n4 → n1 (b) n2 → n1
both isoelectronic and isotopic? (Atomic number: Cr = 24, (c) n4 → n2 (d) n3 → n1
Ar = 18, K = 19, Mg = 12, Fe = 26, Na = 11) 43. Select the pair where both species have same radius?
(a) 24Mg2+ and 40Ar (b) 39K+ and 40K+ (a) r4He+ and r1H (b) r2Be3+ and r1H
(c) 24Mg2+ and 25Mg (d) 23Na and 24Na+ (c) r2He and r1H (d) r3Li2+ and r1H

Structure of Atom 37
44. The radius of the first orbit of hydrogen atom is 0.52 × 10–8 DE BROGLIE RELATION AND HEISENBERG’S
cm. The radius of the first orbit of He+ ion is UNCERTAINTY PRINCIPLE
(a) 0.26 × 10–8 cm (b) 0.52 × 10–8 cm 56. The velocities of two particles A and B are 0.05 and 0.02
–8
(c) 1.04 × 10 cm (d) 2.08 × 10–8 cm ms–1 respectively. The mass of B is five times the mass of A.
45. The ratio of ionization energy of H and Be+3 is The ratio of their de-Broglie’s wavelength is
(a) 1 : 1 (b) 1 : 3 (c) 1 : 9 (d) 1 : 16 (a) 2 : 1 (b) 1 : 4
46. As we move away from nucleus, the energy of orbit (c) 1 : 1 (d) 4 : 1
(a) Decreases (b) Increases 57. The energy absorbed by each molecule (A2) of a substance
(c) Remains unchanged (d) None of these is 4.4 × 10–19J and bond energy per molecule is 4.0 × 10–19J.
The kinetic energy of the molecule per atom will be:
47. In hydrogen atom, energy of first excited state is –3.4 eV.
(a) 2.2 × 10–19J (b) 2.0 × 10–19J
Then find out KE of same orbit of hydrogen atom
(c) 4.0 × 10–20J (d) 2.0 × 10–20J
(a) +3.4 eV (b) +6.8 eV
58. The uncertainty in the velocity of particle of mass
(c) –13.6 eV (d) +13.6 eV
6.626 × 10–28 Kg is 10–6 m/sec. What is the uncertainty in its
48. The ratio of the velocity of the electron in the third and fifth position in nm?
shell for He+ would be :
(a) 5 : 3 (b) 1 : 2 (c) 3 : 5 (d) 3 : 4 1 2.5
(a) (b)
2π π
49. The wavelength of the electron emitted when in a hydrogen
atom, electron falls from infinity to stationary sate would be 4 1
(Rydberg constant = 1.097 × 107m–1) (c) (d)
π 4π
(a) 91 nm (b) 192 nm
59. The ratio of de-broglie wavelength of a deuterium atom to
(c) 406 nm (d) 9.1 × 10–8 nm that of α-particle when the velocity of former is five times
50. In two H atoms A and B the electrons move around the nucleus greater then that of latter is:
in circular orbits of radius r and 4 r respectively. The ratio of the (a) 4    (b) 10.2 (c) 2     (d) 0.4
times taken by them to complete one revolution is:
60. The mass of the electrons 9.1 × 10–28 gram and uncertainty
(a) 1 : 4 (b) 1 : 2 (c) 1 : 8 (d) 2 : 1 in the velocity equal to 2 × 10–3 cm/ sec. The uncertainty in
51. The equation corresponding to the wave number of spectral the position of an electron is (h = 6.62 10–27 erg sec)
line in the Bracket series (a) 2.9 × 10+2 cm (b) 2.9 × 10–2 cm
(a) R[(1/ 22) − (1/ 42)] (b) R[(1/ 42) − (1/ 52)] –12
(c) 2.9 × 10 cm –1 (d) 2.9 × 10+12 cm–1
2 2
(c) R[(1/ 3 ) − (1/ 5 )] (d) R[(1/ 62) − (1/ 42)] 61. The uncertainty in the positions of an electron and proton
52. The ratio of highest possible wavelength to lowest possible is equal, the ratio of the uncertainties in the velocity of an
wavelength of Lyman series is electron and proton is
(a) 4/3 (b) 9/8 (c) 27/5 (d) 16/5 (a) 103 : 1 (b) 1 : 1838
53. Which of the following lines will have a wave no. equal in (c) 3672 : 1 (d) 1838:1
magnitude to the value of R in the H - Spectral series 62. The de-Broglie wavelength for a proton with a velocity 15%
(a) Limiting line of Balmer series of the speed of light is:
(b) Limiting line of Lyman series (a) 8.8 × 10–12m (b) 8.8 × 10–15cm
–15
(c) 8.8 × 10 m (d) 4.4 × 10–15cm
(c) First line of Lyman series
(d) First line of Balmer series 63. de-Broglie wave length associated with a material particle is
54. If the wave number of the first line in the Balmer series of (a) Inversely proportional to momentum
hydrogen atom is 15000 cm–1 , the wave number of the first (b) Inversely proportional to its energy
line of the Balmer series of Li2+ is (c) Directly proportional to momentum
(a) 1.43 × 104cm–1 (b) 1.66 × 109cm–1 (d) Directly proportional to its energy
5
(c) 13.5 × 10 cm –1 (d) 1.35 × 105cm–1 64. A microscope using suitable photons is employed to locate
55. The ionisation energy for the Hydrogen atom in the ground an electron in an atom within a distance of 0.1Å. What is the
state is 2.18 × 10–18 Jatom–1 . The energy required for the uncertainty involved in the measurement of its velocity?
following process (a) 2.69 × 106ms–1 (b) 5.79 × 105ms–1
He+ (g)→He2+ (g ) + e– is (c) 5.79 × 106ms–1 (d) 4.62 × 106ms–1
(a) 8.72 × 10–18 Jatom–1 65. A cricket ball of 0.5 Kg is moving with a velocity of 100m
(b) 8.72 × 10–19 Jatom–1 per sec. the wavelength associated with its motion is
(c) 4.35 × 10–18 Jatom–1 (a) 1/100m (b) 6.6 × 10–34 m
(d) 2.62 × 10–19 Jatom–1 (c) 1.32 × 10–35m (d) 6.6 × 10–28 m
38 Dropper NEET
66. As per de-broglie formula a microscopic particle of mass 76. If uncertainty in position and momentum are equal, then
100 g and moving at a velocity of 100 cm/sec will have a uncertainly in velocity is:
wavelength of:
(a) 6.6 × 10–29 cm/s (b) 6.6 × 10–30 cm/s (a) 1 h (b) h
–31
2m π 2π
(c) 6.6 × 10 cm/s (d) 6.6 × 10–32 cm/s
67. The wavelengths of electron waves in two orbits is 3 :5. The (c) 1 h (d) h
ratio of kinetic energy of electron m π π
(a) 25 : 9 (b) 5 : 3 77. The de-broglie wavelength associated with a mass of 1kg
(c) 9 : 25 (d) 3 : 5 having K.E. 0.5J is:
68. If Ee, Eα, and Ep represents the kinetic energy of an electron, (a) 6.626 × 10–34 m (b) 13.20 × 10–34 m
α-particle and proton respectively each moving with same –31
(c) 10.38 × 10 m (d) 6.626 × 10–34 Å
de-broglie wavelength then: 78. The velocity of an electron with de Broglie wavelength of
(a) Ee = Eα = Ep (b) Ee > Eα > Ep 1.0 × 102 nm is:
(c) Eα > Ep > Eα (d) Ee > Ep > Eα (a) 7.2 × 105 cm/sec
69. In sommerfeld’s modification of Bohr’s theory the trajectory (b) 72 × 105 cm/sec
of an electron of hydrogen atom is: (c) 7.2 × 104 cm/sec
(a) A perfect ecllipse (d) 3.6 × 105 cm/sec
(b) A close ecllipse like curve, narrower at the perihelion
QUANTUM MECHANICAL MODEL OF ATOM
position and flatter at the aphelion position
(c) A close loop on spherical surface 79. The number of waves made by an electron moving in an
(d) Rosette orbit having maximum magnetic quantum number + 3 is -
70. A particle having a mass of 1 mg has a velocity 1 Km/sec. (a) 4    (b) 3 (c) 5   (d) 6
Calculate the wavelength of the particle: 80. Which have the same number of s-electrons as the d-electrons
in Fe2+?
(a) 6.626 × 10–28 cm (b) 6.626 × 10–29 cm
(a) Li    (b) Na (c) N    (d) P
(c) 6.626 × 10–30 cm (d) 6.626 × 10–31 cm
81. The set of quantum no. not applicable for an electron -
71. The uncertainty in the position of an electron moving with a
velocity of 3 × 104 cm/sec accurate upto 0.011% will be: 1
(a) 2, 0, 0, −
(a) 1.92 cm (b) 7.66 cm 2
(c) 0.175 cm (d) 3.84 cm 1
(b) 3,1, −2, +
72. If uncertainty in position and velocity are equal, then 2
uncertainty in momentum will be: 1
(c) 4, 2, −2, −
1 mh 1 h 2
(a) (b)
2 π 2 πm 1
(d) 6, 0, 0, +
h mh 2
(c) (d) 82. Which of the following pair of ions have same paramagnetic
4πm 4π
moment ?
73. The velocity of particle “A” is 0.1m/sec and that of particle
(a) Cu+2, Ti+3 (b) Mn+2, Cu+2
“B” is 0.05m/sec. If the mass of particle B is 5 times that of
particle A, then the ratio of de-broglie wavelength associated (c) Ti+4, Cu+2 (d) Ti+3, Ni+2
with particle A and B: 83. The atomic number of an element ‘M’ is 26. How many
(a) 2:5 (b) 3:4 (c) 6:4 (d) 5:2 electrons are present in the M-shell of the element in its M3+
state?
74. If de-broglie wavelength of mass ‘m’ is 100 times of its
velocity then its value in term of its mass ‘m’ and planck’s (a) 11 (b) 15 (c) 14 (d) 13
constant “h” is: 84. The minimum angular momentum of an electron with the
magnetic quantum numbers –1 , 0 , +1
h
(a) 1 ´ m (b) 10 3h h 2h 3h
10 h m (a) (b) (c) (d)
2π π π 2π
m
(c) 1 h (d) 10 85. The values of four quantum numbers of valence electron
10 m h
1
75. The wavelength of electron waves in two orbit is in ratio 3:5. of an element X is n = 4, 1 = 0, m = 0, s = . The element
2
The ratio of K.E. of electrons will be: is
(a) 25:9 (b) 15:3 (c) 9:25 (d) 3:5 (a) K    (b) Ti (c) Na   (d) Sc

Structure of Atom 39
86. Which of the following elements has least number of 96. Two particles A and B are in motion. If the wavelength
electrons in its M shell? associated with the particle A is 5.0 × 10–8 m, the wavelength
(a) K    (b) Mn (c) Ni    (d) Sc of particle B having momentum half of A is:
87. Which one of the following set of quantum numbers is not (a) 2.5 × 10–8 m (b) 1.254 × 10–8 m
–7
(c) 1.0 × 10 m (d) 1.0 × 10–8 m
possible for a 4p electron?
1 97. Which one of the following expressions represent the
(a) n = 4, l = 1, m = +1, ms = + electron probability function (D)?
2
(a) 4pr dry2 (b) 4pr2 dry
1 2
(c) 4pr dry 2 (d) 4pr dry
(b) n = 4, l = 1, m = 0, ms = +
2
98. The quantum of light energy is called:
1 (a) Photon (b) Neutron
(c) n = 4, l = 1, m = 2, ms = +
2 (c) Electron (d) Proton
1 99. What is the orbital angular momentum of an electron in
(d) n = 4, l = 1, m = –1, ms = + f-orbital?
2
88. The maximum number of electrons with spin value +1/2 in (a) 1.5h (b) 6h
the orbital with azimuthal quantum number value l = 2 and π π
magnetic quantum number m = +2 is ............... (c) 3h (d) 3h
(a) 5   (b) 6 (c) 3    (d) 1 π π
89. The correct set of n, l, m, quantum numbers for the unpaired 100. The magnetic moment of Ni2+ ion in BM unit is:
electron of chlorine atom is - (a) 1.73 (b) 4.81
(a) 2, 1, 0 (b) 2, 1, 1 (c) 5.96 (d) 2.83
(c) 3, 1, 1, (d) 3, 0 , 0 101. The angular momentum of an electron present in the excited
90. Which quantum number is not related with Schrodinger 1.5h
state of Hydrogen is . The electron present in
equation? π
(a) Principal (b) Azimuthanl (a) Third orbit (b) Second orbit
(c) Magnetic (d) Spin (c) Fourth orbit (d) Fifth orbit
91. Which of the following set of quantum numbers is correct 102. The maximum number of electrons which can be held by
for valence electron of potassium ? subshell with azimuthal quantum number “l” in an atom is
(a) n = 4, l = 0, m = 0, s = –1/2 given by:
(b) n = 3, l = 0, m = 0, s = –1/2 (a) 2 + 1 (b) 2 + 2
(c) n = 4, l = 1 , m = 0, s = –1/2 (c) 2(2 + 1) (d) 2(2 + 2)
(d) n = 3, l = 1, m = –1, s = –1/2 103. How many ‘d’ electrons are present in Cr2+ ion ?
(a) 4 (b) 5
92. Given K L M N (c) 6 (d) 3
2 8 11 2 104. The atomic numbers of elements X,Y and Z are 19, 21 and
The number of electrons present in l = 2 is 25 respectively. The number of electrons present in the M
(a) 3 (b) 6 (c) 5 (d) 4 shells of these elements follow the order.
93. In which transition, one quantum of energy is emitted? (a) Z > X > Y (b) X > Y > Z
(a) n 2 = 4 → n 1 = 2 (b) n 2 = 3 → n 1 = 1 (c) Z > Y > X (d) Y > Z > X
(c) n 2 = 2 → n 1 = 1 (d) All of these 105. How many electrons are present in the M-shell of an atom of
the element with atomic number Z = 24?
94. Which of the following statement concerning the four
quantum number is incorrect? (a) 5 (b) 6
(c) 12 (d) 13
(a) n gives the size of an orbital
(b) l gives the shape of an orbital 106. A transition metal ‘X’ has a configuration [Ar] 3d4 in its +3
oxidation state, its atomic Number is
(c) m gives the energy of the electron in orbital
(a) 25 (b) 26
(d) Both (a) and (b)
(c) 22 (d) 19
95. Consider the ground state of Cr atom (Z = 24). The number
107. An element has 2 electrons in K shell, 8 electrons in L shell,
of electrons with the azimuthal quantum numbers l = 1 and 2
13 electrons in M shell and one electron in N shell. The
are respectively.
element is
(a) 12 and 4 (b) 16 and 5
(a) Cr (b) Fe
(c) 16 and 4 (d) 12 and 5
(c) V (d) Ti
40 Dropper NEET
108. This electronic configuration shows element of: 113. The number of unpaired electron in Fe3+(Z = 26) are:
(a) 5 (b) 6
↑↓ ↑↓ ↑↓ ↑↓ ↑
(c) 3 (d) 4
1s 2s 2p
114. Which of the following statement in relation to the hydrogen
(a) Oxygen (b) Neon
atom is correct:
(c) Fluorine (d) Nitrogen (a) 3s, 3p and 3d orbital all have the same energy
109. Sum of electronic spins of all electrons with the configuration (b) 3s and 3p orbital are of lower energy than 3d orbital
3d7 is (c) Both of these
(a) + 3/2 (b) + 5/2 (d) None of these
(c) + 7/2 (d) 9/2 115. According to Bohr’s theory, the angular momentum of an
110. The quantum number “m” of a free gaseous atom is electron in the 4th orbit is:
associated with: (a) h (b) 2h
(a) Effect volume of orbital 2π π
(b) Shape of orbital (c) 3h (d) 3h
(c) Spatial orientation of orbital 2π π
(d) Energy of orbital in the absence of magnetic field 116. The total number of orbitals in a shell with principle quantum
number n is:
111. The atomic number of element is 17. The number of orbital (a) 2n (b) 2n2 (c) n2 (d) n + 1
containing electron pair in its valence shell is:
117. If the radius of 1st Bohr orbit be a0, then radius of 3rd Bohr
(a) 3 (b) 4 orbit would be:
(c) 6 (d) 8 1
(a) 3a0 (b) 6a0 (c) 9a0 (d) a0
112. The total number of electrons present in all the p– orbital of 9
bromine are (Given: Atomic no. of Br = 35) 118. Which of the following is not possible?
(a) 5 (b) 15 (a) n = 3,  = 0, m = 0 (b) n = 3,  = 1, m = –1
(c) 17 (d) 35 (c) n = 2,  = 0, m = –1 (d) n = 2,  = 1, m = –1

Learning Plus

1. Which of the following conclusions could not be derived 5. Which of the following options does not represent ground
from Rutherford’s α-particle scattering experiment? state electronic configuration of an atom?
(a) Most of the space in the atom is empty (a) 1s2 2s2 2p6 3s2 3p6 3d8 4s2 (b) 1s2 2s2 2p6 3s2 3p6 3d9 4s2
(b) The radius of the atom is about 10–10 m while that of
(c) 1s2 2s2 2p6 3s2 3p6 3d10 4s1 (d) 1s2 2s2 2p6 3s2 3p6 3d5 4s1
nucleus is 10–15 m
(c) Electrons move in a circular path of fixed energy called 6. Nitrogen laser produces a radiation at a wavelength of 337.1
orbits nm. If the number of photons emitted is 5.6 × 1024 . Calculate
(d) Electrons and the nucleus are held together by the power of this laser.
electrostatic forces of attraction (a) 3.33 × 106 J (b) 3.33 × 105 J
2. How much energy is required to ionise a H-atom if the (c) 1.56 × 106 J (d) 15.6 × 108 J
electron occupies n = 5 orbit? 7. Which of the following does not characterise X-rays?
(a) 5.44 eV (b) 10.8 eV (a) The radiation can ionise the gas
(c) 0.544 eV (d) 1.08 eV (b) It causes ZnS to fluorescence
3. The hydride ion is isoelectronic with (c) Deflected by electric and magnetic fields
(a) H+ (b) He+ (d) Have wavelength shorter than ultra-violet rays
(c) He (d) Be 8. In a photoelectric effect, the energy of the photon striking a
4. The momentum of radiation of wavelength 0.33 nm is ……. metallic surface is 5.6 × 10–19 J. The kinetic energy of the
kg m sec–1 ejected electrons is 12.0 × 10–20 J. The work function is :
(a) 2 × 10–24 (b) 2 × 10–12 (c) 2 × 10–6 (d) 2 × 10–48 (a) 6.4 × 10–19 J (b) 6.8 × 10–19 J
(c) 4.4 × 10–19 J (d) 6.4 × 10–20 J
Structure of Atom 41
9. The probability density plots of 1s and 2s orbitals are given 17. Orbital angular momentum depends on:
in figure: (a) l (b) n and l (c) n and m (d) m and s
18. Chlorine exists in two isotopic forms. Cl-37 and Cl-35 but
its atomic mass is 35.5. This indicates the ratio of Cl-37 and
Cl-35 is approximately:
(a) 1 : 2 (b) 1 : 1 (c) 1 : 3 (d) 3 : 1
19. The pair of ions having same electronic configuration
is:
The density of dots in a region represents the probability (a) Cr3+, Fe3+ (b) Fe3+, Mn2+
density of finding electrons in the region. 3+
(c) Fe , Co 3+ (d) Sc3+, Cr3+

On the basis of above diagram which of the following 20. For the electrons of oxygen atom, which of the following
statements is incorrect? statements is correct?
(a) 1s and 2s orbitals are spherical in shape (a) Zeff for an electron in a 2s orbital is the same as Zeff for
(b) The probability of finding the electron is maximum near an electron in a 2p orbital
the nucleus (b) An electron in the 2s orbital has the same energy as an
(c) The probability of finding the electron at a given distance electron in the 2p orbital
is equal in all directions (c) Zeff for an electron in 1s orbital is the same as Zeff for an
(d) The probability density of electrons for 2s orbital electron in a 2s orbital
decreases uniformly as distance from the nucleus (d) The two electrons present in the 2s orbital have spin
increases quantum numbers ms but of opposite sign
10. The work function of a photoelectric material is 3.3 eV. It 21. A ball of mass 200 gm is moving with velocity of 10 m.s–1. If
threshold frequency will be the error in measurement of velocity is 0.1%, the uncertainity
(a) 7.96 × 1014 Hz (b) 7.96 × 1010 Hz in its position is:
(c) 5 × 1033 Hz (d) 4 × 1011 Hz (a) 3.3. × 10–31 m (b) 3.3 × 10–27 m
11. Which of the following statement is not correct about the (c) 5.3 × 10–25 m (d) 2.64 × 10–32 m
characteristics of cathode rays? 22. If waves travelling at same speeds, which of the following
(a) They start from the cathode and move towards the anode matter waves have the shortest wavelength?
(b) They travel in straight line in the absence of an external (a) Electron (b) Alpha particle (He2+)
electrical or magnetic field (c) Neutron (d) Proton
(c) Characteristics of cathode rays do not depend upon the 23. An ion with mass number 56 contains 3 units of positive
material of electrodes in cathode ray tube charge and 30.4% more neutrons than electrons. Assign the
(d) Characteristics of cathode rays depend upon the nature symbol to this ion
55 3+ 59 3+
of gas present in the cathode ray tube (a) 26 Fe (b) 57
26 Fe
3+
(c) 26 Fe (d) 56
26 Fe
3+

12. A gas of mono atomic hydrogen is excited by an energy of 24. A quantum of light having energy E has wavelength equal to
12.75 eV/atom. Which spectral lines of the following are 7200Å . The frequency of light which corresponds to energy
formed in Lyman, Balmer and Paschen series respectively. equal to 3E, is
(a) 3, 2, 1 (b) 2, 3, 1 (a) 1.25 × 1014 s–1 (b) 1.25 × 1015s–1
(c) 1, 3, 2 (d) 1, 2, 3 (c) 1.25 × 1013s–1 (d) 1.25 × 1014 s–1
13. The number of radial nodes for 3p orbital is: 25. Bond dissociation energy of AB molecules is 300 kJ/mole.
The number of moles of photons of wavelength 6625Å
(a) 3 (b) 4
requires to dissociate 3 moles of AB molecules is
(c) 2 (d) 1
(a) 1 (b) 2 (c) 4 (d) 5
14. Number of angular nodes for 4d orbital:
26. Ultraviolet light of 6.2 eV falls on Cesium surface (work
(a) 4 (b) 3 (c) 2 (d) 1 function = 1.2 eV). The kinetic energy (in electron volts) of
15. Which of the following is responsible to rule out the the fastest electron emitted is approximately
existence of definite paths or trajectories of electrons? (a) 5 eV (b) 4 eV
(a) Pauli’s exclusion principle (c) 3 eV (d) 2 eV
(b) Heisenberg’s uncertainty principle 27. The ionization potential of hydrogen atom is 13.6 eV. The
(c) Hund’s rule of maximum multiplicity wavelength of the energy radiation required for the ionization
(d) Aufbau principle of H-atom
(a) 1911 nm (b) 912 nm
16. Total number of orbitals associated with third shell will be:
(c) 68 nm (d) 91.2 nm
(a) 2 (b) 4 (c) 9 (d) 3
42 Dropper NEET
28. Each hydrogen atom is excited by giving 10.2eV. The 32. Which set of quantum numbers is possible for the last
maximum number of spectral lines in the emission is equal electron of Mg+ ion -
to (a) n = 3 , l = 2 , m = 0 , s = +1/2
(a) 1 (b) 2 (c) 3 (d) 4 (b) n = 2 , l = 3 , m = 0 , s = +1/2
29. The ratio of the difference in energy between the first and the (c) n = 1 , l = 0 , m = 0 , s = +1/2
second Bohr orbit to that between the second and the third (d) n = 3 , l = 0 , m = 0 , s = +1/2
Bohr orbit is 33. The number of radial nodes, nodal planes for an orbital with
(a) 1/2 (b) 34/19 (c) 102/121 (d) 27/5 n = 4 ; l = 1 is
(a) 3, 1 (b) 2, 1 (c) 2, 0 (d) 4, 0
30. Uncertainty in position of a particle of 25 gram in space
is 10–5 m. Hence uncertainty in velocity (m/sec) is 34. A single electron in an ion has ionization energy equal to
(h = 6.6 10–34 J −sec) 217.6 eV. What is the total number of neutrons present in one
ion of it?
(a) 2.1 × 10–28 (b) 2.1 × 10–34
(a) 2 (b) 4 (c) 5 (d) 9
(c) 0.5 × 10–34 (d) 5 × 10–24
35. Which one of the following transitions of an electron in
31. An electron of a velocity ‘x’ is found to have a certain hydrogen atom emits radiation of the lowest wavelength?
wavelength. The velocity to be possessed by the neutron to
(a) n2 = ∞ to n1 = 2
have half the de-Broglie wavelength possessed by electron
(b) n2 = 4 to n1 = 3
is:
(a) x/1840 (b) x/920 (c) n2 = 2 to n1 = 1  
(c) 3680x (d) x/3680 (d) n2 = 5 to n1 = 3

Multiconcept MCQs

1. An electronic transition in hydrogen atom results in the 5. Which of the following statements is false ?
formation of the Hα spectral line in Lyman series of hydrogen (a) The energy of red photon is more than the energy of
atomic spectrum is formed due to an electronic transition in violet photon
hydrogen atom. The energies associated with the electron in (b) The momentum of photon is inversely proportional to its
each of the orbits involved in the transition (in kcal mol–1) are: wave length
(a) –313.6, –34.84 (b) –313.6, –78.4 (c) The energy of a photon is inversely proportional to its
(c) –78.4, –34.84 (d) –78.4, –19.6 wave length
2. In an atom, an electron is moving with a speed of 600 m/s (d) The particle nature of electromagnetic radiations is able
with an accuracy upto 0.005%. What is the uncertainty in to explain the photoelectric effect
position?
6. Photons of frequency 3.2 × 1016 Hz is used to irradiate a
(a) 1.52 × 10–4 m (b) 5.10 ×10–3 m
–3
metal surface, the maximum kinetic energy of the emitted
(c) 1.92 × 10 m (d) 3.84 × 10–3 m th
photo-electron is 3 of the energy of the irradiating
3. Photon having wavelength 310 nm is used to break the 4
bond of A2 molecule having bond energy 288 kJ/mol, then photon? What is the threshold frequency of the metal?
the percentage of energy of photon converted to KE is
(a) 2.4 × 1025 Hz (b) 2.4 × 1016 Hz
(1 eV = 96 kJ/mol):
(c) 1.6 × 1015 Hz (d) 8 × 1015 Hz
(a) 25   (b) 50 (c) 75   (d) 80
7. For the energy levels in an atom which one of the following
4. O2 undergoes photochemical dissociation into one normal
statements is correct :
oxygen and one excited oxygen atom, 1.967 eV more
energetic than normal. The dissociation of O2 into two (a) The 4s sub-energy level is at a higher energy than the 3d
normal atoms of oxygen atoms requires 498kJ mole–1. What sub-energy level
is the maximum wavelength effective for photochemical (b) The second principal energy level can have four orbitals
dissociation of O2? and contain a maximum of 8 electrons
(a) 1645.3 Å (b) 1741.5 Å (c) The M-energy level can have maximum of 32 electrons
(c) 1545.2 Å (d) 1942. 3 Å (d) None of these
Structure of Atom 43
1 14. Match the following
8. If an electron travels with a velocity of th speed of light
100 A Energy of ground state of He+ (i) + 6.04 eV
in Bohr’s first orbit, what is its velocity (relative to the speed
of light) in the 5th Bohr orbit? B Potential energy of Ι orbit of (ii) –27.2 eV
(a) 0.002 (b) 0.1 H-atom
(c) 0.5 (d) 0.7 C Kinetic energy of ΙΙ excited (iii) 54.4 eV
9. According to Bohr’s theory, En = Total energy , Kn = Kinetic state of He+
energy , Vn = Potential energy , rn = Radius of nth orbit D Ionisation potential of He+ (iv) – 54.4 eV
Match the following:
Column I Column I (a) A-(i) B-(ii) C-(iii) D-(iv)
A VN / KN = ? (i) 0 (b) A-(iv) B-(iii) C-(ii) D-(i)
B If radius of nth orbit ∝ Enx , (ii) –l (c) A-(iv) B-(ii) C-(i) D-(iii)
x=?
(d) A-(ii) B-(iii) C-(i) D-(iv)
C Angular momentum in (iii) –2
lowest orbital 15. Photoelectric emission is observed from a metal surface
with incident frequencies ν1 & ν2 where ν1> ν2. If the kinetic
1 energies of the photoelectrons emitted in the two cases are in
D ∝ Z y, y =
? (iv) 1 the ratio 2:1, then the threshold frequency νo of the metal is-
rn
(a) 2ν2 – ν1 (b) 2ν1 – ν2
(a) A–(iii) B-(ii) C-(i) D-(iv) ν − ν2
(c) 1 (d) ν2 – ν1
2
(b) A-(iv) B-(ii) C-(iii) D-(i)
16. Consider the following sets of quantum number
(c) A-(i) B-(iv) C-(iii) D-(ii)
(d) A-(i) B-(iii) C-(iv) D-(ii) n l m s
(i) 3 0 0 +1/2
10. In a multi-electron atom, which of the following orbitals
described by the three quantum numbers will have the same (ii) 2 2 1 +1/2
energy in the absence of magnetic and electric fields? (iii) 4 3 –2 –1/2
A. n = 1, 1 = 0, m = 0 B. n = 2, 1 = 0, m = 0 (iv) 1 0 –1 –1/2
C. n = 2, 1 = 1, m = 1 D. n = 3, 1 = 2, m = 1
E. n = 3, 1 = 2, m = 0 (v) 3 2 3 +1/2
(a) A and B (b) B and C Which of the following sets of quantum number is not
(c) C and D (d) D and E possible?
11. The frequency of light emitted for the transition n = 4 to (a) (i), (ii), (iii) and (iv) (b) (ii), (iv) and (v)
n = 2 of He+ is equal to the transition in H atom corresponding
(c) (i) and (iii) (d) (ii), (iii) and (iv)
to which of the following?
17. The dissociation energy of H2 is 430.53 kJ mol–1. If H2 is
(a) n = 2 to n = 1 (b) n = 3 to n = 2
dissociated by illumination with radiation of wavelength
(c) n = 4 to n = 3 (d) n = 3 to n = 1
253.7 nm. The fraction of the radiant energy which will be
12. For a hydrogen atom in an n = 4 state, the maximum possible converted into kinetic energy is given by:
z-component of orbital angular momentum is (a) 8.86% (b) 2.33%
(a) 2 (b) 3 (c) 1.3% (d) 90%
(c) 12 (b) 6 18. Light of wavelength λ shines on a metal surface with intensity
x and the metal emits Y electrons per second of average
13. Last line of Lyman series for H-atom has wavelength λ1 Å.
energy, Z. What will happen to Y and Z if x is doubled ?
The 2nd line of Balmer series has wavelength λ2 Å, then:
(a) Y will be double and Z will become half
(a) 16 = 9 (b) 16 = 3
λ1 λ 2 λ 2 λ1 (b) Y will remain same and Z will be doubled
(c) Both Y and Z will be doubled
16 3
(c) 4 = 1 (d) = (d) Y will be doubled but Z will remain same
λ1 λ 2 λ1 λ 2
44 Dropper NEET
19. Arrange the electrons represented by the following set of 23. The diagram shows the energy levels for an electron in certain
quantum numbers in the decreasing order of energy atom. Which transition shown represents the emission of a
(i) n = 4, l = 0, m = 0, s = + 1/2 photon with the most energy?
(ii) n = 3, l = 1, m = 1, s = – 1/2
(iii) n = 3, l = 2, m = 0, s = + 1/2
(iv) n = 3, l = 0, m = 0, s = – 1/2
(a) (i) > (ii) > (iii) > (iv) (b) (iv) > (iii) > (ii) > (i)
(c) (iii) > (i) > (ii) > (iv) (d) (i) > (iii) > (ii) > (iv)
20. Select the correct statement(s):
(a) II
(a) The value of spin only magnetic moment of Co3+ ion (in
(b) I
BM) = 24
(c) IV
(b) The number of radial nodes in a 3p-orbital = 1
(d) III
(c) The number of electrons with (m = 0) in Mn2+ ion = 11
24. Which of the following radial distribution graphs corresponds
(d) All are correct
to l = 2 for H atom for the least value of n for which l = 2 is
21. The probability density curve for 2s electron appears like allowed?

(a) (b) (a)

(c) (d)
(b)

22. The plot of orbital wave function ψ(r) as a function of distance


r of the electron from the nucleus for 2s orbital is.

(c)

(a) (b)

(d)

50 (2s)
40
ψ(r) 30
20
(c) 10 (d)
0
–10
0 0.4 0.8
r(nm)

Structure of Atom 45
NEET Past 10 Years Questions
1. A particular station of All India Radio, New Delhi, broadcasts h
on a frequency of 1,368 kHz (kilohertz). The wavelength of (c) The uncertainty principle is ∆E × ∆t ≥

the electromagnetic radiation emitted by the transmitter is: (d) Half-filled and fully filled orbitals have greater stability
[speed of light, c = 3.0 × 108 ms–1] (2021) due to greater exchange energy, greater symmetry and
(a) 219.2 m (b) 2192 m more balanced arrangement
(c) 21.92 cm (d) 219.3 m 9. The total number of orbitals present for principal quantum
175 number, n = 4 is: (2017-Gujarat)
2. The number of protons, neutrons and electrons in Lu,
71 (a) 30 (b) 12
respectively, are :  (2020)
(c) 15 (d) 16
(a) 104, 71 and 71 (b) 71, 71 and 104
(c) 175, 104 and 71 (d) 71, 104 and 71 10. Which of the following pairs of d-orbitals will have electron
density along the axes? (2016 - II)
3. The number of angular nodes and radial nodes in 3s orbital
(a) d z2 , d x 2 − y2 (b) d xy ,d x 2 − y2
are  (2020 Covid Re-NEET)
(a) 1 and 0, respectively (c) d z 2 , d xz (d) dxz, dyz
(b) 3 and 0, respectively
(c) 0 and 1, respectively 11. How many electrons can fit in the orbital for which: n = 3
and l = 1? (2016 - II)
(d) 0 and 2, respectively
(a) 10 (b) 14 (c) 2 (d) 6
4. 4d, 5p, 5f and 6p orbitals are arranged in the order of
decreasing energy. The correct option is  (2019)
12. Two electrons occupying the same orbital are distinguished
by: (2016 - I)
(a) 5f > 6p > 5p > 4d (b) 6p > 5f > 5p > 4d
(a) Spin quantum number
(c) 6p > 5f > 4d > 5p (d) 5f > 6p > 4d > 5p
(b) Principal quantum number
5. Which of the following series of transitions in the spectrum (c) Magnetic quantum number
of hydrogen atom fall in visible region?  (2019)
(d) Azimuthal quantum number
(a) Lyman series (b) Balmer series
13. Which is the correct order of increasing energy of the listed
(c) Paschen series (d) Brackett series
orbitals in the atom of titanium?
6. Magnesium reacts with an element (X) to form an ionic    (Atomic number Z = 22): (2015 Re)
compoun(d) If the ground state electronic configuration of (X)
(a) 3s 3p 4s 3d (b) 3s 4s 3p 3d
is 1s2 2s2 2p3, the simplest formula for this compound is(2018)
(c) 4s 3s 3p 3d (d) 3s 3p 3d 4s
(a) Mg2X3 (b) MgX2
(c) Mg3X2 (d) Mg2X 14. The angular momentum of electron in ‘d’ orbital is equal to:
 (2015)
7. Which one is a wrong statement? (2018)
(a) 2 3 (b) 0
(a) Total orbital angular momentum of electron in ‘s’ orbital
is equal to zero (c) 6 (d) 2
(b) An orbital is designated by three quantum numbers while 15. What is the maximum number of orbitals that can be
an electron in an atom is designated by four quantum identified with the following quantum numbers:
numbers n = 3, l = l, ml = 0? (2014)
(c) The value of m for dz2 is zero (a) 2 (b) 3 (c) 4 (d) 1
(d) The electronic configuration of N atom is 16. Calculate the energy in joule corresponding to
light of wavelength 45 nm (Planck’s constant
h = 6.63 × 10–34 Js; speed of light c = 3 × 108 ms–1) (2014)
(a) 6.67 × 1011 (b) 4.42 × 10–15
8. Which one is the wrong statement? (2017-Delhi)
(c) 4.42 × 10–18 (d) 6.67 × 1015
(a) The energy of 2s orbital is less than the energy of 2p
orbital in case of hydrogen like atoms 17. What is the maximum number of electrons that can be
h associated with the following set of quantum numbers?
(b) de-Broglie’s wavelength is given by λ = , where (2013)
mv
   n = 3, l = 1 and m = –1.
m = mass of the particle, v = group velocity of the
(a) 10 (b) 6 (c) 4 (d) 2
particle
46 Dropper NEET
18. The value of Planck’s constant is 6.63 × 10–34 Js. The speed of (d) For n = 1, the electron has a more negative energy than
light is 3 × 108 ms–1. Which value is closest to the wavelength it does for n = 6 which means that the electron is more
in nanometer of a quantum of light with frequency of 6 × 1015 loosely bound in the smallest allowed orbit.
s–1? (2013)
20. Maximum number of electrons in a subshell with
(a) 10 nm (b) 25 nm l = 3 and n = 4 is: (2012 Pre)
(c) 50 nm (d) 75 nm (a) 14 (b) 16
 Z2  (c) 10 (d) 12
19. Based on equation, −2.178 × 10−18 J  2 
E= certain
n  21. The correct set of four quantum numbers for the valence
electron of rubidium atom (Z = 37) is: (2012 Pre)
conclusions are written. Which of them is not correct?(2013)
(a) 5, 1, 1, +1/2 (b) 6, 0, 0, +1/2
(a) The negative sign in equation simply means that the
energy of electron bound to the nucleus is lower than (c) 5, 0, 0, +1/2 (d) 5, 1, 0, +1/2
it would be if the electrons were at the infinite distance 22. The orbital angular momentum of a p-electron is given as:
from the nucleus.  (2012 Mains)
(b) Larger the value of n, the larger is the orbit radius. 3h h
(a) (b) 6
(c) Equation can be used to calculate the change in energy 2π 2π
when the electron changes orbit. h h
(c) (d) 3
2π 2π

Structure of Atom 47
ANSWER KEY
Topicwise Questions
1. (d) 2. (d) 3. (a) 4. (b) 5. (b) 6. (b) 7. (b) 8. (d) 9. (a) 10. (a)
11. (b) 12. (a) 13. (c) 14. (b) 15. (d) 16. (b) 17. (c) 18. (a) 19. (c) 20. (c)
21. (a) 22. (b) 23. (c) 24. (b) 25. (d) 26. (d) 27. (c) 28. (c) 29. (a) 30. (a)
31. (b) 32. (c) 33. (c) 34. (a) 35. (b) 36. (d) 37. (b) 38. (c) 39. (a) 40. (c)
41. (d) 42. (a) 43. (b) 44. (a) 45. (d) 46. (b) 47. (a) 48. (a) 49. (a) 50. (c)
51. (b) 52. (a) 53. (b) 54. (d) 55. (a) 56. (a) 57. (d) 58. (d) 59. (d) 60. (a)
61. (d) 62. (c) 63. (a) 64. (c) 65. (c) 66. (c) 67. (a) 68. (d) 69. (d) 70. (b)
71. (c) 72. (a) 73. (d) 74. (b) 75. (a) 76. (a) 77. (a) 78. (a) 79. (a) 80. (d)
81. (b) 82. (a) 83. (d) 84. (b) 85. (a) 86. (a) 87. (c) 88. (d) 89. (c) 90. (d)
91. (a) 92. (a) 93. (d) 94. (c) 95. (d) 96. (c) 97. (c) 98. (a) 99. (d) 100. (d)
101. (a) 102. (c) 103. (a) 104. (c) 105. (d) 106. (a) 107. (a) 108. (c) 109. (a) 110. (c)
111. (a) 112. (c) 113. (a) 114. (a) 115. (b) 116. (c) 117. (c) 118. (c)

Learning Plus
1. (c) 2. (c) 3. (c) 4. (a) 5. (c) 6. (a) 7. (c) 8. (c) 9. (d) 10. (a)
11. (d) 12. (a) 13. (d) 14. (c) 15. (b) 16. (c) 17. (a) 18. (c) 19. (b) 20. (d)
21. (d) 22. (b) 23. (d) 24. (b) 25. (d) 26. (a) 27. (d) 28. (a) 29. (d) 30. (a)
31. (b) 32. (d) 33. (b) 34. (c) 35. (a)

Multiconcept MCQs
1. (b) 2. (c) 3. (a) 4. (b) 5. (a) 6. (d) 7. (b) 8. (a) 9. (a) 10. (d)
11. (a) 12. (b) 13. (b) 14. (c) 15. (a) 16. (b) 17. (a) 18. (d) 19. (c) 20. (d)
21. (a) 22. (d) 23. (d) 24. (c)

NEET Past 10 Years Questions


1. (d) 2. (d) 3. (d) 4. (a) 5. (b) 6. (c) 7. (d) 8. (a) 9. (d) 10. (a)
11. (c) 12. (a) 13. (a) 14. (c) 15. (d) 16. (c) 17. (d) 18. (c) 19. (d) 20. (a)
21. (c) 22. (c)

48 Dropper NEET
2 Structure of Atom

Topicwise Questions
1. (d) It was determined by Millikan’s with a help of oil drop 11. (b) KE of photoelectrons increases linearly with frequency
experiment. of incident light (hv = hv0 + KE or KE = hv – hv0)
4×e 12. (a) 109,677 cm–1.
2. (d) S.C = e/m =
28
 1 1 
13. (c) We know that= ∆E hc R H  2 − 2 
3. (a) Neutrality of atom. n n
 1 2 

4. (b) The ratio of charge and mass would be greater for electron For the lowest energy of spectral line in the lyman series.
as the mass of electron is least among the species. n1 = 1, n2 = 2
5. (b) Number of neutrons = M – A
 1 1  1 1 
Number of neutrons in 14Si 30 = 30 – 14 = 16 Hence,
= ∆E hc R H  2 −=  hcR H  − 
 (1) ( 2 )2  1 4 
Number of neutrons in 16S32 = 32 – 16 = 16  

Because of the presence of same number of neutrons, 3hcR H


=
30 32 4
14Si and 16S are called isotones.
14. (b) Neil Bohr utilised the concept of quantisation of energy
Hence; 32Ge­76 , 34Se76 and 14Si30 and 16S32 are the isobars
(proposed by max planck) first time to give a new model
and isotones respectively. of an atom.
6. (b) Positron is lightest.
h
7. (b) Isoelectronic species have same number of electrons. 15. (d) λ =
+ 2m K.E
19K : Number of electrons = 19 – 1 = 18

17Cl : Number of electrons = 17 + 1 = 18 hc Eλ
2+ 16. (b) E= n ⇒ n=
20Ca : Number of electrons = 20 – 2 = 18 λ hc
hence, K+, Cl– and Ca2+ are isoelectronic species. 17. (c) For Balmer series, transition from n = 2 to n = 3
1  1 1  1 1
8. (d)
=
λ
R H ( Z) 
2
2
− 
 ( n ) ( n )2  ( 
)
v =1.097 × 107  2 − 2 
2 3 
 1 2 
where, RH = R, Z = 3 ( for Li2+, atomic no. 3) E1 v2
18. (a) =
n2 = 2, n1 = 1 E2 v1

1  1 1  R ×9×3 4 c 3 × 1010 cm / sec


= R ( 3)  2 − =
2
 = ⇒λ 19. (c) v= = = 6 × 1014 sec −1
λ  (1) ( 2 )2  4 27R λ 500 × 10−8 cm
 
20. (c) E = hcv
9. (a) Zeeman effect is the splitting of spectral line emitted by
an excited electron under the influence of magnetic field. = 6.625 × 10–27erg sec × 3 × 1010 cm sec–1 × 2.5 × 10–5 cm–1

10. (a) Energy of emitted electrons = hv – hv0. = 4.96 × 10–21 ergs


c 3 × 1010 cm / s 29. (a) The lines are called Balmer series.
21. (a) λ = = = 3 × 104 cm
v 1× 106 30. (a) Li3N → 3Li+ + N3–
(Lithium Nitride) (Nitride ion)
n×h×c
22. (b) E = Number of protons = 7
λ
Number of electrons = 7 + 3 = 10
E λ 10−17 J × 550 × 10−9 m Therefore, nitride ions contain 7-protons + 10 electrons.
n =
=
hc 6.625 × 10−34 × 3 × 108 31. (b) 39K+ and 40K+ both contain 18 electrons; i.e., they are
= 27.6 ≈ 28 isoelectronic and isotopic as they have same atomic
number.
23. (c) Equation for Lyman series
1312
1 1 1  32. (c) =
I.E. 2
× Z 2 kJ mol−1
v= = R  2 − 2  n
λ 1 n 2 
For one electron species in the ground state, n = 1
15R 1 1  ⸫ 1312 × Z2 =11810 or Z2 = 9 or Z = 3
= R 2 − 2
16 1 n 2  ( n )( n − 1)
33. (c) The number of spectral lines =
2
1  15  1
= 2
1=
  ∴ When an electrons jump from n = 4 level, number of
n2  16  16
spectral line: =
( 4 )( 4 − 1) 4 × 3 12
⇒ = = 6
n2 = 4 2 2 2
−1312 34. (a) Wave number of spectral lines in emission series of
24. (b) E n =
2
kJ mole −1
n hydrogen,
−1312
E2 = = −328kJ mole −1  1 1 
4 =ν RH  2 − 2  ...(i)
 n1 n 2 
25. (d) In 1885, Balmer for the first time showed that the wave
numbers of spectral lines present in the visible region in 8
hydrogen spectrum are given by: Given; ν = (RH )
9
 
ν ( cm
= −1
) 109677  1 2 − n12  On putting the value of ν in equation (i), we get
 ( 2) 2 
8  1 1 
Hence; n2 = 3, 4, 5 ........ = RH RH  2 − 2 
9  n1 n 2 
Hence; the Balmer spectrum of hydrogen was discovered
first and it lies in visible region. 8 1 1
= −
(1)
2
9 n 22
26. (d) According to Bohr, velocity of an electron (v)
Z 8 1 1 1
= 2.188 × 106 m/ sec 1− = ⇒ = 2
9 n 22 9 n2
n
for H ; Z = 1 n = 2 (given) n2 = 3
2.188 × 106 × 1 ∴ electrons jumps from n2 = 3 to n1 = 1
∴V
= = 1.094 × 106 m/sec
2 35. (b) Radius of H-like species,
27. (c) Bohr model of an atom is applicable for those species 0.529 × n 2
As rn = is radius for first Bohr orbit of H-atom
which have only 1 electron but He2+ do not have any Z
electron. Therefore, this model is not applicable to He2+ which is equal to r (given).
28. (c) Ionisation energy of an atom of atomic number, Z is given So, radius of first orbit of Li2+.
by (n = 1, Z = 3, for Li2+) is r/3
( I.E )H × Z2 Z2
I.E.
= = 13.6 × eV 36. (d) In OH–, number of protons = 8 + 1 = 9
n2 n2
number of neutrons = 8 + 0 = 8
Thus, second ionisation energy for He-atom is given by
number of electrons = 8 + 1 + 1 = 10
13.6 × ( 2 )
2

=I.E. = eV 54.4 eV Hence, in OH– the number of protons is greater than the
(1)
2
number of neutrons but number of protons is less than the
number of electrons.
18 Dropper NEET
o o 46. (b) Orbitals away from nucleus have higher energy due to:
37.=
(b) λ1 3000 A,
= λ 2 6000 A
(i) The inner electrons which repell them, results in
hc hc hc hc increases their energy.
E=
1 = , E=
2 =
λ1 3000 λ 2 6000 (ii) Sheilding effect or screening effect i.e., the electrons
E1
away from nucleus get less attracted by nucleus.
hc 6000
= × = 2 :1
E 2 3000 hc KE
47. (a) = −1
E Total
38. (c) As the Bohr’s model applicable only to H-atom and
H-like species containing one electron system. Total energy = –3.4 eV (Given)
1 1 ∴ KE = –(–3.4 eV) = +3.4 eV
39. (a) Rydberg equation;= ν R 2 − 2
 n1 n 2  V0 Z V 5
48. (a) Vn = ⇒ 3=
for Balmer series; n1 = 2, n2 = 3 n V5 3
 9 − 4  5R 1  1 1 
ν R= −1 49.=
(a) v R 2 − 2 
=  36  36 cm 
λ  n1 n2 

40. (c) Second Bohr orbit of hydrogen atom, i.e., n = 2. 1 1 


1.097 × 107 m −1  2 − 2 
=
Atomic number of hydrogen (z) = 1 1 ∞ 
0.529n 2 0.529 × ( 2 )
2

using r
by = = = 1.097 × 107 m–1
z 1 1
o
= 2.116
= A 0.2116 nm or λ = m
1.097 × 107
z2 = 0.91 × 10–7 m = 91 nm
41. (d) As En =
−21.8 × 10−19 ×
n2
1 1 1
−1 50. (c) v ∝ , vA ∝ , vB ∝
Thus; E n ∝  2  where n is the number of orbit. Hence; r r 4r
n
 
as the value of “n” increases, energy of the electron vA
∴ 2
=
also increases; due to this reason; maximum energy is vB
possessed by an electron when it is present at infinite
distance from the nucleus. tA 2pr / v A 1 vA
= =
t B 2 p ( 4 r ) / vB 4 vB
hc
42. (a)  E =
λ 1 1 1
= × = = 1: 8
1 4 2 8
∴E ∝
λ
∴ Decrease in wavelength, increases the energy. 51. (b) For Bracket series n1 = 4
Energy difference in n4 → n1 transition is maximum. n2 = 5, 6, 7.....
∴ n4 → n1, transition has minimum wavelength
1 1 1 
2 52. (a) = R −  = R
3+ r2 H r1H × 2 λ1 1 ∞ 
43. (b) r2 Be= = = r1H
4 4
1 1 1  3R
n2 o = R −  =
44. (a)=
rn 0.529 × A λ2 1 4  4
Z
λ1 4
 12 o  n 1=
= & z 2 =
=  0.529 × λ2 3
 A  
 2   for He 
= 0.26 Å = 0.26 × 10–8 cm 53. (b)= 1 1 
v R − 
1 ∞ 
45. (d) Ionisation energy of H = 13.6 eV
Ionisation energy of Be+3 (H-like atom) v =R

z2 42 54. (d) v = z 2 vH
= 13.6 × 2
13.6
eV =× 2
= 13.6 × 16
n 1
v= 32 × 1500 cm −1
13.6
∴ ratio (required) = = 1:16 = 135000 cm–1
13.6 × 16
= 1.35 × 105 cm–1
Structure of Atom 19
55. (a) [ IE=
z2 ( −2.18 ×10 ) × 2
−18 2
63. (a) λ =
h
]z [ IE ]h = E=
n mv
n2 12

1
h λ∝
56. (a) λ = mv
mv
h
h λ 64. (c) Dv =
=
λA = ; λB 4pmDx
m × 0.05 5m × 0.02
λ A 5m × 0.02 h 6.625 × 10−34
= = 2 :1 65. (c) =
λ = = 1.32 × 10−35 m
λB m × 0.05 mv 0.5 × 100
66. (c) According to de-Broglie equation;
57. (d) K.E. per atom
h

=
( 4.4 ×10 ) − ( 4.0 ×10 ) J
−19 −19 Wavelength ( λ ) =
mv
2 Given; m = 100g, v = 100 cm/sec
0.4 × 10−19 h = 6.6 × 10–34 Js = 6.6 × 10–27 erg-s
= =J 2.0 × 10−20 J
2 on substituting values, we get

58. (d) According to Heisenberg’s uncertainty principle 6.6 × 10−27 6.6 × 10−27
=λ = = 4
6.6 × 10−31 cm/s
h 100 × 100 10
mDv × Dx ≥
4p 67. (a) According to de-Broglie equation
m = 6.626 × 10–28 kg h
=λ But =
λ1 : λ 2 3 : 5
∆v = 10–6 m/sec mv
h = 6.626 × 10–34 Js \ v1 : v2 = 5 : 3
6.626 × 10 −34
1
6.626 × 10−28 × 10−6 × Dx = KE = mv 2 KE1 : KE 2
4p 2
6.626 × 10−34 1 = 52 : 32 = 25 : 9
=Dx = −34
4p× 6.626 × 10 4p 68. (d) de-Broglie equation
h h h
59. (d) λ = =
λ v
⇒= .....(1)
mv mv mλ
mass of α–particle = 4u, mass of deuterium = 2u K.E. = 1/2 mv2 ...(2)
VD = 5Vα on putting the value of v in e. (2) we get
2
1 1 1  h  1  h2 
=λD =, λα =K.E. =m   
2 × VD 4Vα 2  mλ  2  mλ 2 

λ D 4Vα 4 4 Hence, K.E. ∝ 1/m (if λ is same) and order of mass is as


= = = = 0.4 me < mp < mα
λ α 2VD 2 × 5 10
Thus; the order of K.E. is
h Ee > Ep > Eα
60. (a) Dx.m.Dv =
4p
69. (d)
h
Dx =
4p.m.Dv h 6.626 × 10−27 6.626 × 10−27
70. (b) =
λ = −3 5
⇒ = 6.626 × 10−29 cm
mv 10 × 10 102
Dxe .me .Dve−
61. (d) =1 71. (c) According to Heisenberg principle
Dx p .m p .Dvp
h
Dve m p Dx =
= 4pmDν
Dv p me
6.6 × 10−27 × 100
Dx =
h 6.626 × 10−34 4 × 3.14 × 9.1× 10−28 × 3 × 104 × 0.011
62. (c) =
λ =
mv 1.67 × 10−27 × 0.15 × 3 × 108 ∆x = 0.175 cm
20 Dropper NEET
72. (a) Here; ∆x = ∆v h 6.626 × 10−34
h
77. =
(a) λ =
Dx × Dp ≥ 2mE 2 × 1× 0.5
4p
h = 6.626 × 10−34 m.
Dx × mDv =
4p h
78. (a) λ =
h mv
(Dv) 2 = 79. (a) If m = + 3 (maximum), then  = 3 and n can have
4pm
maximum value of four. Also no. of waves in an orbit =
(⸪ ∆x = ∆v) no. of orbit.
h 80. (d) P(1s2, 2s22p6, 3s2 3p3) has 6 electrons in
Dv =
4pm s-subshells as in d-shell of Fe2+.
∆p = m. ∆v 81. (b) for any value of  the possible value of
h mh m = – to + 
=Dp m =  = 1, m = –1, 0, +1
4pm 4p
82. (a) Cu2+ (3d9) and Ti3+ (3d1) have one unpaired electron each
1 mh
Dp = So they have same paramagnetic moment
2 p
83. (d) Z = 26 = Fe = 1s22s22p63s23p64s23d6
h 1s 2 2s 2 2p6 3s 2 3p6 3d 5
73. (d) λ = Fe3+ =
mv
K L M
h h \ Electron in M shell = 13
=λA = , λB
m A × 0.1 5 × m A × 0.05
84. (b) l = 1
λ A m A × 0.05 × 5 5
= = ⸫ minimum value of n = 2
λB m A × 0.1 2
nh h
mvr
= =
74. (b) Let, the wavelength of particle be x, 2p p
Thus; x = 100 × velocity 85. (a) Valence electron is 4s1
x h
velocity=
(v) =, λ 86. (a) K
= 19 = 1s 2 2s 2 2p6 3s 2 3p6 4s1 K has only 8 electron in
100 mv 
M shell M
h × 100
x= 87. (c) For a p electron 1 value must be one and m values are –1,
m×x
0, +1.
h h 1
x 2 = 100 × ⇒ x = 10 88. (d) An orbital can accommodate one electron with s = +
m m 2
h
89. (c) 3 p 5 : ↑↓ ↑↓ ↑
75. (a) de-broglie equation is λ = n = 3, l = 1 m = 1
mv

2K.E. 90. (d) Spin quantum number is not related with Schrodinger
=K.E. 1= mv 2 or v wave equation.
2 m
91. (a) Electronic configuration of K = 1s22s22p63s23p64s1
h For valence electron (4s1)
λ=
2m K.E.
92. (a) Electronic configuration is
2 2
K.E1 λ 5 25 1s 2 2 s 2 2 p 6 3s 2 3 p 6 3d 3 4 s 2
= =  = 
2
2
K.E 2 λ 3 9
1
93. (d) (Fact Based) All transition emit one quantum of energy
76. (a) According to Heisenberg’s uncertainty principle but the energy of each quantum differs.
h 94. (c) m = represents the orientation of orbital in magnetic field.
Dx × Dp ≥
4p m = orbitals
Here ∆x = ∆p and ∆p = m.∆v 95. (d) Configuration of Cr (Z = 24) is 1s2 2s22p63s23p63d54s1
1 = 1 denotes p-electrons (2p6, 3p6) 1 = 2, denotes
h 1 h
∴ ( Dv 2 )
= 2
or Dv
= d-electrons (3d5)
m 4p 2m p
Structure of Atom 21
h λA pB 1 109. (a) ↑↓ ↑↓ ↑ ↑ ↑
96. (c) =
λ ∴ = =
p λB pA 2
or λB = 2 × λA = 2 × 5 × 10–8 m = 10–7 m 1 −1
Sum of spin Q. No. = 5 × + 2 ×
2 2
97. (c) D functions is = 4pr2. dry2 = probability function
98. (a) The quantum of light energy is called photon. 5 3
= −1 =
h 2 2
99. (d) Orbital angular momentum
=  (  + 1)
2p
For f-orbital, l = 3 110. (c) Azimuthal quantum number determines shape of the,

h 3( 4) orbital magnetic quantum number, m is associated with


h
∴ orbital angular momentum
= 3 ( 3 + 1) =
2p 2p spatial orientation of the orbital. Principal quantum

2h 3 h 3 3h number (n) determines energy of the electron & size


= = =
2p p p (volume) of the atom.
100. (d) Electronic configuration of 111. (a) Atomic number 17, 1s22s22p63s13p6
Ni-28 is [Ar] 3d8, 4s2 Valence shell has 3s2 3p5 orbit.
Thus, outer orbital diagram for Ni2+ is

Valence shell has 3 orbitals with paired electrons.


⇒n=2
112. (c) Electronic configuration of bromine is
=µ n(n + 2) B.M.
1s2 2s2 2p6 3s2 3p6 4s2 3d10 4p5
=
µ 2(2 + 2) B.M.
Then; the total no. of p–electron in bromine is 17.
µ= 8 113. (a) Electronic configuration of 26Fe is
µ = 2.83 B.M. 1s2 2s2 2p6 3s2 3p6 3d6 4s2
nh 1.5h Fe3+ = 1s2 2s2 2p6 3s2 3p6 3d5
101. (a) Angular momentum
= =
2p p ⸪ Outer orbital diagram of Fe3+ is
n=3
102. (c) The maximum number of electrons which can be held by
electron with Azimuthal quantum number “l” in an atom ∴ the no. unpaired electrons are 5.
is given by 2(2l + 1). 114. (a) The hydrogen atom has configuration 1s1.
103. (a) 1s 2 2 s 2 2 p 6 3s 2 3 p 6 3d 4 So; 3s, 3p & 3d orbital will have the same energy with
respect to 1s orbital.
104. (c) 19 X → 1s 2 2 s 2 2 p 6 3s 2 3 p 6 4 s1 nh
115. (b) L =
2p
Y → 1s 2 2 s 2 2 p 6 3s 2 3 p 6 4 s 2 3d 1
21
n=4
4h 2h
2 2 6 2
Z → 1s 2 s 2 p 3s 3 p 4 s 3d 6 2 5
∴ L= =
25
2p p
Z>Y>X 116. (c) Total no. of orbital in a shell is n2.
n2 × a0
105. (d) E.C. of Cr = 1s 2 2 s 2 2 p 6 3s 2 3 p 6 3d 5 4 s1 117. (c) Radius of Bohr’s orbit =
z
106. (a) Element is ‘Mn’ (3) 2 × a 0
= = 9a 0
107. (a) 2 K 8L 13M 1N 1
118. (c) ⸪ l = 0
1s 2 2 s 2 2 p 6 3s 2 3 p 6 3d 5 4 s1
⸫ m = – l to + l = 0
108. (c) This electronic configuration shows atomic number ‘9’.
But m (given) = –1, so (c) is incorrect
So that element is fluorine.

22 Dropper NEET
Learning Plus
1. (c) Bohr put forward concept of electrons move in a circular h
path of fixed energy called orbits but not derived from 17. (a) Orbital angular momentum = l ( l + 1) 2p
Rutherford’s scattering experiment. So it depend upon only ‘l’
2. (c)= 13.6 13.6
E = = 0.544 eV 18. (c) The fractional atomic mass (35.5) of chlorine is due to
n2 25
the fact that in ordinary chlorine atom. Cl-37 and Cl-35
3. (c) Hydride ion is H– that is isoelectronic with He. are present in the ratio of 1 : 3.
∴ Average atomic mass of
h
4. (a) P = 3 × 35 + 1 × 37 105 + 32
λ =Cl = = 35.5 amu
5. (c) Correct configuration should be 1s2 2s2 2p6 3s23p6 3d10 4 4
4s1 for the copper which has atomic number 29 (29Cu). 19. (b) 24Cr = [Ar]3d5, 4s1 24Cr3+ = [Ar]3d3
Due to extra stability of full filled orbital of d-subshell, 6 2 3+ 5
26Fe = [Ar]3d , 4s 26Fe = [Ar]3d
the last electron enter into d-orbital instead of s-orbital. 5 2 2+ 5
25Mn = [Ar]3d , 4s 25Mn = [Ar]3d
nhc 27Co = [Ar]3d7, 4s2 27Co3+ = [Ar]3d6
6. (a) E =
λ 1 2 3+
21Sc = [Ar]3d , 4s 21Sc = [Ar]
7. (c) X-rays not deflected by electric and magnetic field.
Thus, Fe3+ and Mn2+ have the same electronic
8. (c) E = w + K.E configuration
9. (d) The probability density of electrons first increases then
decreases and after that it begins to increases again as 20. (d) (a) Electrons in 2s and 2p orbitals have different screen
distance increases from nucleus. effect. Hence, their Zeff is different. Zeff of 2s orbital >
Zeff of 2p orbital.
10. (a) Work function Therefore, it is not correct.
W = hv0 (b) Energy of 2s orbital < energy of 2p orbital.
11. (d) It does not depend upon nature of gas. Negatively charged Hence, it is not correct.
material particles are present in cathode rays (c) Zeff of 1s orbital ≠ Zeff of 2s orbital
12. (a) Lyman n1 to n = 1 n1 = 3   Hence, it is incorrect.
(d) For the two electrons of 2s orbital, the value of ms is
Balmer n2 to n = 2 n2 = 2
1 1
Paschen n3 to n = 3 n3 = 1 + and − .
2 2
13. (d) Number of radial nodes = n – l – 1 h
21. (d) Dx =
For 3p orbital = 3 – 1 – 1 = 1 4p.m.Dv
No of radial nodes = 1 h
22. (b) From De-Broglie equation,wavelength, λ =
mv
14. (c) Number of angular node = l
For same speed of different particles, i.e., electron,
For 4th orbital (n = 4) and l = 2 for d-orbital proton, neutron and α-particle,
Number of angular nodes = 2 1
λ∝
15. (b) A German physicist Werner Heisenberg in 1927, stated m
uncertainty principle which states that it is not possible As h is constant. Greater the mass of matter waves, lesser
to determine simultaneously, the exact position and exact is wavelength and vice-versa. In these matter waves,
momentum of an electron. alpha particle (He2+) has higher mass, therefore, shortest
h
wavelength.
Mathematically, Dx × Dp ≥ 23. (d) Let the no. of electrons in A3+ = x
4p
56 = x + 3 + 1.304x
The important implications of the Heisenberg uncertainty
x = 23
principle is that it rules out existence of definite paths or
trajectories of electrons and other similar particles. E1 v1
24. (b) =
E2 v2
16. (c) Total number of orbitals associated with nth shell = n2
∴ Total number of orbitals associated with third shell Nhc
25. (d) E =
(3)2 = 9 λ
Structure of Atom 23
26. (a) KE = E – W h
λ e me .ve
hc 31. (b) =
27. (d) E = λh h
λ
mn .vn
28. (a) Electron excited to 2nd orbit
1 mn vn
=
n ( n − 1) 1 me.ve
No. of spectral lines =
2 2
 1 1 2 × me × ve x
−13.6  −  =Vn =
E2 − E1  4 1 mn 920
29. (d) =
E3 − E2 1 1 32. (d) Electronic configuration : 1s2 2s2 2p6 3s1
−13.6  − 
9 4 For 3s orbital n = 3 , l = 0 , m = 0 , s = +1/2
33. (b) No. of radial nodes = n – l – 1
−3 3 36 27
= = × = = 5.4 Nodel planes = l
4 4 5 5
Z2
−34
34. (c) Ionization energy −217.6 =
−13.6 ×
h 6.626 × 10 12
30. (a)=
DV = z=4
4p.m.Dx 4 × 3.14 × 25 × 10−3 × 10−5
= 2.1× 10−28 m / sec So, it is 94 Be3+ ; no. of neutrons 9 – 4 = 5
35. (a)

Multiconcept MCQs

1. (b) Energy of a quantum level or orbit in hydrogen atom in K.E = 386 kJ/mole – 288 kJ/mole = 98 kJ/mole
kcal mol–1 = –313.6/n2 Percentage of total energy that got converted into
Hα line in Lyman series of hydrogen atomic spectrum
98
corresponds to n1= 1 to n2 = 2. It is the first line in Lyman KE = × 100 =
25%
series. 386
Energy of n = 1 level = –313.6/12 = –313.6 kCal mol–1 4. (b) O2 → ON + Oexcited
Energy of n = 2 level = –313.6/22 = –78.4 kCal mol–1 O2 → ON + ON
2. (c) For numerical purposes, E = 498 × 103 J / mole
= 498 × 103 / 6.023 × 1023J  per molecule = 8.268 × 10–
h 19 J
⇒ Dx. Dν =
4pm
Energy required for excitation = 1.967 eV = 3.146 ×10–
19J
0.005
Dν= × 600= 3 × 10−2 ms −1
100 Total energy required for photochemical dissociation of O2
h 6.626 × 10−34 = 8.268 × 10–19 + 3.146 × 10–19   = 11.414 × 10–19 J
Dx
= = hc/λ = 11.414 × 10–19J
4pmDν 4 × 3.14 × 9.1× 10−31 × 3 × 10−2
λ = 6.626 × 10–34 × 3 × 108 / 11.414 × 10–19= 1.7415
Δx = 1.92 × 10–3 m × 10–7 m = 1741.5 Å
3. (a) E = hv 5. (a) Energy of red photon is less than violet photon.
6.626 × 10−34 × 3 × 108
E= J
310 × 10−9
6. (d)
E = 0.0641 × 10–26 × 109 J
E = 0.0641 × 10–17 J
This is the energy of one photon required to break one
A2 molecule. Apply, hν – hνo = kE
To break 1 mole of A2, Energy of photons required = 3 3
23 −17 6
Apply, hν − hν=
o hν ⇒ h ν − hν = h ν o
6.02 × 10 × 0.0641 × 10 = 0.386 × 10 = 386 kJ/mole 4 4

24 Dropper NEET
hν ν 2nd line of Balmer series refers to transition 4 → 2.
= hν o ⇒ = νo
4 4 1  1 1 
= R 2 − 2 
3.2 × 1016 λ2 2 4 
=ν o =8 × 1015 Hz
4 1 1 1 
= R − 
7. (b) Option b is correct as second energy level have 8 electron λ2  4 16 
in 4 orbital. 1 3
= R × ...(2)
z λ2 16
8. (a) Vn = 2.18 × 106 × ms −1
n Divide equation (1) and (2)
z −1
V=
n Vo × ms 1
n
λ1 R R 16
1 = = ×
Vo = × 3 × 108 = 3 × 106 ms −1 1 3R 1 3R
100 λ2 16
z λ 2 16
3 106 × ms −1
Vn =× = =
n λ1 3
Velocity in 5th Bohr orbit
1 14. (c) A – (iv); B – (ii); C – (i); D – (iii)
3 106 × ms −1 =
V5th =× 0.6 × 106 ms −1
5
15. (a) hν1 – hνo = KE1 ...(1)
Velocity relative to speed of light
hν2 – hνo = KE2 ...(2)
0.6 × 106
0.2 × 10−2 =
= 2 × 10−3 =
0.002
3 × 108 h ( ν1 − ν 0 ) 2
=
9. (a) [A — (iii)]; [B – (ii)]; [C – (i)]; [D – (iv)]. h ( ν2 − ν0 ) 1

10. (d) Orbital having (n + l) same have same energy in absence


ν1 – ν0 = 2ν2 – 2ν0 ⇒ ν0 = 2ν2 – ν1
of magnetic field n is same and orbital is 3d.
11. (a) The frequencies and the wave numbers of transitions in H 16. (b) (ii), (iv), and (v) are not possible
atom and He+ ion are equal. (ii) n=2 l=2 m=1 s = +1/2 l not equal to n
i.e., vH = vHe+ not possible
(iv) n = 1 l = 0 m = –1 s = –1/2 Not possible
2 1 1 1 1
( )
2
or R ( ZH )  = 2
− 2  R Z He+  2 − 2 because m = –1
 n1 n2   n1 n2  where l = 0
1 1 2 1 1  1 1  (v) n=3 l=2 m=3 s = +1/2 Not possible be-
 2 − 2  =( 2 )  2 − 2  = −  cause m = 3 is
n
 1 n2 
  2 4  1 4 
not for l = 2
By comparing above terms:
n1 = 1 and n2 = 2. hc 430.53 × 103
17. (a)
= + K.E.
λ 6.023 × 1023
12. (b) For n = 4, the possible ‘l’ values are 0, 1, 2 & 3.
The z-component of L is given by, 6.626 × 10−34 × 3 × 108 430.53 × 103
K.E. = − 6.9 ×10−20
=
h 253.7 × 10−9 6.023 × 1023
Lz m=
= l ml 
2p
For maximum possible ‘l’ value i.e., 3 and for this ml can 6.9 × 10−20
∴ Fraction = 0.088 =
= 8.86%
have values -3, -2, -1, 0, +1, +2, +3. 7.83 × 10−19
so. ans is 3 18. (d) When intensity is doubled, number of electrons emitted
per second is also doubled but average energy of
13. (b) Last line of lyman series refers to transition ∞ → 1 photoelectrons emitted remains the same.
1 1 1  19. (c) The orbitals described by these sets of quantum numbers
= R 2 − 2 
λ1 1 ∞  are :
1 (i) 4s (ii) 3p (iii) 3d (iv) 3s
=R ...(1)
λ1
Structure of Atom 25
The energy of these orbitals follows the order : 21. (a) Correct graph will be
3d > 4s > 3p > 3s
(iii) (i) (ii) (iv)

20. (d) (a) Co3+ : 1s22s2 2p63s23p63d6 ∴ 4 unpaired electrons


∴=
µ 4 ( 4 +=
2) 24 4.9 BM
=
22. (d)
(b) Number of radial = n – l – 1
 1 1 
Number of radial nodes in 3p orbital = 3 – 1 – 1 = 1 (d) ∆E RhcZ 2  2 − 2  as highest energy difference
23.=
 ni n f 
(c) Number of electrons with (m = 0) in Mn2+  
(1s22s22p63s23p63d5) ion = 1 s (2) + 2s(2) + 2p(2) + between n = 1 and n = 3
3s(2) + 3p(2) + 3d(1) = 11
24. (c)

NEET Past 10 Years Questions


c 7. (d) According to Hund’s Rule of maximum multiplicity, the
1. (d) λ = correct electronic configuration of N-atom is
v
3 × 108
λ
= = 219.298
= m 219.3m
1368 × 103

2. (d) 175
71 Lu
Z = atomic number
Z = No. of Protons = 71 = No. of Electrons
No. of Neutrons = Mass no. – No. of Protons ⸪ Option (d) violates Hund’s Rule.
= 175 – 71 8. (a) The energy of 2s orbital is less than the energy of 2p orbital
= 104 in case of hydrogen like atoms is a wrong statement because:
3. (d) Number of radial nodes = n – l– 1
1s > 2s = 2p > 3s = 3p = 3d ...etc.
Number of angular nodes = l
9. (d) The total number of orbital present in n = 4 is n2.
For 3s orbital,
= (4)2 = 16
l =0
Shell No. of orbital
• Number of radial nodes = 3 – 0 – 1 = 2 s 1
• Number of angular nodes = 0 p 3
d 5
4. (a) n l
f 7
(n + 1) value for, 4d = 4 + 2 = 6
5p = 5 + 1 = 6 10. (a) Among d-orbitals d z 2 & d x2 − y 2 have their electron densities
5f = 5 + 3 = 8 orientated towards axes.
6p = 6 + 1 = 7
11. (c) When n = 3 and l = 1 orbital is 3p, so total number of electron
Lower value of (n + l) signifies lower energy
In case of 4d and 5p, lower value of n in 4d has compare to 5p. that can be filled are 6 but in any orbital only 2 electron can
So, 4d has less energy in comparison to 5p. accumulate.
∴ Correct order of energy will be 5f > 6p > 5p > 4d
12. (a) 2 electron occupying the same orbital can be distinguished using
5. (b) In visible region Balmer series transitions fall in H-spectrum. their spin quantum number (ms) where one is clockwise (↑) and
6. (c) Element (X) electronic configuration other is anti-clockwise (↓). In such case, the value of spin quantum
1s2 2s2 2p3 number changes rest all quantum number remains the same.
So, valency of X will be 3. 13. (a) According to Aufbau’s rule of increasing order of energy for
Valency of Mg is 2. filling up of electron.
Formula of compound formed by Mg and X will be Mg3X2. 3s < 3p < 4s < 3d

26 Dropper NEET
h 18. (c) According to formula:
14. (c) Angular momentum =  (  + 1)
2p hc
For d orbital l = 2 =E or hν
λ
h h
So, angular momentum = 2 ( 2 + 1) = 6 c 3 × 108
2p 2p Where, = ν ⇒ λ
= = 50 nm
λ 6 × 1015
h
=  → called as Planck’s reducing constant 19. (d) The orbitals which are closer to nucleus are more strongly
2p
or Dirac constant. affected by the positive charged field of protons in nucleus.
Whereas when we move away from nucleus & n (principal
15. (d) n=3 l=1 ml = 0
quantum number) increases, the effective influence of positive
l=p 3p = 1 orbital as m = 0 so 3pz. charged field decreases & electron in later (n) orbitals become
l
loosely bounded.
20. (a) n = 4 and l = 3, this represent = 4f
Total number of electron in a sub shell = 2(2l + 1)
= 2 (2 × 3 +1)
= 14
Thus, maximum no. of orbitals identified in 3pz is 1.
21. (c) 37Rb → 1s22s22p63s23p64s23d104p65s1
hc
16. (c) E = from here we can say
λ
6.63 × 10−34 × 3 × 108 for 5s → n = 5
=E = 4.42 × 10−18 J l=0, m=0
45 × 10−9
1
17. (d) n = 3 l=1 ml = –1 s= ±
2
for p orbital l = 1 h
22. (c) Orbital angular momentum =  (  + 1)
So orbital must be 3px or 3py, where number of electron in each
2p
For p-orbital l = 1
orbital will be 2.
h
m
=  (  + 1)
2p
h h h
m= 1(1 + 1) → m= 2 ⇒ m=
2p 2p 2p

Structure of Atom 27
3 Classification of Elements and
Periodicity in Properties

Past Years NEET Trend

4
No. of MCQs

0
2021 2020 2019 2018 2017 2016 2015 2014 2013 2012

Investigation Report
TARGET EXAM PREDICTED NO. OF MCQs CRITICAL CONCEPTS
• Acid base behaviour of oxides and hydroxides,
NEET 0-1 Periodic trends in physical properties of element

Perfect Practice Plan


Topicwise Questions Learning Plus Multiconcept MCQs NEET Past 10 Years Total MCQs
Questions
51 26 19 11 107
GENESIS OF PERIODIC CLASSIFICATION properties should occupied the same vertical columns and leaving
out blank spaces where necessary.
(a) Dobereiner’s Triads
He arranged similar elements in the groups of three elements This table was divided into nine vertical columns called groups
known called as triads, in which the atomic mass of the central and seven horizontal rows called periods.
element was merely the arithmetic mean of atomic weight of Periods Number of Elements Called as
other two elements or all the three elements possessed nearly the
same atomic weight. (1)st n = 1 2 Very short period
Dobereiner’s Triads (2)nd n = 2 8 Short period
Li Na K (3)rd n = 3 8 Short period
7 23 39 (7 + 39)/2 = 23
(4)th n = 4 18 Long period
Fe Co Ni
(5)th n = 5 18 Long period
55.85 58.93 58.71 Nearly same atomic masses
(6)th n = 6 32 Very long period
It was restricted to few elements, hence discarded.
(7)th n=7 19 Incomplete period
(b) Newland’s Law of Octave
The groups were numbered as I, II, III, IV, V, VI, VII, VIII and
He was the first to correlate the chemical properties of the elements Zero group.
with their atomic masses.
● Mendeleev’s predicted the properties of those missing
According to octave, if the elements are arranged in the order of elements from the known properties of the other elements in
their increasing atomic weight the eighth element starting from
the same group.
given one is similar in properties to the first element.
● Eka - aluminium and Eka-silicon names were given
This arrangement of elements is called as Newland’s law of for gallium and germanium (not discovered at the time
Octave.
of Mendeleev’s). Later on it was found that properties
Newlands's Octaves predicted by Mendeleev’s for these elements and those found
Element Li Be B C N O F experimentally were almost similar.
At. wt. 7 9 11 12 14 16 19
Eka-alu- Eka-sili- Germa-
Gallium
Element Na Mg Al Si P S Cl Property minium con (pre- nium
(found)
At. wt. 23 24 27 29 31 32 35.5 (predicted) dicted) (found)

Element K Ca Atomic Mass 68 70 72 72.6


At. wt. 39 40
Density/(g/cm3) 5.9 5.94 5.5 5.36
This classification worked quite well for the lighter elements but it
Melting point (K) Low 30.2 High 1231
failed in case of heavier elements and therefore, it was discarded.
Formula of oxide E2O3 Ga2O3 EO2 GeO2
(c) Lothar Meyer’s Classification
He determined the atomic volumes by dividing atomic masses Formula of chloride ECl3 GaCl3 ECl4 GeCl4
with their densities in solid states. Table: Mendeleev's Predictions for the Elements Eka-aluminium
He plotted a graph between atomic masses against their respective (Gallium) and Eka-silicon (Germanium)
atomic volumes for a number of elements. He found the following
observations. Merits of Mendeleev’s Periodic table
Elements with similar properties occupied similar positions on He has simplified and systematised the study of elements and
the curve. their compounds.
He has helped in predicting the discovery of new elements on the
On the basis of his observations he concluded that the atomic
volumes (a physical property) of the elements are the periodic basis of the blank spaces given in its periodic table.
functions of their atomic weights.
It was discarded as it lacks practical utility . Demerits in Mendeleev’s Periodic Table
● Position of hydrogen is uncertain. It has been placed in IA
(d) Mendeleev’s Periodic Table and VIIA groups because of its resemblance with both the
According to him the physical and chemical properties of the groups.
elements are the periodic functions of their atomic masses. ● No separate positions were given to isotopes.
He arranged the known elements in order of their increasing ● Anomalous positions of lanthanides and actinides in periodic
atomic masses considering the facts that elements with similar table.
50 Dropper NEET
MODERN PERIODIC LAW AND PRESENT FORM ● The names are derived by using roots for the three digits in
OF PERIODIC TABLE the atomic number of the element and adding “ium” at the
end. The roots for the numbers are.
Moseley studied (1909) the frequency of the X-ray produced by
Table: Notation for IUPAC Nomenclature of Elements
the bombardment of a strong beam of electrons on metal target.
He found that the square root of the frequency of X-rays ( )
v Digit
0
Latin Word
nil
Abbreviation
n
is directly proportional to number of effective nuclear charge (Z)
of metal i.e. to atomic number and not to atomic mass of the atom 1 un u
of that metal (as nuclear charge of metal atom is equal to atomic 2 bi b
number), i.e. ( v ) = a (Z - b). 3
4
tri
quad
t
q
Where ‘a’ and ‘b’ are constant. Thus, he, concluded that atomic 5 pent p
number was a better fundamental property of an element than its 6 hex h
atomic weight. Then he suggested that the atomic number (Z) 7 sept s
instead of atomic weight should be basis of the classification
8 oct o
of the elements.
9 enn e
Modern Periodic Law (Moseley’s Periodic Law) Table: Name and Symbols in current Use (or proposed) for
Physical and chemical properties of the elements are the periodic Trans-fermium Elements (Z=101-118)
functions of their atomic number.
Atomic number Systematic 1977 IUPAC 1997
If the elements are arranged in order of their increasing atomic
101 Unnilunium (Unu) Mendelevium (Md)
number, after a regular interval, elements with similar properties are
repeated. 102 Unnilbium (Unb) Nobelium(No)
103 Unniltrium(Unt) Lawrencium(Lr)
Periodicity
104 Unnilquadium(Unq) Rutherfordium(Rf)
The repetition of the properties of elements after regular intervals
105 Unnipentium(Unp) Dubnium(Db)
when the elements are arranged in the order of increasing atomic
number is called periodicity. 106 Unnilhexium (Unh) Seaborgium(Sg)
The periodic repetition of the properties of the elements is due to 107 Unnilseptium(Uns) Bohrium(Bh)
the recurrence of similar valence shell electronic configurations 108 Unniloctium(Uno) Hassium (Hs)
after certain regular intervals. For example, alkali metals have 109 Unnilennium(Une) Meitnerium(Mt)
same valence shell electronic configuration ns1 , therefore, have
110 Ununnillium(Uun) Darmstadtium(Ds)
similar properties.
111 Unununium(Uuu) Rontgenium(Rt)
The modern periodic table consists of horizontal rows (periods)
and vertical column (groups). 112 Ununbium(Uub) Copernicium (Cn)
113 Ununtrium(Uub) Nihonium(Nn)
Periods
114 Ununquadium(Uuq) Flerovium (Fl)
There are seven periods numbered as 1, 2, 3, 4, 5, 6 and 7.
115 Ununpentium(Uup) Moscovium(Mc)
● Each period consists of a series of elements having same
116 Ununhexium(Uuh) Livermorium (Lv)
valence shell.
117 Ununseptium(Uus) Tennessine(Ts)
● Each period corresponds to a particular principal quantum
number of the valence shell present in it. 118 Ununoctium(Uuo) Oganesson(Og)
● Each period starts with an alkali metal having outermost CLASSIFICATION OF THE ELEMENTS
electronic configuration as ns1.
It is based on the type of orbitals which receives the differentiating
Groups electron (i.e., last electron).
(a) s-block elements: When shells upto (n – 1) are completely
● There are eighteen groups numbered as 1, 2, 3, 4, 5, ........... filled and the last electron enters the s-orbital of the outermost
13, 14, 15, 16, 17, 18. (nth) shell, the elements of this class are called s-block elements.
● Group consists of a series of elements having similar valence ● Group 1 & 2 elements constitute the s-block.
shell electronic configuration. ● General electronic configuration is ns1-2
● s-block elements lie on the extreme left of the periodic table.
IUPAC NOMENCLATURE FOR ELEMENTS ● Includes metals.
WITH Z >100 (b) p-block elements: When shells upto (n – 1) are completely
● Nomenclature of elements CNIC (commission on nomenclature filled and last electron enters the p-orbital of the nth orbit, elements
of inorganic chemistry) appointed by IUPAC in 1994, approved of this class are called p-block elements.
a nomenclature scheme as well as also gave official names ● Group 13 to 18 elements constitute the p-block.

for elements after Z > 100 (upto atomic number 104 to 109 ● General electronic configuration is ns2 np1-6
discovered by that time). This nomenclature is to be followed for ● p-block elements lie on the extreme right of the periodic
naming the elements until their names are officially recognised. table.
Classification of Elements ... 51
● This block includes some metals, all non metals and TRAIN YOUR BRAIN
metalloids. Q. Elements A, B, C, D and E have the following electronic
● s-block and p-block elements are collectively called main configurations :
group or representative elements. A : 1s2 2s2 2p1 B : 1s2 2s2 2p6 3s2 3p1
2 2
C : 1s 2s 2p 3s 3p6 2 3 D : 1s2 2s2 2p6 3s2 3p5
(c) d-Block elements
2 2 6
E : 1s 2s 2p 3s 3p 4s2 6 2
When outermost (nth) and penultimate shells (n–1)th shells
are incompletely filled and last electron enters the (n–1) d Which among these will belong to the same group in the
orbitals (i.e., d-orbital of penultimate shell) then elements of periodic table ?
this class are called d-block elements. Ans. Out of these, elements A and B will belong to the same group
● Group 3 to 12 elements constitute the d-block. of the periodic table because they have same outer electronic
configuration, ns2 np1.
● General electronic configuration is (n–1) d1-10 ns0-2 (except,
palladium which has valence shell electronic configuration Q. An element X with Z = 112 has been recently discovered.
4d10 5s0). What is the electronic configuration of the element ? To
which group and period will it belong ?
● d-block elements are classified into four series.
Ans. (a) T he electronic configuration of element X is [Rn]86 5f14
● Those elements which have partially filled d-orbitals in neutral
6d107s2
state or in any stable oxidation state are called transition elements.
(b) It belongs to d-block as last electron enters in d subshell.
● All the transition elements are metals and most of them form
(c) As number of electrons in (n – 1)d subshell and valence shell
coloured complexes or ions.
is equal to twelve i.e. 10 + 2. So it belongs to group 12.
(d) f-Block elements (d) It belongs to period 7 of the periodic table as principal
When n, (n–1) and (n–2) shells are incompletely filled and quantum number of valence shell is 7 (i.e., 7s2).
last electron enters into f-orbital of antipenultimate i.e., (n–2)
th shell, elements of this class are called f-block elements. PERIODIC TRENDS IN PROPERTIES OF ELEMENT
● General electronic configuration is (n – 2) f 1-14 (n – 1) d 0-1 Trends in Physical Properties
ns2 (i) Effective Nuclear Charge: In an polyelectronic atom between
● The elements coming after uranium are called transuranium the outer most valence electrons and the nucleus of an atom,
elements. there exists number of shells containing electrons. Due to the
● They are also called as inner-transition elements as they presence of these inner electrons, the valence electrons are unable
contain three outer most shell in complete and were also to experience the attractive pull of the actual number of protons
referred to as rare earth elements since their oxides were in the nucleus. These inner electrons act as shield between the
rare in earlier days. valence electrons and protons in the nucleus.
The elements of f-blocks have been classified into two series. Thus, the presence of intervening (shielding) electrons reduces
the electrostatic attraction between the protons in the nucleus
1. Ist inner transition or 4 f-series, contains 14 elements58Ce and the valence electrons because intervening electrons repel the
to 71Lu. Filling of electrons takes place in 4f subshell.
valence electrons.
2. IInd inner transition or 5 f-series, contains 14 elements
The effective nuclear charge (Zeff) is the charge actually felt by
90Th to 103Lr. Filling of electrons takes place in 5f subshell.
the valence electron. Zeff is given by Zeff = Z – σ, (where Z is the
● The actinides and lanthanides have been placed at the bottom actual nuclear charge (atomic number of the element) and σ is the
of the periodic table to avoid the undue expansion of the shielding (screening) constant).
periodic table.
(ii) Atomic radius: Atomic radius is taken as the effective size
 KEY NOTE that is the distance of the closest approach of one atom to another
Prediction of period, group and block atom in a given bonding state.
Š Period of an element corresponds to the principal quantum
number of the valence shell.
Types of Atomic Radius
Š The block of an element corresponds to the type of subshell (a) Covalent radius : It is one-half of the distance between the
which receives the last electron. centres of two nuclei (of like atoms) bonded by a single covalent
Š The group is predicted from the number of electrons in the bond as shown in figure.
valence shell or/and penultimate shell as follows.
(a) For s-block elements, Group number = the number of
valence electrons
(b) For p-block elements, Group number = 10 + number
of valence electrons
(c) For d-block elements, Group number = number of
electrons in (n – 1) d sub shell + number of electrons
in valence shell.
Covalent radius is generally used for non metals.

52 Dropper NEET
Single Bond Covalent Radius, SBCR (bond length)
Variation in a Period Variation in a Group
(a) For homodiatomic molecules dA–A = rA + rA or 2rA
d A− A As a result, the electrons are The effect of increased number
so, rA =
pulled closer to the nucleus by of atomic shells overweights
2
(b) For heterodiatomic molecules in which electronegativity 1 the effect of increased nuclear
the increased Zeff. rn ∝ charge. As a result of this the size
remains approximately same. Z* of atom increases from top to
dA – B = rA + rB Hence atomic radii decrease with bottom in a given group.
For heteronuclear diatomic molecule, A–B, where difference increase in atomic number in a
between the electronegativity values of atom A and atom B is period from left to right.
relatively larger,
dA – B = rA + rB – 0.09 Δχ The atomic radius of inert gas (zero group) is given largest in
Electronegativity values are given in Pauling units and radius in a period because it is represented by vander Waals’s radius is
picometers. generally larger than the covalent radius.
Δχ = XA – XB where XA and XB are electronegativity values The Van der Waal’s radius of inert gases also increases from top
of high electronegative element A and less electronegative to bottom in a group.
element B. In the transition series (e.g. in first transition series), the covalent
Van der Waal’s radius (Collision radius) : It is one-half of the radii of the elements decrease from left to right across a row until
internuclear distance between two adjacent atoms in two nearest near the end when the size increases slightly.
neighbouring molecules of the substance in solid state as shown
in figure. The radii of the elements from Cr to Cu, are very close to one
another as the successive addition of d-electrons screen the outer
electrons (4s) from the inward pull of the nucleus. As a result of
this, the size of the atom does not change much in moving from
Cr to Cu.
There are 14 lanthanide elements between lanthanum and
hafnium, in which the antipenultimate 4f shell of electrons (exert
very poor shielding effect) is filled. There is a gradual decrease in
size of the 14 lanthanide elements from cerium to lutetium. This
(c) Metallic radius (Crystal radius): It is one-half distance is called lanthanide contraction. This lanthanide contraction
between the nuclei of two adjacent metal atoms in the metallic
cancels out the normal size increase on descending a group in
crystal lattice as shown in figure.
case of transition elements.
(iii) Ionic radius: It is the effective distance from the centre of
nucleus of the ion that is cation /anion up to which it has an
influence in the ionic bond is know ionic radius.
Cl– (ionic radius Cl (covalent
1.84 Å) radius 0.99 Å)
Number of electrons : 17 18
Number of protons : 17 17
Thus, the covalent, Vander Waal’s and metallic radius magnitude
wise follows the order, So, there is reduction in effective nuclear charge and hence the
rcovalent < rcrystal < rvander waal's electron cloud expands in case of Cl–.
The sizes of ions increases as we go down a group (considering
Table: Atom Radius Variation the ions of same charge). For example :
Li+ (0.76) < Na+ (1.02) < K+ (1.38) < Rb+ → (in Å)
Variation in a Period Variation in a Group
Be2+ < Mg2+ < Ca2+ < Sr2+
In a period left to right In a group top to bottom
F– < Cl– < Br– < I–
Nuclear charge (Z) increases by Nuclear charge (Z) increases by The species containing the same number of electrons but differ in
one unit more than one unit the magnitude of their nuclear charges are called as isoelectronic
Effective nuclear charge (Zeff) Effective nuclear charge (Zeff) species. For example, N3– , O2–, F–, Ne, Na+ , Mg2+ and Al3+ are
also increases almost remains constant because all isoelectronic species with same number of electrons (i.e 10)
of increased screening effect of but different nuclear charges of +7, +8, +9, +10, +11, +12 and +13
inner shells electrons. respectively.
But number of orbitals (n) But number of orbitals (n) Within a series of isoelectronic species as the nuclear charge
remains constant increases. increases, the force of attraction by the nucleus on the electrons
also increases. So, the ionic radii of isoelectronic species decrease
Classification of Elements ... 53
with increases in the magnitude of nuclear charges. For example, (b) Nuclear Charge: The ionisation energy increases with
as shown in figure increase in the nuclear charge.
It is due to the fact that with increase in the nuclear charge,
the electrons of the outermost shell are tightly held by the
nucleus and hence large amount of energy is required to pull
out an electron from the atom.
As effective nuclear charge decrease. Ionisation energy increases as we move from left to right
● Following are the examples of isoelectronic series
along a period due to increase in nuclear charge.
(i) S2–, Cl–, K+ , Ca+2 , Sc+3 (ii) SO2, NO3– , CO32–,
(c) Shielding or screening effect: The electrons in the innermost
(iii) N2 , CO, CN– (iv) NH3 , H3O+
shells will act as a screen or shield between the nucleus and
● Pauling’s empirical formula for ionic radius
the electrons in the outermost shell. This is called shielding
1 effect. The larger the number of electrons in the inner shells,
∝ (only for isoelectronic species)
nuclear ch arg e greater is the screening effect and smaller the force of
attraction and thus ionization energy (IE) decreases.
TRAIN YOUR BRAIN Screening Effect
Q. X – X bond length is 1.00 Å and C–C bond length is 1.54 Å.
If electronegativities of X and C are 3.0 and 2.0 respectively,
then C–X bond length is likely to be ? (using Stevension &
Schomaker formula).
Ans. rC–X = rC + rX – 0.09 Δχ
1.00 1.54 (d) Penetration effect of the electron: The ionization energy also
=+ − 0.09 [ ∆χ= 1=
] 1.27 − 0.09 depends on the type of electron that is removed. s, p, d and
2 2 f electrons have orbitals with different shapes. An s electron
C–X bond length = 1. 18 Å. penetrates closer to the nucleus, and is therefore more tightly
Q. Mg2+ is smaller than O2– in size, though both have same held than a p electron. Similarly p-orbital electron is more
electronic configuration. Explain ? tightly held than a d-orbital electron and a d-orbital electron
Ans. Mg2+ and O2– both are isoelectronic i.e., have same number is more tightly held than an f-orbital electron.
of electrons. But Mg2+ having 12 protons in its nucleus apply If other factors being equal, penetration order are in the order
s > p > d > f.
a higher effective nuclear charge than O2– having 8 protons
and thus valence shell as well as inner shells electrons are (e) Electronic Configuration: If an atom has exactly half-filled
more strongly attracted by the nucleus in Mg2+ resulting or completely filled orbitals, then such an arrangement has
smaller size than O2–. extra stability.
(iv) Ionisation Energy (IE): Ionisation energy (IE) is defined as The removal of an electron from such an atom requires more
the amount of energy required to remove the most loosely bound energy then expected.
electron from an isolated gaseous atom to form a cation. (v) Electron Affinity: The electron gain enthalpy (ΔegH) is the
change in standard molar enthalpy when a neutral gaseous atom
gains an electron to form an anion.
M ( g )  → M + ( g ) + e − : M + ( g ) + IE2 → M 2 + ( g ) + e −
1 E

X (g) + e– (g) → X– (g)


M2+ (g) + IE3 → M+3 (g) + e– Electron gain enthalpy provides a measure of the ease with which
IE1, IE2 & IE3 are the Ist, IInd & IIIrd ionization energies to an atom adds an electron to form anion. Electron gain may be
remove electron from a neutral atom, monovalent and divalent either exothermic or endothermic depending on the elements.
cations respectively.
When an electron is added to the atom and the energy is released,
In general, (IE)1 < (IE)2 < (IE)3 < .............., as the number of the electron gain enthalpy is negative and when energy is needed
electrons decreases, the attraction between the nucleus and the to add an electron to the atom, the electron gain enthalpy is
remaining electrons increases considerably and thus subsequent positive.
ionization energies increase.
Ea = E (X, g) – E (X–, g),
● It is measured in kJ mol–1, k Cal mol–1, eV (electron volt).
An element has a high electron affinity if the additional electron can
● Factors Influencing Ionisation energy
enter a shell where it experiences a strong effective nuclear charge.
It is influenced by the following factors.
Across a period, with increase in atomic number, electron gain
(a) Size of the Atom: Ionisation energy decreases with increase
enthalpy becomes more negative because left to right across a
in atomic size.
period effective nuclear charge increases and as a result it will be
As the distance between the outermost electrons and the easier to add an electron to a small atom.
nucleus increases, the force of attraction between the valence
shell electrons and the nucleus decreases. As a result, outer As we move in a group from top to bottom, electron gain enthalpy
most electrons are held loosely and lesser amount of energy becomes less negative because the size of the atom increases and
is required to break them. the added electron would be at larger distance from the nucleus.

54 Dropper NEET
The electronegativity also increases as the s-character in the
 KEY NOTE hybrid orbitals increases.
Š Group 17 elements (halogens) have very high negative Hybrid orbital sp3 sp 2 sp
electron gain enthalpies (i.e. high electron affinity) because
s −character 25% 33% 50%
they can attain stable noble gas electronic configuration by →
Electronegativity increases
picking up an electron.
Š Noble gases have large positive electron gain enthalpies Variation of Electronegativity in a Group
because the electron has to enter the next higher energy level
leading to a very unstable electronic configuration. On moving down the groups, Z increases but Zeff almost remains
constant, number of shells (n) increases, rn (atomic radius) increases.
Thus, electronegativity decreases moving down the groups.
Negative electron gain enthalpy of O or F is less than S or Cl. It is
due to the fact that when an electron is added to O or F, the added Variation of Electronegativity in a Period
electron goes to the smaller n = 2 energy state and experiences While moving across a period left to right, Z, Zeff increases & rn
significant repulsion from the other electrons present in this level.
decreases. Thus, electronegativity increases along a period.
In S or Cl, the electron goes to the larger n = 3 energy state and
consequently occupies a larger region of space leading to much Measurement of Electro negativity
less electron-electron repulsion. (a) Pauling’s scale : Linus Pauling developed a method for
Nitrogen has very low electron affinity because there is high calculating relative electronegativities of most elements.
electron repulsion when the incoming electron enters an orbital According to Pauling
that is already half filled.
1 Δ = XA – XB = 0.208 E. A − B − E A − A × EB − B
Electron affinity ∝
Atomic size (b) Mulliken’s scale : Electronegativity χ(chi) will be the
Electron affinity ∝ Effective nuclear charge (zeff) average of the ionisation energy (IE) and the electron affinity
(EA) of an atom (both expressed in electron volts).
1
Electron affinity ∝ .
Screening effect IE + EA
χM =
Stability of half filled and completely filled orbitals of a subshell 2
is comparatively more and the addition of an extra electron to Paulings’s electronegativity χp is related to Mulliken’s
such an system is difficult and thus, the electron affinity value electronegativity χM as given below.
decreases. χP = 1.35 (χM)1/2 – 1.37
Mulliken’s values were about 2.8 times larger than the
TRAIN YOUR BRAIN Pauling’s values.
Q. Consider the elements N, P, O and S and arrange them in
order of increasing negative electron gain enthalpy.
Ans. Order of increasing negative electron gain enthalpy is N < P  KEY NOTE
< O < S. Application of Electronegativity
(vi) Electronegativity: Electronegativity is a measure of the (a) Nomenclature
tendency of an element to attract shared electrons towards itself Name of more electronegative element is written at
in a covalently bonded molecules. the end and ‘ide’ is suffixed to it. The name of less
The magnitude of electronegativity of an element depends upon its electronegative element is written before the name
ionisation potential & electron affinity. Higher ionisation potential of more electronegative element of the formula. For
& electron affinity values indicate higher electronegativity value. example-
When atomic size increases, the distance between nucleus and Correct formula Name
valence shell electrons increases, thus the force of attraction (a) I+ Cl– Iodine chloride
between the nucleus and the valence shell electrons decreases and (b) Partial Ionic Character in Covalent bonds
so, the electronegativity values also decrease.
Partial ionic characters are induced in covalent
With increase in nuclear charge force of attraction between compounds by the difference of electronegativities.
nucleus and the valence shell electrons increases and, hence, Hanney and Smith calculated percentage of ionic
electronegativity value increases character from the difference of electronegativity.
In higher oxidation state, the element has higher magnitude of Percentage of ionic character
positive charge. = 16(XA – XB) + 3.5(XA – XB)2 = 16Δ + 3.5Δ2
So, due to more positive charge on element, it has higher polarising = (0.16Δ + 0.035Δ2) × 100
power. XA is electronegativity of element A (Higher)
Thus, with increase in the oxidation state of element, its XB is electronegativity of element B (Lower)
electronegativity also increases. Δ = XA – X B

Classification of Elements ... 55


TRAIN YOUR BRAIN having completely filled shells have rather positive electron gain
Q. If electronegativity of x be 3.2 and that of y be 2.2, the enthalpy values). This results in high chemical reactivity at the
two extremes and the lowest in the centre.
percentage ionic character of xy is –
So, the maximum chemical reactivity at the extreme left (among
(a) 19.5 (b) 18.5 alkali metals) is represented by the loss of an electron leading to
(c) 9.5 (d) 29.5. the formation of cation and at the extreme right (among halogens)
Ans. (a) ENx – ENy = 3.2 – 2.2 = 1. represented by the gain of an electron forming an anion.
Δ=1 The loss and gain of electron can be related with the reducing and
[Δ = difference of electronegativity values between x and y] oxidizing behaviour of the elements respectively. Therefore, it can
also be directly related to the metallic and nonmetallic character
% ionic character = 16Δ + 3.5Δ2 = 19.5. of elements.
Thus, the metallic character of an element, that is highest at the
extremely left decreases and the nonmetallic character increases
while moving from left to right across the period.
The chemical reactivity of an element will be best understood by
its reactions with oxygen and halogens.
Nature of oxide
Elements on two extremes of a period easily combine with oxygen
to form oxides.
The normal oxide formed by the element on extreme left is the
most basic (e.g. Na2O), whereas that formed by the element on
The periodic trends of elements in the periodic table extreme right is the acidic (e.g. Cl2O7).
Oxides of elements in the centre are amphoteric (e.g. Al2O3,
Periodic Trends in Chemical Properties As2O3) or neutral (e.g., CO, NO, N2O).
1. Periodicity of Valence or Oxidation States: The valence of Amphoteric oxides behave as acidic with bases and as basic with
representative elements is usually (though not necessarily) equal acids, while neutral oxides have no acidic or basic properties.
to the number of electrons in the outermost orbitals and / or equal In general, metallic oxides (O2–), peroxides (O22 –) and super
to eight minus the number of outermost electrons. Frequently oxides (O21 –) are ionic solids.
used for valence. Oxides of IA and IIA dissolve in water forming basic solutions
where as other oxides do not dissolve in water.
Being highest electronegative element, fluorine is given oxidation Na2O + H2O → 2NaOH
state –1. In a group, basic nature of oxides increases or acidic nature
Since there are two fluorine atoms in this molecule, oxygen with decreases.
outer electronic configuration 2s22p4 shares two electrons with In a period the nature of the oxides varies from basic to acidic.
fluorine atoms and thereby exhibits oxidation state +2. Oxide Nature
On the other hand sodium with electronic configuration 3s1 loses Na2O Strongly basic
one electron to oxygen and is given oxidation state +1.
MgO Basic
There are many elements which exhibit variable valence. This is Al2O3 amphoteric
particularly characteristic of transition elements and actinoids. SiO2 Weakly acidic
2. Anomalous properties of Second period Elements: Some P4O10 Acidic
elements of certain groups of 2nd period resemble much in SO3 Acidic
properties with the elements of third period of next group i.e. Cl2O7 Strongly acidic
elements of second and third period are diagonally related in
properties. This phenomenon is known as diagonal relationship. Nature of Hydroxide
For example, the similarity between lithium (the first member of If electronegativity of E and O is larger than H and O in H2O then
group 1) and magnesium (the second element in group 2) is called EOH is basic.
a diagonal relationship. Diagonal relationship also exist between E—O—H + H2O → [EOH2]+ + OH–
other pairs of elements Be and Al, B and Si as shown in figure ; If electronegativities of E and O is less than that of H and O in
Diagonal Relationship H2O then EOH is acidic due to the formation of H3O+.
E—O—H + H2O → H3O+ + EO–
2nd period Li Be B C
3rd period Na Mg Al Si TRAIN YOUR BRAIN
Q. Arrange the following in decreasing basic nature LiOH,
PERIODIC TRENDS AND CHEMICAL
REACTIVITY NaOH, KOH, CsOH.
The ionization enthalpy of the extreme left element in a period Ans. The basic nature of hydroxides of elements of group 1st
is the least and the electron gain enthalpy of the element on the increases on descending the group with increase in size of
extreme right is the highest negative (except noble gases which cation as CsOH > RbOH > KOH > NaOH > LiOH.
56 Dropper NEET
Topicwise Questions

GENESIS OF PERIODIC TABLE, MODERN 11. Electronic configuration of the 4th transitional element is:
PERIODIC LAW & NOMENCLATURE OF (a) 1s2 2s2 2p6 3s2 3p6 4s2 (b) 1s2 2s2 2p6 3s2 3p6 3d4 4s2
ELEMENTS (c) 1s2 2s2 2p6 3s2 3p6 3d4 (d) 1s2 2s2 2p6 3s2 3p6 3d5 4s1
1. The third period of periodic table contains 12. Which of the following ions does not have the configuration
(a) 8 elements (b) 32 elements of Argon?
(c) 3 elements (d) 18 elements (a) Cl– (b) K+
(c) Ca2+ (d) I–
2. The element Californium belongs to the family of
(a) Actinoids series (b) Alkali metals 13. Which of the following species have the same number of
(b) Lanthanoid series (d) Alkali Earth metals electrons in its outermost as well as penultimate shell?
(a) Mg2+ (b) O2–
3. The tenth element in the periodic table resembles the element
with atomic number (c) F– (d) Ca2+
(a) 2 as well as 30 (b) 2 as well as 54 14. Name an element of p-block of the periodic table in which
(c) 8 as well as 18 (d) 8 only last electron goes to the s-orbital of valence shell instead of
p-orbital:
4. Without looking at the periodic table select from each of the
(a) As
following list, the elements belonging to the same group
(b) Ga
(a) Z = 12, 38, 4, 88 (b) Z = 9, 16, 3, 35
(c) No such element is there
(c) Z = 5, 11, 27, 19 (d) Z = 24, 47, 42, 55
(d) He
5. The plot of square root of frequency of X-rays emitted
against atomic number led to the suggestion of which law 15. Find the total no. of d-block elements in the following
(rule)? Atomic numbers?
(a) Mendeleev’s periodic law 48, 28, 70, 100, 55, 45, 34, 36
(b) Modern periodic law (a) 5 (b) 3
(c) Hund's rule (c) 6 (d) 2
(d) Newland's law 16. Which of the following electronic configuration is of
transition elements?
6. The discovery of which of the following group of element
gave death to the Newland law of octave? (a) 1s2 2s2 2p6 3s2 3p6 4s2
(a) Inert gas (b) Alkaline earth metal (b) 1s2 2s2 2p6 3s2 3p6 3d10 4s2 4p1
(c) Rare earth (d) Actinoid series (c) 1s2 2s2 2p6 3s2 3p6 3d10 4s2 4p6
(d) 1s2 2s2 2p6 3s2 3p6 3d2 4s2
7. The long form of periodic table was based on.
(a) Atomic number (b) Atomic mass 17. Which block of the periodic table contains the element with
configuration 1s2 2s2 2p6 3s2 3p6 3d10 4s1?
(c) Atomic volume (d) Effective nuclear charge
(a) s-block (b) p-block
8. What is the name of element with atomic number 105?
(c) d-block (d) f-block
(a) Kurchatovium (b) Dubnium
18. The electronic configuration of gadolinium (atomic number
(c) Nobelium (d) Holmium
64) is:
(a) [Xe] 4f8 5d9 6s2 (b) [Xe] 4f7 5d1 6s2
ELECTRONIC CONFIGURATION
(c) [Xe] 4f3 5d5 6s2 (d) [Xe] 4f6 5d2 6s2
9. The electronic configuration, element of group 18 can be
19. Which of the following will have total number of d-electrons
represented by.
equal to the difference in the number of total p and s
(a) ns2np5 (b) ns1 electrons?
2
(c) ns np 6 (d) (n-1)d8 ns2 (a) He (b) Ne
10. In transition element , the incoming electron occupies (n-1)d (c) Ar (d) Kr
subshell, in preference to.
20. The radii of the F, F­–, O and O2– are in the order:
(a) np-level (b) ns-level
(a) O2– > O > F– > F (b) F– > O2– > F > O
(c) (n-1) p-level (d) (n+1) s-level 2– –
(c) O > F > F > O (d) O2– > F– > O > F
Classification of Elements ... 57
21. Which of the following configuration represent atoms of 32. Which out of the following has the largest size?
element having the highest second ionisation energy? (a) Rb+ (b) Mg2+
(a) 1s2 2s2 2p4 (b) 1s2 2s2 2p6 (c) Li+ (d) Na+
(c) 1s2 2s2 2p6 3s1 (d) 1s2 2s2 2p6 3s2 33. Which of the following units are used frequently for atomic
22. Which of the following ions are paramagnetic in character? radii?
(a) Zn2+ (b) Cu+ (a) Meter (b) Picometers
(c) Ni2+ (d) Ag+ (c) Kilometers (d) Centimeters
23. Which of the following is not correct for isoelectronic ions? 34. Which of the following does not affect the ionisation
(a) They have the same number of electrons around their nuclei potential of the atom?
(b) Higher the atomic number, higher will be positive charge (a) Nuclear charge (b) Stable configuration
in a series of isoelectronic ions of same period
(c) Penetration effect (d) None of these
(c) Isoelectronic ions have same electric charge
35. The correct order of ionic radii for the ions S2–, Cl–, P3-, Ca2+
(d) An isoelectronic series may have both positively and
is
negatively charged ions
(a) Ca2+ > Cl– > S2– > P3– (b) S2– > P3– > Cl– >Ca2+
24. Which of the following is correct?
(c) P3– > S2– < Cl– > Ca2+ (d) Ca2+ < Cl– < S2– < P3–
(a) rionic ∝ Z (b) rionic ∝ Zeff
36. The correct order of electron affinity of halogens is:
1
(c) rionic
2
∝ (d) rionic ∝ Z eff (a) F > Cl > Br > I (b) I > Br > Cl > F
Zeff
(c) Cl > F > Br > I (d) Cl > F > I > Br
25. Pd has exceptional electronic configuration of 4d10 5s0. It
belong to 37. Which of the following pairs show diagonal relationship?
(a) 4th period, group 11 (b) 5th period, group 10 (a) Li, Mg (b) Be, Al
th
(c) 6 period, group 9 (d) 3rd period, group 16 (c) B, Si (d) All of these
26. Atomic number of Ag is 47. In the same group, the atomic 38. Which of the following element is expected to have highest
numbers of elements placed above and below Ag in long electron affinity?
form of periodic table will be (a) 1s2 2s2 2p6 3s2 3p5 (b) 1s2 2s2 2p3
(a) 29, 65 (b) 39, 79 (c) 1s2 2s2 2p4 ­ (d) 1s2 2s2 2p5
(c) 29, 79 (d) 39, 65 39. Which of the following oxide is amphoteric?
(a) Na2O (b) Al2O3
PERIODIC TRENDS AND PROPERTIES OF THE
ELEMENTS (c) SO3 (d) P2O5
40. The correct order of electronegativity of N, O, F and P is.
27. Which of the following isoelectronic ions has lowest
ionisation energy? (a) F > N > P > O (b) F > O > P > N
(a) K+ (b) Ca2+ (c) F > O > N > P (d) N > O > F > P
(c) Cl– (d) S2– 41. Which set of elements have strongest tendency to form
28. The elements which occupy the peaks of ionisation energy anions?
curve are (a) Na, Cl, Al (b) Cu, Ag, Au
(a) Na, K, Rb, Cs (b) Na, Mg, Cl, I (c) Be, F, N (d) F, Cl, Br
(c) Cl, Br, I, F (d) He, Ne, Ar, Kr 42. Outer most configuration of most electronegative element of
29. Which out of the following has the largest ionisation energy? the periodic table is.
(a) 11Na (b) 19K (a) 3s2 3p6 (b) 2s2 2p5 (c) 4s2 4p5 (d) 2s2 2p4
(c) 12Mg (d) 37Rb 43. In which of the following pairs, the ionisation energy of the
30. Which of the process requires largest energy among them? first species is less than that of the second:
(a) Al(g) → Al+(g) + e– (a) N, P (b) Be2+, Be
(b) Al2+(g) → Al3+(g) + e– (c) N, N– (d) S, P
(c) Al+(g) → Al2+(g) + e– 44. Isoelectronic ions are those which have:
(d) All the processes require same amount of energy (a) Same size
31. Which of the following process refers to the ionisation potential? (b) Same ionisation energy
(a) X(s) → X+(g) + e– (b) X(g) + 2e– → X2–(g) (c) Same electronic configuration
(c) X(g) → X+(g) + e– (d) X(g) + e– → X–(g) (d) Same nuclear charge
58 Dropper NEET
45. The correct order of the size of C, N, P, S following the 49. Which is mismatched regarding the position of the element
order: as given below?
(a) N < C < P < S (b) C < N < S < P (a) X(Z = 89) - f block, 6th period
(c) C < N < P < S (d) N < C < S < P (b) Y(Z = 100) - f block, 7th period
46. Which of the following has smallest size? (c) Z(Z = 115) - d block, 7th period
(a) Al3+ (b) Al2+
(d) Both (a) & (c)
+
(c) Al (d) Al
50. Correct order of Ist ionization potential among elements Be,
47. Which one of the following is an incorrect statement?
B, C, N, O is
(a) The ionisation potential of nitrogen is greater than that of
oxygen (a) B < Be < C < O < N
(b) The electron affinity of fluorine is greater than that of chlorine (b) B < Be < C < N < O
(c) The ionization potential of beryllium is greater than that (c) Be < B < C < N < O
of boron
(d) Be < B < C < O < N
(d) The electronegativity of fluorine is greater than that of
chlorine 51. An atom of an element has electronic configuration 2, 8, 1.
48. Identify the correct order of the size of the following Which of the following statement is correct?
(a) Ca2+ < K+ < Ar < Cl– < S2– (a) The valency of element is 7
(b) Ar < Ca2+ < K+ < Cl– < S2– (b) The element exists as a triatomic molecule
(c) Ca2+ < Ar < K+ < Cl– < S2– (c) The element is metalloid
(d) Ca2+ < K+ < Ar < S2– < Cl– (d) The element forms basic oxide

Learning Plus

1. The size of ionic species is correctly given in the order 6. Which of the following is not an actinoid?
(a) Na+ > Mg+2 > Cl+7 > Si4+ (a) Curium (Z = 96) (b) Californium (Z = 98)
(b) Na+ > Mg+2 > Si4+ > Cl+7 (c) Uranium (Z= 92) (d) Terbium (Z= 65)
(c) Cl+7 > Si+4 > Mg+2 > Na+ 7. The order of screening effect of electrons of s, p, d and f
(d) Cl+7 > Na+ > Mg+2 > Si+4 orbitals of a given shell of an atom on its outer shell electrons is:
2. A neutral atom (A) is converted to (A3+) by the following (a) s > p > d > f (b) f > d > p > s
E1 E2 E3 (c) p < d < s > f (d) f > p > s > d
process : A →
− e−
A + →− e−
A 2 + →
− e−
A 3+
The correct order of E1, E2 and E3 energies is 8. The first ionisation enthalpies of Na, Mg, Al and Si are in the
order:
(a) E1 < E2 < E3 (b) E1 > E2 > E3
(a) Na < Mg > Al < Si (b) Na > Mg > Al > Si
(c) E1 = E2 = E3 (d) E1 > E2 < E3
(c) Na < Mg < Al < Si (d) Na > Mg > Al < Si
3. Which of the following is a favourable factor for cation
formation? 9. The electronic configuration of gadolinium (Atomic number
64) is:
(a) High electronegativity (b) High electron affinity
(a) [Xe] 4f3 5d5 6s2 (b) [Xe] 4f7 5d2 6s1
(c) Low ionisation potential (d) Smaller atomic size
7 1
(c) [Xe] 4f 5d 6s 2 (d) [Xe] 4f8 5d6 6s2
4. Generally, the first ionisation energy increases along a
period. But there are some exceptions. The one which is not 10. The statement that is not correct for periodic classification of
an exception is elements is:
(a) Na and Mg (b) Be and B (a) The properties of elements are periodic function of their
(c) N and O (d) Mg and Al. atomic numbers
(b) Non-metallic elements are less in number than metallic
5. Consider the isoelectronic species, Na+, Mg2+, F– and O2–.
elements
The correct order of increasing length of their radii is:
– (c) For transition elements, the 3d-orbitals are filled with
(a) F < O2– < Mg2+ < Na+ electrons after 3p-orbital and before 4s-orbitals

(b) Mg2+ < Na+ < F < O2– (d) The first ionisation enthalpies of elements generally

(c) O2– < F < Na+ < Mg2+ increase with increase in atomic number as we go along a

(d) O2– < F < Mg2+ < Na+ period
Classification of Elements ... 59
11. Among halogens, the correct order of amount of energy 19. The elements with atomic numbers 35, 53 and 85 are all:
released in electron gain (electron gain enthalpy) is: (a) Noble gases (b) Halogens
(a) F > Cl > Br > I (b) F < Cl < Br < I (c) Heavy metal (d) Light metals
(c) F < Cl > Br > I (d) F < Cl < Br < I 20. Electronic configuration of four elements A, B, C, and D are
12. The period number in the long form of the periodic table is given below:
equal to: (a) 1s2 2s2 2p6 (b) 1s2 2s2 2p4
(a) Magnetic quantum number of any element of the period 2 2 6
(c) 1s 2s 2p 3s 1 (d) 1s2 2s2 2p5
(b) Atomic number of any element of the period Arrange them in increasing order of electron gain enthalpies.
(c) Maximum principal quantum number of any element of (a) A < C < B < D (b) A < B < C < D
the period (c) D > B < C < A (d) A > B < C > D
(d) Maximum azimuthal quantum number of any element of 21. Which one of the following represents a d-block element?
the period (a) [Rn] 6d10 7s2 7p2
13. The elements in which electrons are progressively filled in (b) [Xe] 4f1 5d1 6s2
4f-orbital are called:
(c) [Xe] 4f14 5d1 6s2
(a) Actinoids (b) Transition elements
(d) [Xe] 5d1 6s2
(c) Lanthanoids (d) Halogens
22. The electronic configuration having maximum difference in
14. Which of the following is the correct order of size of the first and second ionization energies is
given species:
(a) 1s2 2s2 2p6 3s2 (b) 1s2 2s2 2p6 3s2 3p1
(a) I > I– > I+ (b) I+ > I– > I (c) 1s2 2s2 2p6 3s2 3p2 (d) 1s2 2s2 2p6 3s1
+
(c) I > I > I (d) I– > I > I+
23. The successive ionization energies for element X is given
15. The element with atomic number 57 belongs to: below
(a) s - block (b) p - block IE1: 250 kJ mol–1
(c) d - block (d) f - block IE2: 820 kJ mol–1
16. The last element of the p-block in 6th period is represented IE3: 1100 kJ mol–1
by the outermost electronic configuration: IE4: 1400 kJ mol–1
(a) 7s2 7p6 (b) 5f14 6d10 7s2 7p0 Find out the number of valence electrons for the element X.
(c) 4f14 5d10 6s2 6p6 (d) 4f14 5d10 6s2 6p4 (a) 3 (b) 4 (c) 2 (d) 1
17. Which of the elements whose atomic numbers are given 24. What is the value of electron gain enthalpy of Na+ if IE1 of
below, cannot be accommodated in the present set up of the Na = 5.1 eV?
long form of the periodic table? (a) +2.55 eV (b) +10.2 eV
(a) 107 (b) 118 (c) 126 (d) 102 (c) –5.1 eV (d) –10.2 eV
18. The electronic configuration of the element which is just 25. An element X with Z = 112 has been recently discovered. To
above the element with atomic number 43 in the same group which group and period will it belong, respectively-
is: (a) 12, 7 (b) 10, 6
(a) 1s2 2s2 2p6 3s2 3p6 3d5 4s2 (c) 8, 4 (d) 6, 6
(b) 1s2 2s2 2p6 3s2 3p6 3d5 4s3 4p6 26. Which of the following has lowest Ist ionisation energy?
(c) 1s2 2s2 2p6 3s2 3p6 3d6 4s2 (a) Li (b) C
(d) 1s2 2s2 2p6 3s2 3p6 3d7 4s2 (c) O (d) F

Multiconcept MCQs
1. Ionisation energies of element A are given below in kJ/mol 2. In 4th period of periodic table, how many elements have one
IE1 = 120 , IE2 = 430 , IE3 = 540 or more 4d electrons?
If A reacts with different elements, which compound of A is (a) 2 (b) 0 (c) 13 (d) 6
not possible? 3. Few reactions are given below:
1. O( g ) + e  → O(−g ) , ∆1H 2. F( g ) + e − → F(−g ) , ∆ 2 H

(a) AF (b) A2O
(c) A3N (d) A3N2 3. Cl( g ) + e 
→ Cl(−g ) , ∆ 3 H 4. O(−g ) + e − 

→ O(2g−) , ∆ 4 H

60 Dropper NEET

Which of the following statement(s) is/are correct with (c) Amongst isoelectronic species, smaller the positive
respect to the reactions mentioned above? charge on the cation, smaller is the ionic radius
(a) ∆3H is more –ve than ∆1H and ∆2H (d) Amongst isoelectronic species, greater the negative
(b) ∆1H is less negative than ∆2H charge on the anion, larger is the ionic radius
(c) ∆1H , ∆2H and ∆3H are –ve whereas ∆4H is +ve 11. The diagram below is a part of the skeleton of the periodic
(d) All are correct tables in which elements are indicated by letter which are not
their usual symbols.
4. Which of the following statement is not correct?
(a) Ionisation enthalpy for a neutral atom is always positive
(b) 2nd I.E. of monovalent cation is equal to the first I.E. of
bivalent cation.
(c) Ionisation enthalpy of an ion is always positive
The correct option is :
(d) Both (a) and (b)
(a) 'D' has maximum electron affinity
5. Which of the following is/are incorrect?
(b) 'B' exist in nature in a liquid state
(a) I.P. of O(g) is less than I.P of O( g )

(b) I.P. of Ne(g) is greater than I.P. of Ne+(g) (c) 'A' is a 5th period element in a periodic table
(c) I.P. of N(g) is greater than I.P. of N ( g )
+ (d) (b) & (c)
(d) All are incorrect 12. From the following given electronic configuration, identify
6. Which of the following reactions will involve release of energy? the correct order of electron affinity:
(I) [He] 2s2 2p5 (II) [He] 2s2 2p3
→ S(2g−)
(a) S( g ) 

(III) [Ne] 3s2 3p5 (IV) [Ne] 3s2 3p3


(b) N ( g ) 
→N −
(g)
(a) I > II > III > IV (b) III > I > IV > II
(c) O + e– → O–
(g)   (g)      (s) (c) I < II < III < IV (d) II > III > IV > I
2+
→ Al3( g+)
(d) Al( g )  13. Few elements are matched with their successive ionisation
energies. Identify the elements.
7. Out of N, O, Ne, Na and Na+, which of these species will have
the maximum and minimum ionization energy respectively: Element IE1 (kJ/mol) IE2 (kJ/mol)
(a) Na+ , O (b) Na+ , Na X 2372 5251
(c) N , Ne (d) Ne , N Y 520 7297
8. Which of the following is/are correct with respect to Z 900 1758
reactions given below?
X Y Z
→ M (+g ) + e − ; ∆H = 100 eV
M ( g )  (a) A noble gas Alkali metal Alkaline earth metal
→ M (2+g ) + e − ; ∆H = 250 eV
M + ( g )  (b) Alkali metal A noble gas Alkaline earth metal
(c) Alkaline earth Alkali metal A noble gas
(a) ∆H1 of M(g) is 100 eV (b) ∆H1 of M (+g )is 150 eV
metal
(c) ∆H2 of M(g) is 250 eV (d) All are correct (d) Alkali metal Alkaline earth A noble gas
metal
9. Which of the following statement is incorrect?
(a) The first ionization potential of nitrogen is greater than 14. In the given graph, a periodic property (R) is plotted against
that of oxygen atomic numbers (Z) of the elements. Which property is
(b) The electron affinity of fluorine is greater than that of shown in the graph and how is it correlated with reactivity of
chlorine the elements?
(c) The first ionization potential of Mg is greater than
aluminium
(d) The electronegativity of fluorine is greater than that of
chlorine
10. Identify the wrong statement in the following.
(a) Atomic radius of the elements increases as one moves
down the first group of the periodic table
(b) Atomic radius of the elements decreases as one moves
across from left to right in the 2nd period of the periodic
table
Classification of Elements ... 61
(a) Ionisation enthalpy in a group decrease reactivity (c) The first I.E. of elements with atomic number 1 to 5
decreases from a → e. (d) Successive I.E. for transition elements with four electrons
(b) Ionisation enthalpy in a group decreases reactivity in d-subshell.
increases from a → e.
17. In a periodic table, the basic character of oxides
(c) Atomic radius in a group decrease reactivity decreases
from a→ e. (a) Increases from left to right and decreases from top to
(d) Metallic character in a group decrease reactivity increases bottom
from a → e. (b) Decreases from right to left and increases from top to
bottom
15. Match the entries of List I with appropriate entries of List II
(c) Decreases from left to right and increases from top to
and select the correct answer using the codes given below the
lists: bottom
(d) Decreases from left to right and increases from bottom to
List I List II
top
P. Rutherfordium (At. No. = 1. Period number = 7
18. Identify the incorrect statement.
104)
(a) The first ionisation potential of Al is less than the first
Q. Roentgenium (At. No. = 2. Group number = 4
ionisation potential of Mg.
111)
(b) The second ionisation potential of Mg is greater than the
R. Thorium (At. No. = 90) 3. d-block elements
second ionisation potential of Na.
S. Neptunium (At. No. = 93) 4. f-block elements
(c) The first ionisation potential of Na is less than the first
P Q R S ionisation potential of Mg.
(a) 3 2,4 2,3 1,2,3 (d) The third ionisation potential of Mg is greater than that ofAl.
(b) 1,2,3 1,3 1,4 1,4 19. Elements X, Y and Z have atomic numbers 19, 37 and 55
(c) 1,2 2,1 4,3 3,1 respectively. Which of the following statements is true about
(d) 3,1 1,3 2,4 4,1,2 them?
16. Five ionization energy values in kJ/mol are listed below: (a) Their ionization potential would increase with increasing
E1 = 870, E2 = 830, E3 = 1010, E4 = 1290, E5 = 376. These atomic number.
are (b) Y would have an ionization potential between those of X
(a) Successive ionization energies for the element with and Z.
atomic number 5
(c) Z would have the highest ionization potential.
(b) The first I.E. of successive elements in group 15, 16, 17,
18 and 1 respectively (d) Y would have the highest ionization potential.

NEET Past 10 Years Questions


1. From the following pairs of ions which one is not an iso- 3. For the second period elements the correct increasing order
electronic pair? (2021) of first ionisation enthalpy is: (2019)
(a) Na+, Mg2+ (a) Li < Be < B < C < N < C < F < Ne
(b) Mn2+, Fe3+
(b) Li < B < Be < C < O < N < F < Ne
(c) Fe2+, Mn2+
(c) Li < B < Be < C < N < O < F < Ne
(d) O2–, F–
2. Identify the incorrect match (2020)
(d) Li < Be < B < C < O < N < F < Ne

Name IUPAC Official Name 4. The element Z = 114 has been discovered recently. It will
(A) Unnilunium (i) Mendelevium belong to which of the following family group and electronic
configuration? (2017-Delhi)
(B) Unniltrium (ii) Lawrencium
(C) Unnilhexium (iii) Seaborgium (a) Nitrogen family, [Rn] 5f146d107s27p6

(D) Unununnium (iv) Darmstadtium (b) Halogen family, [Rn] 5f146d107s27p5


(a) (B), (ii) (b) (C), (iii) (c) Carbon family, [Rn] 5f146d107s27p2
(c) (D), (iv) (d) (A), (i) (d) Oxygen family, [Rn] 5f146d107s27p4
62 Dropper NEET
5. In which of the following options the order of arrangement does 8. The species Ar, K+ and Ca2+ contain the same number of
not agree with the variation of property indicated against it? electrons. In which order do their radii increase? (2015)
 (2016 - I) 2+ +
(a) Ca < Ar < K
(a) Li < Na < K < Rb (increasing metallic radius)
(b) Ca2+ < K+ < Ar
(b) Al3+ < Mg2+ < Na+ < F─ (increasing ionic size)
(c) K+ < Ar < Ca2+
(c) B < C < N < O (increasing first ionization enthalpy)
(d) Ar < K+ < Ca2+
(d) I < Br < Cl < F (increasing electron gain enthalpy)
6. The formation of the oxide ion, O2– (g) from oxygen atom 9. Be2+ is isoelectronic with which of the following ions?(2014)
requires first and exothermic and then an endothermic step as (a) Li+ (b) Na+
shown below: (c) Mg2+ (d) H+
O (g) + e– → O– (g) ; ΔfHo = –141 kJ mol–1
10. Which of the following orders of ionic radii is correctly
O– (g) + e– → O2– (g) ; ΔfHo = +780 kJ mol–1 represented? (2014)
Thus, process of formation of O2– in gas phase is unfavourable + –
(a) Na > F > O 2–
even though O2– is isoelectronic with neon. It is due to the
(b) O2– > F– > Na+
fact that, (2015 Re)

(a) O ion has comparatively smaller size than oxygen atom (c) Al3+ > Mg2+ > N3–
(b) Oxygen is more electronegative (d) H– > H+ > H
(c) Addition of electron in oxygen results in larger size of 11. Identify the wrong statement in the following: (2012 Pre)
the ion (a) Atomic radius of the elements decreases as one moves
(d) Electron repulsion outweighs the stability gained by across from left to right in the 2nd period of the periodic
achieving noble gas configuration table
7. The number of d-electrons in Fe2+ (Z = 26) is not equal to the (b) Amongst isoelectronic species, smaller the positive charge
number of electrons in which one of the following? (2015) on the carbon, smaller is the ionic radius
(a) p-electrons in Cl (Z = 17) (c) Amongst isoelectronic species, greater the negative
(b) d-electrons in Fe (Z = 26) charge on the anion, larger is the ionic radius
(c) p-electrons in Ne (Z = 10) (d) Atomic radius of the elements increases as one moves
(d) s-electrons in Mg (Z = 12) down the first group of the periodic table.

Classification of Elements ... 63


ANSWER KEY

Topicwise Questions

1. (a) 2. (a) 3. (b) 4. (a) 5. (b) 6. (a) 7. (a) 8. (b) 9. (c) 10. (a)
11. (d) 12. (d) 13. (d) 14. (d) 15. (b) 16. (d) 17. (c) 18. (b) 19. (d) 20. (d)
21. (c) 22. (c) 23. (c) 24. (c) 25. (b) 26. (c) 27. (d) 28. (d) 29. (c) 30. (b)
31. (c) 32. (a) 33. (b) 34. (d) 35. (d) 36. (c) 37. (d) 38. (a) 39. (b) 40. (c)
41. (d) 42. (b) 43. (d) 44. (c) 45. (d) 46. (a) 47. (b) 48. (a) 49. (d) 50. (a)
51. (d)

Learning Plus

1. (b) 2. (a) 3. (c) 4. (a) 5. (b) 6. (d) 7. (a) 8. (a) 9. (c) 10. (c)
11. (c) 12. (c) 13. (c) 14. (d) 15. (c) 16. (c) 17. (c) 18. (a) 19. (b) 20. (a)
21. (d) 22. (d) 23. (d) 24. (c) 25. (a) 26. (a)

Multiconcept MCQs

1. (a) 2. (b) 3. (d) 4. (d) 5. (d) 6. (c) 7. (b) 8. (d) 9. (b) 10. (c)
11. (a) 12. (b) 13. (a) 14. (b) 15. (b) 16. (b) 17. (c) 18. (b) 19. (b)

NEET Past 10 Years Questions

1. (c) 2. (c) 3. (b) 4. (c) 5. (c), (d) 6. (d) 7. (a) 8. (b) 9. (a) 10. (b)
11. (b)

64 Dropper NEET
3 Classification of Elements and
Periodicity in Properties

Topicwise Questions
1. (a) According to periodic table. 16. (d) Electronic filling configurations is of transition elements.
2. (a) According to modern periodic table. 17. (c) It is copper, a d-block transition element.
3. (b) Element with atomic number (2) and (54) (Xe) are also
noble gas. The tenth atom is Neon (Ne). It is a noble gas 18. (b) Electronic filling configuration.
element with atomic number 10. 19. (d) The first inert gas which contains d electrons is in 4th
4. (a) They have same valence shell configuration or their atomic period i.e., Kr with atomic no. = 36
no. should differ by magic number. 2, 8, 8, 18, 18, 32 . E.C. of Kr = 1s2 2s2 2p6 3s2 3p6 4s2 3d10 4p6
5. (b) Moseley gave the modern periodic law. He showed Total number of d-electrons = 10
that atomic number is more fundamental property of an
Total number of p-electrons = 6 + 6 + 6 = 18
element than its atomic mass. He found that the square
root of the frequency of a line (of a X-Ray spectrum) is Total number of s-electrons = 2 + 2 + 2 + 2 = 8
related to the atomic number (Z) of target material; as Difference in p and s electron is = 18 – 8 = 10
ν= a(z − b) 20. (d) Radii are in the following order
O2– > F– > O > F
6. (a) With the discovery of inert gases (group zero in
Mendeleev's periodic table), the law of octaves lost its O2– and F– are isoelectronic and O,F belong to the same
original significance since it was now the 9th element period.
which had properties similar to the 1st one, which is not 21. (c) Electronic configuration.
according to the law. 22. (c) Zn2+ : [Ar]18 3d10 ; Ag+ : [Kr]36 4d10
7. (a) The long form of periodic table is based on atomic
Cu+ : [Ar]18 3d10 ; Ni2+ : [Ar]18 3d8
number of element.
As such Ni2+ has two unpaired electrons and is paramagnetic.
8. (b) In IUPAC nomenclature, it is known as un-nil-pentium.
23. (c) Isoelectronic species have same number of electrons not
Db (105) = [Rn]86 5f 14 6d3 7s2 same isoelectric charge. For example N3–, O2–, F– are
9. (c) Periodic table (ns2 np6) isoelectronic but have different charge.
10. (a) (n – 1)d < np 24. (c) On increasing effective nuclear charge ionic size
11. (d) 4th transition element has atomic no. 24 decreases.
12. (d) I– has electronic configuration of Xe. 1
13. (d) Ca2+ has electronic arrangement (2, 8, 8) rionic ∝
Zeff
14. (d) Helium is a p-block element with configuration of s
block. 25. (b) Pd = 4d10, 5s0 member of 4d series i.e., 5th period and 10th
15. (b) 28 → d-block group.
48 → d-block
26. (c) Silver belongs to 5th period. So, the atomic number of
70 → f-block elements placed above and below will be 47 – 18 = 29
100 → f-block and 47 + 32 = 79 respectively.
55 → s-block 27. (d) In isoelectronic species, greater the negative charge,
45 → d-block lower the ionisation energy.
34 → p-block 28. (d) Peaks in the IE curve are occupied by the noble gases
because IE of noble gases is highest in the respective
36 → p-block periods.
29. (c) Among Rb, K and Na, the IE1 decrease as Rb < K < Na 42. (b) The most electronegative element is Fluorine.
but IE1 of Na < Mg. 43. (d) P has higher IE due to stable half filled configuration.
30. (b) It refers to IE3 which has a higher value.
44. (c) Same number of electrons and hence same electronic
31. (c) Based on definition of ionisation energy.
configuration.
32. (a) Down the group; size increase with decrease in nuclear
charge. 45. (d) Along a period size decrease while down the group size
increases.
33. (b) Unit of atomic radii is picometer.
Grp 13 Grp 14 Grp 15 Grp 16
34. (d) None of these.
B C N O
35. (d) (15)P3– , (16)S2– , (17)Cl–, belong to 3rd period whereas
20 Ca2+ belong to 4th period. Al Si P S
These are isoelectronic ions and therefore size depends on 46. (a) Greater the positive charge on the ion, smaller is its size.
nuclear charge. More is the nuclear charge, smaller is the size.
47. (b) The electron affinity of chlorine is greater than flourine.
36. (c) Since the atomic size increases down the group, electron
affinity generally decreases (At < I < Br < F < Cl) 48. (a) Among isoelectronic ions ionic radii of anions is more
37. (d) Diagonal Relationship than that of cations. Further size of the anion increases
1 2 13 14 15 with increase in negative charge and size of the cation
Period 2 Li Be B C N decreases with increase in positive charge.
Period 3 Na Mg Al Si P 49. (d)  Z= 89= Ac= d − block 7 th period 
 th 
38. (a) It represents halogen Cl. The E.A. of Cl is highest among  Z= 115= Uup= p − block 7 period 
halogens.
50. (a) Left to right in period I.E increases and half filled have
39. (b) Na2O is basic, SO3 and P2O5 are acidic but Al2O3 can
more I.E.
react both with acids and bases.
40. (c) The correct order of electronegativities of N, O, F and P 1st I.E [B < Be (more I.E. due to half filled) < C < O < N
is F > O > N > P. (more I.E. than 'O' due to half filled)]
In a period, electronegativity increases from left to right. 51. (d) Electronic configuration indicates that 1 e– is present in
Hence, N < O < F. outermost shell.
In a group, electronegativity decreases down the group.
Hence, N > P. It will easily lose electrons
41. (d) Halogens have a high value of electron affinity also ∴ It is metal and form basic oxide
by gaining one electron, it acquire stable noble gas 1
configuration.
2Na + O 2 → Na 2 O
2

Learning Plus
1. (b) For isoelectronic more is the positive charge smaller will 7. (a) For the same shell, screening effect decreases in the order
be the size s>p>d>f
∴ Na+ > Mg2+ > Si4+ > Cl+7 8. (a) The E.C are as follows:
2. (a) For a particular atom the successive ionisation potential Na: [Ne] 3s1, Mg: [Ne]3s2, Al: [Ne]3s23p1, Si: [Ne]3s23p2
always increases. Thus, E1 < E2 < E3.
The I.E of Mg will be larger than that of Na due to fully
3. (c) The tendency to lose electron is higher with elements
filled configuration. The I.E of Al will be smaller than
having lower ionisation potential.
that of Mg due to 1e– extra than the stable configuration
4. (a) Na and Mg is not an exception because there is no half- but smaller than Si due to increase in EAN of Si.
filled or completely filled orbital in them.
9. (c) The electronic configuration of Gd (Z = 64) is [Xe] 4f 7
5. (b) In case of isoelectronic species, more the number of protons
smaller the size. 5d1 6s2.

6. (d) Elements with atomic number, Z = 90 to 103 are called 10. (c) 3d orbital is filled when 4s orbital gets completely filled. But
actinoids. Thus, terbium (Z = 65) is not an actinoid. in case of transition elements (or any elements), the order of
Terbium belongs to lanthanoids. filling of electrons in various orbital is 3p < 4s < 3d.
Classification of Elements ... 29
11. (c) As we move from Cl to I, the electron gain enthalpy (i.e., 19. (b) Elements with atomic numbers 35 (36 – 1), 53 (54 – 1),
energy released in electron gain) become less and less and 85 (86 – 1), lie in a group before noble gases, i.e.,
negative due to a corresponding increase in the atomic halogens (group 17) elements because each period ends
size. However, the electron gain enthalpy of F is less with a noble gas. The atomic number of noble gases (i.e.,
negative than that of Cl due to its small size.
group 18 elements) are 2, 10, 18, 36, 54 and 86.
12. (c) As each period starts with the filling of electrons in a new
Thus, the elements with atomic number 35, 53 and 85 are
principal quantum number. Thus, the period number in
all belongs to halogens.
the long form of the periodic table refers to the maximum
principal quantum number of any element in the period. 20. (a) Electronic configuration of elements indicate that A is a
Period number = maximum n of any element (where, n = noble gas (i.e., Ne), B is oxygen (group 16), C is sodium
principal quantum number). metal (group 1) and D is fluorine (group 17).
(i) Noble gases have no tendency to gain electrons since
13. (c) The elements in which electrons are progressively filled
in 4f-orbital are called lanthanoids. Lanthanoids consist all their orbitals are completely filled. Thus, element
of elements from Z = 58 (cerium) to 71 (lutetium). A has the least electron gain enthalpy.
14. (d) Cation is formed after the loss of electron from outer (ii) Since, element D has one electron less and element B
shell and anion is formed after the gain of electron to the has two electrons less than the corresponding noble
neutral atom. Hence, cation has smaller size but anion gas configuration, hence, element D has the highest
has bigger size than its neutral atom. Thus, I– > I > I+. electron, gain enthalpy followed by element B
15. (c) The element with atomic number 57 belongs to d-block (iii) Since, element C has one electron in the s-orbital
element as the last electron enters the 5d-orbital against and hence needs one more electron to complete it,
the Aufbau principle. This anomalous behaviour can be therefore, electron gain enthalpy of C is less than that
explained on the basis of greater stability of the xenon
of element B. Combining all the facts given above
(inert gas) core.
the electron gain enthalpies of the four elements
Thus, the outer electronic configuration of La(Z = 57) is
increase in the order A < C < B < D.
5d1 6s2 rather than the expected 4f1 6s2.
16. (c) 6th period starts with the filling of 6s-orbital and ends when 21. (d) Last electrons enters in d-subshell
6 p-orbitals are completely filled because each period ∴ It is d-block elements.
starts with the filling of electrons in a new principal energy
22. (d) After removing 1 e– it will get stable noble gas configuration.
shell.
17. (c) The long form of the periodic table contain element with 23. (d) Difference between IE1 and IE2 is high then the number
atomic number 1 to 118. of valence electron in the element is one.
18. (a) The fifth period begins with Rb (Z = 37) and ends at Xe (Z 24. (c) Electron gain enthalpy is negative of I.E. i.e., –5.1 eV
= 54). Thus, the element with Z = 43 lies in the 5th period.
25. (a) At No. 112 will have configuration (Rn)86 5f14 6d10
Since, the 4th period has 18 elements, therefore, the atomic
7s2. It’s a d–block element, belongs to 7th period & 12th
number of the element which lies immediately above the
element with atomic number 43 is 43 – 18 = 25 . group.
Now, the electronic configuration of element with Z = 25 is 26. (a) Order of Ist IP
1s2 2s2 2p6 3s2 3p6 3d5 4s2 F > O > C > Li

Multiconcept MCQs
1. (a) The maximum jump is between Ist and 2nd I.E., therefore For option 3, adding e– in O– will certainly require energy,
it will exhibit oxidation state of +1.
so electron gain enthalpy is +ve.
2. (b) 4th period comprises of 4s, 3d and 4p.
4d elements will lie in 5th period 4. (d) Do it yourself

3. (d) For option 1, Cl has most negative electron gain enthalpy. 5. (d) IE3 is always greater than IE2 which in turn is greater
For option 2, F has greater negative electron gain enthalpy than IE1.
than O.
30 Dropper NEET
6. (c) For option a, ∆Heg is always +ve. 14. (b) I.E. in a group decreases and reactivity increases.
For option b, N has half filled configuration, so ∆egH is +ve 15. (b) (P → 1, 2, 3)
For option d, IE will be positive Rf (Z = 104) : [Rn] 5f146d27s2
In option c, lattice energy is released
Period no. 7, d-block element, group no. 4.
7. (b) Na+ and Ne are isoelectronic but Na+ has more protons.
So, its I.E. is highest. (Q →1, 3)
Na is alkali metal with a very low I.E. Rg (Z = 111) : [Rn] 5f 14 6d107s1
8. (d) M ( g ) 
→ M (+g ) + e − ; ∆H1 = 100ev Period no. 7, group no. 11, d-block element.
→ M (2g+) + e − ; ∆H = 150ev
M +(g )  (R →1, 4)
Th (Z = 90) : [Rn] 5f0 6d27s2
→ M (2g+) + e − ; ∆H2 = 250ev
M (g ) 
Period no. 7, group no. 3 , f-block element.
9. (b) Electron affinity of F is less than chlorine because of smaller (S →1, 4)
size more will be the repulsion towards new electron.
10. (c) In isoelectronic species i.e., same number of electrons. Np (Z = 93) : [Rn] 5f4 6d17s2
more the positive charge; smaller will be the size  Period no. 7, group no 3, f-block element.
 
more the negative charge; larger will be the size  16. (b) Do it yourself.
17. (c) As the electronegativity of element increases, acidic
11. (a) character of oxides increases. So, in a group, basic nature
increases on moving down and decreases along a period.
18. (b) IE2 of Na is higher than that of Mg because in case of Na,
12. (b) I = F III = Cl the second electron has to be removed from the noble
II = N IV = P
gas core while in case of Mg, removal of second electron
order Cl > F > P > N
gives a noble gas core.
13. (a) X has highest IE1 and IE2 hence, it is a noble gas. Y has
low IE1, but very high IE2 hence, it is an alkali metal. 19. (b) Elements X, Y and Z with atomic numbers 19, 37, 55 lie in
Z has low IE1 than IE2 and IE2 is even lower than IE2 of group 1 (alkali metals). Within a group, IE decreases from
alkali metal hence, it is an alkaline earth metal. top to bottom. Therefore, IE of Y could be between X and Z.

NEET Past 10 Years Questions


1. (c) Total no. of e– Number of s electron in Mg (Z = 12) = 6
3d64s2, Fe+2 3d6 Number of p electron in Ne = 6
26Fe → → 24
25Mn → 3d54s2, Mn+2 → 3d5 23 8. (b) 
In case of isoelectronic species, radius decreases with
2. (c) Unununnium is the element that has Atomic number = 111 increase in nuclear charge.
IUPAC official name of Unununium: Roentgenium 9. (a) Be2+ = 2e–
Thus option (c) is correct.
Li+ = 2e–
3. (b) 
‘Be’ and ‘N’ have comparatively more stable valence sub-
Isoelectronic species means ions with same number of
shell than ‘B’ and ‘O’.
electron.
Generally Ionisation energies increases across a period.
Thus, correct increasing order of first ionisation enthalpy is: 10. (b) 
Cations lose electrons and are smaller in size than the parent
Li < B < Be < C < O < N < F < Ne atom, whereas anions add electrons and are larger in size
than the parent atom.
4. (c) Carbon family: [Rn] 5f146d107s27p2
5. (c, d) Increasing first ionization enthalpy will be B < C < O < N. Hence, the order is H– > H > H+.
Electron gain enthalpy: I < Br < F < Cl For isoelectronic species, the ionic radii decreases with
6. (d) 
There is a lot of repulsion when similar charges approach increase in atomic number i.e., nuclear charge.
each other as O–(g) and e– are both negatively charged. To Hence, the correct orders are:
add an electron under such situation, the force of repulsion O2– > F– > Na+ and N3– > Mg2+ > Al3+
is to be overcome by applying external energy
11. (b) 
Among isoelectronic species the ion with the maximum
7. (a) Number of d electrons in Fe2+ (26) = 6
positive charge will have the smallest radius.
Number of p electron in Cl (Z = 17) = 11
Classification of Elements ... 31
4 Chemical Bonding and
Molecular Structure

Past Years NEET Trend

5
No. of MCQs

0
2021 2020 2019 2018 2017 2016 2015 2014 2013 2012

Investigation Report
TARGET EXAM PREDICTED NO. OF MCQs CRITICAL CONCEPTS
• Molecular orbital theory
NEET 4-5 • Dipole moment, VSEPR Theory

Perfect Practice Plan


Topicwise Questions Learning Plus Multiconcept MCQs NEET Past 10 Years Total MCQs
Questions
193 36 24 41 294
INTRODUCTION 1. The incomplete octet of the central atom: In some compounds,
An attractive force that acts between two or more particles the number of electrons surrounding the central atom is less than
eight. This is especially the case with elements having less than
(atoms, ions or molecules) to hold them together, is known as a
four valence electrons. Examples are LiCl, BeH2 and BCl3.
chemical bond.
Cl
It is a union of two or more atoms to aquire stable inert gas Li:Cl H:Be:H Cl:B:Cl

:
configuration ns2np6. Li, Be and B have 1, 2 and 3 valence electrons only. Some other
In a Chemical Bond both attractive and repulsive forces exist in such compounds are AlCl3 and BF3.
equilibrium.
2. Odd-electron molecules: In molecules with an odd number
Atoms are less stable and more energetic hence they form of electrons like nitric oxide. NO and nitrogen dioxide, NO2, the
molecules by loosing some energy by participating in Chemical octet rule is not satisfied for all the atoms
Bond. .+
N. =O O = N – O–:

:
: :

: :

: :
H + H → H2 + 434.72kJ
Cl + Cl → Cl2 + 239.1kJ 3. The expanded octet/ super octet / hypervalent compound:
In some compounds, the number of electrons surrounding the
Formation of bond is accompanied by decrease in potential energy.
central atom is more than eight due to availability of 3d orbitals
Therefore, Chemical Bond formation is always Exothermic.
(incomplete octet of the central atom).
To explain nature of chemical bond different theories are Some of the examples of such compounds are: PF5 SF6, H2SO4
given, at first KOSSEL & LEWIS proposed the theory. and a number of coordination compounds.
Kossel & Lewis approch chemical bonding: In early days,
the ability of various elements to combine with one another was
expressed in terms of their valency. The concept of valency
was not based on any logical understanding. W. Kossel and G.
N. Lewis were the first to provide some logical explanation of
valency which is based on the inertness of noble gases.
This view later on, came to be known as Octet rule.

OCTET RULE FORMAL CHARGE


(Tendency to acquire noble gas configuration) Formal charge is a factor based on a pure covalent bond formed
It has been observed that atoms of noble gases have little or no by the sharing of electron pairs equally by neighbouring atoms.
tendency to combine with each other or with atoms of other Formal charge may be regarded as the charge that an atom in a
elements. molecule would have if all the atoms had the same electronegativity.
It may or may not approximate the real ionic charge. In case of
It means that these atoms must have a stable electronic
a polyatomic ions, the net charge is possessed by the real ion as
configuration.
a whole and not by a particular atom. It is, however, feasible to
These elements (noble gases) have 8 electrons (ns2 np6) except assign a formal charge on an atom in a polyatomic molecule or
helium which has 2 electrons (1s2) in their outer most shell. ion.
The formal charge of an atom in a polyatomic molecule (or) ion
Element Outer most shell Configuration
may be defined as the difference between the number of valence
Ne 2s22p6 electrons of that atom in an isolated or free state and the number
electrons assigned to that atom in the lewis structure.
Ar 3s2sp6
Qf = [NA – NM] = [NA – NLP – 1/2NBP]
Kr 4s24p6 Qf = Formal charge
Xe 5s25p6 NA = number of electrons in the valence shell in the free atom
NM =  number of electrons belonging to the atom in the
Rn 6s26p6 molecule
NLP = number of electrons in unshared pairs, i.e. number of
It is therefore concluded that ns2np6 configuration in the outer electrons in lone pairs
energy level constitutes a structure of maximum stability or
NBP = number of electrons in bond pairs, respectively.
minimum energy.
Example: The Lewis dot formula of PH3 is
“Tendency of atoms to have eight electrons in their outermost shell
is known as Lewis octet rule”.To achieve inert gas configuration,
atoms lose, gain or share electrons.

Limitations of the Octet Rule Formal Charge of P


It applies mainly to the second period elements of the periodic Qf = [NA – NM] = [NA – NLP – 1/2 NBP]
table. There are three types of exceptions to the octet rule: {5-2-1/2(6)} = (5-5) =0
66 Dropper NEET
Formal Charge of H Electronegativity of Changes occuring Nature of the
Q1 = [NA – NM] = [NA – NLP – 1/2 NBP] Elements in the bond formed
valence electrons
= (1-0-2/2)=0
Low High Transfer of e– Ionic bond
● Formal charges on oxygen atoms of ozone
High High Sharing of e– Covalent
bond
3 Low Low Sea of e– molecular Metallic bond
● Formal charge of oxygen (1) = +1 orbital bond
● Formal charge of side oxygen atom(2) = 0
Bond Bond Energy per mole
● Formal charge of side oxygen atom (3) = –1
Ionic bond –––––––––– 700 – 4000 kJ
The formal charge on an atom may or may not be same. It changes
Covalent and coordinate bond –––––––––– 200 – 500 kJ
with the structural environment of the atom in the molecule. In
resonating structures electronic environment changes. Hence the Hydrogen bonding –––––––––– 10 – 40 kJ
formal charge also may change. Van der–waal’s –––––––––– 2 – 10 kJ
Example: Resonating structures of N2O
ELECTROVALENT OR IONIC BOND
Example: N = N = O ↔ N≡N→O
1 2 3 4 5 6
The strong electrostatic forces of attraction between two oppositely
charged ions which are formed due to transfer of electrons from
1
Qf 1 =5 − 4 − ( 4 ) =−1 one atom to another is called Ionic Bond (or) Electrovalent Bond.
2
Electrovalent bond is not possible between similar atoms. This
1 type of bonding requires two atoms of different nature. One atom
Qf 2 =5 − 0 − ( 8 ) =+1
2 should have the tendency to loose electrons i.e. electropositive
in nature and the other atom should have the tendency to accept
1
Qf 3 = 6 − 4 − ( 4) = 0 electrons i.e. electronegative in nature.
2
Formation of Ionic bond is a redox process, because one atom
1 undergoes oxidation and other one undergoes reduction.
Qf 4 = 5 − 2 − (6) = 0
2
Mg → Mg+2+ 2e–
1 2F + 2e– → 2F–
Qf 5 =5 − 0 − (8) =+1
2
Mg+2 + 2F– → MgF2 or Mg+2 (F–)2
1
Qf 6 =6 − 6 − ( 2 ) =−1 Example: IA and VII A group elements form strong ionic
2 compound.
The formal charges do not represent real ionic charges. Na + Cl → Na+ + Cl–
Formal charges represent a tendency to build up positive (or) 2, 8, 1 2, 8, 7 2, 8 2, 8, 8
negative charges.
(Ne) (Ar)
The lowest energy structure is the one with the smallest formal
charges on the atoms. (configuration) (configuration)
Force of attraction is equal in all direction so ionic bond is non–
 KEY NOTE directional.
The most stable structure is the one which has the smallest A definite three dimensional structure are called crystal lattice.
formal charge on the atoms or zero formal charge on the atom.
Energy released during the formation of one mole crystal lattice
is called lattice energy.
CLASSIFICATION OF CHEMICAL BONDS
e.g. (6.023 × 1023)Na+(g) + (6.023 × 1023) Cl–(g) →
On the basis of electronic valency theory and structure, chemical
bonds can be classified as follows: (6.23 × 1023) NaCl(s) + 788 KJ mol–1 (Lattice energy).
Ionic compounds do not have a molecular formula, they only have
empirical formula.

 KEY NOTE
Š More the distance between two elements in the periodic table
more will be the ionic character of the bond.
Š Total number of electrons lost or gained is called
electrovalency.

Chemical Bonding and Molecular Structure 67


Factors Favouring Formation of Ionic Bonds In size and charge, charge will dominate
1. Ionisation energy (I.E.): Amount of energy required to remove Na2O > NaF
an electron from the outermost orbit of an isolated gaseous atom NaCl < Na2S
to form positive ion or cation is called ionzation energy [energy is (d) Al2O3 Na2O MgO
absorbed so it is an endothermic process] Al2O3 > MgO > Na2O
M + I.E. → M+ + e–
Less Ionisation energy ⇒ Greater tendency to form  KEY NOTE
cation. Born - Haber cycle is used to calculate lattice energy.

Example : Characteristics of Ionic Compounds


(i) Physical state:
● Electrovalent compounds are generally crystalline, hard &
2. Electron affinity: Amount of energy released when an electron
brittle in nature.
is added to an isolated gaseous atom to form negative ion or anion
● These compounds are generally made from ions which are
is called electron affinity [energy is released so it is an exothermic
arranged in a regular way as a lattice structure.
process]
● Thus electrovalent compounds exist as three dimensional
         X + e– → X– + EA
solid aggregates.
High electron affinity ⇒ Greater tendency to form anions ● Normally each ion is surrounded by a number of oppositely
charged ions and this number is called co-ordination number.

LATTICE ENTHALPY
The Lattice Enthalpy of an ionic solid is defined as the energy
required to completely separate one mole of a solid ionic compound
into gaseous constituent ions or energy released during the formation
(ii) Boiling point and melting point: High boiling point and
of one mole crystal from its gaseous constituent ions. For example, melting points are due to strong electrostatic force of attraction.
the lattice enthalpy of NaCI is 788 kJ mol-1. This means that 788 kJ of
(iii) Electrical conductivity: It depends on ionic mobility. In
energy is required to separate one mole of solid NaCI into one mole
solid state there are no free ions so they are bad conductors of
of Na+1 (g) and one mole of Cl–(g) to an infinite distance. electricity.
The solid crystal being 3D; it is not possible to calculate lattice In fused state or aqueous solution free ions are present so they are
enthalpy directly from the interaction of forces of attraction and good conductors of electricity.
repulsion only.
Solid state < Fused state < Aqueous solution (Conductivity order)
Factors associated with the crystal geometry have to be included. (iv) Ionic reactions: Ionic compounds show ionic reactions &
Na+(g) + Cl–(g) → NaCl(s) covalent compounds show molecular reactions.
ΔH = ΔHlattice ⇒ –ve (always) Ionic reactions are fast reactions.
Mg2+(g) + 2Cl– (g) → MgCl2 (s) Example: When NaCl is added to AgNO3 solution, white ppt of
AgCl is formed at once.
Factors affecting L.E. Ag + NO3− + Na + Cl− 
→ Na + NO3− + AgCl ↓
1 white ppt
(i) Lattice energy (L.E.) ∝ r = r+ + r– = interionic distance
r (v) Solubility: Ionic compounds are soluble in polar solvent like
(ii) L.E. ∝ Z+, Z– H2O, HF etc.
Z+ ⇒ charge on cation in terms of electronic charge To explain solubility of ionic compound consider an example of
Z– ⇒ charge on anion in terms of electronic charge NaCl in water.
(iii) Size of cation ↑ L.E. ↑ H2O is polar solvent it can be represented as

Lattice energy of Factor
(a) NaCl > KCl (size) ● The Na+ ions gets associated with partially negatively
charged ‘O’ of water.
(b) NaCl < MgO (size, charge)
● And Cl– ions get associated with partially positively charged
(c) NaCl < MgCl2 (size) ‘H’ of water.
68 Dropper NEET
● Mobility of the ion: more is the hydration smaller will be the
mobility of the ions.

+ H 2O

Li+(aq) < Na+(aq) < K+(aq) < Rb+(aq) < Cs+(aq).
● Electrical conductance: It is related to mobility so follows the
same order.

– OH 2

COVALENT CHARACTER IN IONIC COMPOUNDS
(FAJAN’S RULE)
No Bond is 100% ionic in nature. It has some percentage of
Thus charge on Na+
and Cl–
decreases and electrostatic force also covalent character which is explained on the basis of Fajan’s
rule.
decreases which leads to free ion
In a binary compound AB, if the electronegativity difference
● Here, attraction force between H2O and Na+ or Cl– (Hydration between the elements A and B is equal to 1.7, the compound AB
energy) > Electrostatic force between ions in lattice (Lattice is 50% ionic.
energy)
Greater than 1.7, the compound AB is an ionic compound less than
● Energy released due to interaction between polar solvent 1.7, the compound AB is a covalent compound.(Exceptionally HF is
molecule and ions of solute is called solvation energy. If covalent compound even though electronegativity diffrence is 1.7)
water used as solvent it is called hydration energy.
● Condition of the solubility of ionic compound in water is Polarising Power
(Hydration energy > Lattice energy) When anion and cation approach each other, the valence shell of
anion is pulled towards cation nucleus and thus shape of anion is
Solvation or Hydration deformed. This phenomenon of deformation of anion by a cation
is known as polarisation and the ability of cation to polarize a
nearby anion is called as polarizing power of cation.

Polarisability: The tendency of anion to get distorted or polarised


by the cation is called its polarisability
Whenever any compound generally ionic or polar covalent is Polarising relates to cation:
dissolved in a polar solvent or in water then, different ions of 1
Polarising power ∝ charge on cation ∝
the compound will get separated and will get surrounded by size
polar solvent molecules. This process is known as solvation or Polarisability relates to anion
hydration. Energy released in this process is known as solvation Polarisability ∝ charge ∝ Size
energy or hydration energy Polarising power increases covalent character increases
The ionic compound will be soluble only if solvation energy Ex: In case of NaCl, MgCl2, AlCl3 the polarisation increases
(H.E.) is more than the lattice energy Na+ < Mg2+ < Al3+
Factors affecting solvation energy or hydration energy In case of AlF3 < AlCl3 < AlBr3 < AlI3
F– < Cl– < Br– < I–
1 1 
S.E ∝ Z+ Z– ∝  + 
Covalent character increases with increase in size of halide ion.
r+ r− Fajan pointed out that greater is the polarization of anion in a
 1  molecule, more is covalent character in it.
∝ 1 −  (nature of solvent) where ∈r is dielectric constant.
∈ More distortion of anion, more will be polarisation then covalent
 r 
character increases.
Greater the polarity, greater will be ∈r
Fajan gives some rules which govern the covalent character in the
∈r ↑ ⇒ 1/∈r ↓, 1–(1/ ∈r) ↑ ⇒ S.E. ↑ ionic compounds, which are as follows:
H2O CH3OH C2H5OH C6H6 (i) Size of cation : Smaller is the cation more is its polarizing
power and thus more will be the polarisation of anion.
∈r 81 34 27 3
Hence more will be covalent character in compound.
Applications of hydration energy Size of cation ∝ 1 / polarisation.
● Size of the hydrated ions: Greater the hydration of the ion e.g. BeCl2 MgCl2 CaCl2 SrCl2 BaCl2
greater will be its hydrated radii. Size of cation increases → Polarisation decreases →
Li+(aq) > Na+(aq) Covalent character decreases
Chemical Bonding and Molecular Structure 69
(ii) Size of anion : Larger is the anion, greater is its polarisability Ionic character increases, melting point increases ; since size of
and, therefore, more will be the polarisation. Thus more will cations increases & size of anion is constant.
be covalent character in compound.
CaF2, CaCl2, CaBr2, CaI2
Size of anion ∝ polarisation
LiF LiCl LiBr LiI Covalent character increases, melting point decreases ; since size
e.g . 

− Size of anion increases of anions increases & size of cation is constant.
− Polarisation increases
− Covalent character increases
COVALENT BOND
(iii) Charge on cation : Higher is the oxidation state of cation, A covalent bond may be defined as the bond formed by mutual
more will be the deformation of anion and thus, more will sharing of electrons between the participating atoms (which are
be covalent character in compound. short of electrons) of comparable electronegativity.
Charge on cation α polarisation.
NaCl MgCl2 AlCl3

Na + Mg 2+ Al 3+
e.g . 

− Charge of cation increases
− Polarisation increases
− Covalent character increases

(iv) Charge on anion : Higher is the charge on anion more will Covalency
be the polarisation of anion and thus more will be covalent
character in the compound. (i) It is defined as the number of covalent bonds formed by the
atom of the element with other atoms.
Charge on anion ∝ polarisation.
(ii) The usual covalency of an element except hydrogen is equal
AlF3 Al2O3 AlN to 8 minus the number of group to which the element
F − O2− N 3− belongs.
e.g .  →
− Charge on anion increases Ex: In NH3 the valency of Nitrogen is 3

− Polarisation increases In CO2 the valency of carbon is 4
− Covalent character increases
Lewis Structure and Covalent Bond
(v) Pseudo inert gas configuration of cation : Cation having (i) Structures in which valence electrons are represented by
pseudo inert gas configuration has more polarizing power dots are called Lewis structures.
than the cation that has inert gas configuration. Thus NaCl (ii) All atoms in the formulae will have eight electrons in its
having inert gas configuration will be more ionic whereas valence shell. H atom is an exception.
CuCl having pseudo inert gas configuration will be more (iii) Lewis dot formulae shows only the number of valence
covalent in nature. electrons, the number and kinds of bonds, but does not
Cu+ = [Ne] 3s2 3p6 3d10 Na+ = 1s2 2s2 2p6 depict the three dimensional shapes of molecules and
polyatomic ions.
18e– 8e–
(iv) Lewis formulaes are based on the fact that the representative
Pseudo inert gas configuration Inert gas configuration
elements achieve a noble gas configuration in most of their
(poor shielding of d-electrons) (more shielding of s and p electrons) compounds i.e. 8 electrons in their outermost shell (except
for H2, Li+ and Be2+ ions which have only 2 electrons)
Application & Exceptions of Fajan’s Rules
(i) Ag2S is less soluble than Ag2O in H2O because Ag2 S is No. of valence Example Lewis structure
more covalent due to bigger S2– ion. electrons
(ii) Fe(OH)3 is less soluble than Fe(OH)2 in water because Fe3+ 1 Hydrogen/Group IA H./Li.
is smaller than Fe2+ and thus charge is more. Therefore,
Fe(OH)3 is more covalent than Fe(OH)2 . 2. Helium/Group IIA He:/Mg:
(iii) The colour of some compounds can be explained on the 3. Group IIIA B
basis of polarisation of their bigger negative ions.
(iv) Variation of melting point 4. Group IV A C
[melting point of covalent compound< melting point of ionic
: : : :

5. Group V A N
compound]
BeCl2, MgCl2, CaCl2, SrCl2, BaCl2 6. Group VI A :O

70 Dropper NEET
Factors Favouring the Formation of Covalent Bond Molecular solids: They are formed when one atom combines
(1) Small size of cation with another by a covalent bond and then the molecule combines
(2) Large size of anion with another similar molecule with the help of Van der waal’s
(3) Large charge on cation and Anion force of attraction or hydrogen bond.
(4) Generally a covalent bond is formed between 2 atoms of Example : CH4(Solid), dry ice, ice.
similar E.N. values
(iii) Melting and boiling points: With the exception of a few
High ionisation enthalpy: Atoms having high ionisation which have giant three dimensional structures such as diamond,
enthalpies do not have a tendency to lose electrons to form cations.
carborundum (SiC), Silica (SiO2), others have relatively low
Such elements prefer to form covalent bonds. Eg: Cl-Cl
melting and boiling points.
Comparable electronegativity: Atoms with equal or nearly
equal electronegativities tend to share equally a pair of electrons This is due to the presence of weak attractive forces between the
with opposite spins. This leads to a better overlap of their atomic molecules.
orbitals thus resulting in the formation of a covalent bond. (iv) Electrical conductivity: In general, covalent substances are
Note: Sidgwick concept of maximum covalency bad conductors of electricity. Since they do not contain charged
The maximum covalency of an element actually depends on the particles or free electrons.
period of the periodic table to which it belongs. Substances which have polar character like HCl in a solution, can
Period Elementt Max. Covalency conduct electricity.

1 H 2 Graphite can conduct electricity since electrons can pass from one
layer to other.
2 Li to F 4
Some show conductivity due to self ionisation, example:
3 Na to Cl 6
Liq.NH3, H2O, etc.
4 K to Br 6
NH3 + NH3 → NH4+ + NH2–
5 Rb onwards and rest of the 8
6 elements H2O + H2O → H3O+ + OH–

 KEY NOTE (v) Chemical reactions: Covalent substances give molecular


reactions. Reaction rate is usually low because it involves
This rule explains the formation of PCl5 and SF6. This also
two steps (i) breaking of covalent bonds of the reactants (ii)
explains why Nitrogen does not form NF5 or NCl5. Because
Nitrogen belongs to second period and the maximum covalency establishing new bonds. While in ionic reactions there is only
of Nitrogen is 4. regrouping of ions.
(vi) Solubility: Non polar compounds are soluble in non polar
Characteristics of Covalent Compounds solvents. Non polar solvents are CCl4, benzene, CS2 etc.
(i) Physical state: Under normal temperature and pressure, they Polar compounds are soluble in polar solvents (i.e. ΔE.N. is 0.9 to
exist as gases or liquids of low boiling points.
1.8). Polar solvents are H2O, CHCl3, alcohol etc.
This is due to the fact that very weak forces of attraction (Van
(vii) Isomerism: Covalent bonds are rigid and directional. On
der waal’s forces) exist between the molecules due to which
molecules are far from each other. account of this there is a possibility of different arrangement of
atoms in space. Covalent compounds can thus shows isomerism
If their molecular masses are high they exist as soft solids e.g.
sulphur, phosphorus and iodine. (structural and spatial).

(ii) Crystal structures: Covalent solid – In this type of structure MODERN CONCEPT OF COVALENT BOND (VBT)
every atom is bonded to four other atoms by single covalent bonds
resulting in the formation of a giant structure e.g. SiC, AlN and Enthalpy Diagram for H2 Molecule
diamond. These crystals are very hard and possess high melting When two hydrogen atoms are at an infinite distance from each
point. other, there is no interaction between them and therefore, enthalpy
Diamond is sp3 hybridized carbon atom and it forms a tetrahedral of the system is assumed to be zero in this state (stage-A). As
structure. the two atoms start coming closer to each other, the potential
Graphite is sp2 hybridized carbon atoms and it forms hexagonal enthalpy continues to decrease (stage B). Ultimately a stage is
layers which can slide over each other due to weak Van der reached when the enthalpy of the system becomes minimum and
waal’s forces of attraction. Distance between C–C atom is 1.42 hydrogen atoms are said to be bonded together to form a stable H2
Å distance between layers is 3.6 Å. Graphite is more stable than
molecule (stage C).
diamond due to high value of change in enthalpy.
Chemical Bonding and Molecular Structure 71
If both orbitals having opposite sign, called out of phase
overlap (destructive). And if both orbitals are out of phase
Energy (kJ/mol) due to the direction of approach then zero overlap.

r0
0
Distance of separation

Bond
435.8 Energy
Bond Length 74 pm Internuclear distance
The internuclear distance r0 between two hydrogen atoms at this
stage is referred to as bond length. In the case of the hydrogen
molecule, the bond length is 74 pm. It should be noted that two
hydrogen atoms can not be brought at a distance lesser than ro
(i.e., 74 pm) because the potential enthalpy of the system increases Types of Overlapping and Nature of Covalent Bonds
and curve shows an upward trend (dotted lines) and molecule ● Linear overlapping of atomic orbitals results in the formation
becomes unstable.
of sigma bond (σ).
VBT cannot explain paramagnetic behavior of O2. ● A covalent bond formed by the sidewise overlap of atomic
orbitals of atoms already bonded through a ‘σ’ bond and in
Main Points of Valence Bond Theory which the electron cloud is present on either side of the inter
(i) A covalent bond is formed by partial overlapping of two nuclear axis is known as a pi bond (π).
atomic orbitals ● Lateral overlapping of atomic orbitals results in the formation
of pi bond (π).
● ‘π’ bond is formed only after the formation of ‘σ’ bond.
● Any type of orbitals can involve in ‘σ’ bond formation.
● Only ‘p’ or ‘d’ orbitals can involve in ‘π’ bond formation.
● Single bond is equal to one ‘σ’ bond.
● Double bond is a combination of one ‘σ’ bond and one ‘π’ bond.
(ii) More is the extent of overlapping between the two atomic ● Triple bond is a combination of one ‘σ’ bond and two ‘π’ bonds.
orbital, stronger will be bond. Strength of Sigma and pi Bonds
Strength of the bonds follows the order
triple bond > double bond > single bond.
Strength of the bonds follows the order : σp–p > σs–p > σs–s > π
[Principal Quantum no. same, n = 2]
Q s orbital are spherical in nature so they are least diffused  KEY NOTE
hence it will provide less area for overlapping. ‘σ’ bond between ‘1s’ orbitals is exceptionally stronger
(iii) Orbitals which are undergoing overlapping must be such
that Difference in σ and π Bonds
(a) Each orbital should have one electron with opposite
Sigma (σ) bond Pi (π) bond
spin (for formation of covalent bond).
(b) One orbital have pair of electron and the other orbital 1. It results from the end to It result from the sidewise (lat-
have no electron (for formation of co-ordinate bond). end overlapping of two s eral) overlapping of two p or-
orbitals or two p–orbitals bitals.
(iv) If the overlapping is along the molecular axis then bond will or one s and one p-orbital
be sigma (σ) & in the perpendicular direction, it will be pi(π)
bond. 2. Its bonded orbital consists Its bonded orbital consists of
of a single electron cloud two electron clouds one above
symmetrical about inter- and the other below the plane
nuclear axis of symmetry
3. Strong Weak
4. Bond energy 80 kcals 65 kcals
5. More stable Less Stable
(v) If two atomic orbitals having same sign (+ or – ) overlap with
each other, it is called in phase overlapping (constructive). 6. Less reactive More reactive

72 Dropper NEET
Where two or more resonance structures can represent a molecule,
Sigma (σ) bond Pi (π) bond
the VSEPR model is applicable to any such structure.
7. Can independently exists Always exists along with a σ The repulsion between electron pairs on central atom follows the
bond hybridsation order
8. Hybridization depends doesn’t depend on p bond l.p-l.p>l.p-b.p>b.p-b.p.
upon σ bond In VSEPR theory the number of electron pairs in single bond or
9. The groups or atoms can Due to resistance to rotation double bond or triple bond or dative bond is counted as only one
undergo bond rotation around the π bond the groups pair because all the electron pairs in the same bond are oriented in
about single sigma (σ) attached to it are not free to ro- the same direction.
bonds tate. The magnitude of repulsions between bond pair of electrons
depends on the electronegativity difference between the central
10. The σ electrons are referred In π bond the electrons are held atom and other atom.
as localised less firmly and thus can be easi-
Identification of hybridisation and shape of a molecule or ion
ly dissociated or polarised by an
having single central atom.
external charge and hence the π
– Rules:
e are referred as mobile elec-
trons. Step-1: Calculate valence electrons in central atom.
11. Shape of the molecule is π bonding does not affect the Step-2: If it is cation then substract charge from central atom
determined by the σ bonds shape of the molecule Step-3: If it is anion then add charge to the central atom.
present in the molecule Step-4: Bond pairs = No.of atoms attached to central atom.

TRAIN YOUR BRAIN Step-5: Lone pairs =


 No. of electrons given by bonded atoms 
Q. Identify the correct statement : (Valence e– on central atom) –  
 2 
(a) Single N–N bond is stronger than single P–P bond
Step-6: B.P+L.P = Electron Pairs
(b) Single N–N bond is weaker than single P–P bond
2 3 4 5 6 7
(c) N ≡ N is weaker than P ≡ P
↓ ↓ ↓ ↓ ↓ ↓
(d) None of these
sp sp2 sp3 sp3d sp3d2 sp3d3
Ans. (b) Due to small size of nitrogen, the lp–lp repulsion is more
than that in P. Hence statement B is correct
Q. How many sigma and pi bonds are present in  KEY NOTE
tetracyanoethylene? If bonded atoms are H, F then each one contribute one electron
(a) Nine σ and nine π (b) Five π and nine σ for bond formation. If bonded atoms are O, S then each one
(c) Nine σ and seven π (d) Eight σ and eight π contribute two electrons for bond formation.
Ans. (a)
N C C N Shape (molecular geometry) of Some Simple
C C Molecules/Ions
N C C N

VALENCE SHELL ELECTRON PAIR REPULSION No. of No of Formula Molecular Example


(VSEPR) THEORY B.Ps L.Ps shape
Postulates 2 0 AB2 Linear BeCl2, BeF2,
The shape of a molecule depends upon the number of valence C2H2
shell electron pairs (bonded or non-bonded) around the central 3 0 AB3 Trigonal BF3, BCl3
atom. Planar
Pairs of electrons in the valence shell repel one another since their 4 0 AB4 Tetrahedral CH4, NH4+,
electron clouds are negatively charged. CCl4
These pairs of electrons tend to occupy such positions in space 5 0 AB5 Trigonal PCl5
that minimise repulsion and thus maximise distance between bipyramidal
them.
The valence shell is taken as a sphere with the electron pairs 6 0 AB6 Octahedral SF6
localising on the spherical surface at maximum distance from one 2 1 AB2E Bent SO2, O3
another.
3 1 AB3E Trigonal NH3
A multiple bond is treated as if it is a single electron pair and Pyramidal
the two or three electron pairs of a multiple bond are treated as a
single super pair. 2 2 AB2E2 Bent H2O

Chemical Bonding and Molecular Structure 73


4 1 AB4E See Saw SF4 HYBRIDISATION
3 2 AB3E2 T-Shape ClF3 ● The concept of hybridisation was introduced by Pauling
● The intermixing of atomic orbitals of almost same energy and
5 1 AB5E Square BrF5
their redistribution into an equal number of identical orbitals
Pyramidal
is known as hybridisation
4 2 AB4E2 Square XeF4
● The orbitals of one and the same atom only involve in
Planer
hybridisation.
A = Central atom in the compound. ● The no.of hybrid orbitals formed is numerically equal to
B = atom linked to the central atom no.of orbitals participating in hybridisation.
E = Lone pairs of electron ● The hybrid orbitals symmetrically arranged around the
nucleus such that they have maximum stability.
TRAIN YOUR BRAIN ● The order of repulsions will be in order of lp-lp> lp-bp>bp-
bp repulsions.
Q. The interhalogen compound that cannot exist is
● The orbitals involving in the hybridisation have different
(a) IBr3 (b) ICl7 (c) IF4 (d) BrF5 shapes but almost same energy.
Ans. (c) I does not have its valency 4. It has valency 1, 3, 5 & 7 ● The orbitals formed in hybridisation process are called hybrid
Q. How many number of species do not exist :- orbitals.
PCl5 , PH5 , SF6 , SH6 , PBr6–, OF4 ● The hybrid orbitals have same shape and same energy.
● The angle between any two hybrid orbitals in an atom is
(a) 2 (b) 3
generally same.
(c) 4 (d) 5
● Electron filling in hybrid orbitals obeys Hunds rule and
Ans. (c) PH5, SH6 → not exist due to non expandable covalency Pauli’s rule.
OF4 → O-atom does not have vacant d-orbital. ● The hybrid orbitals involve only in σ bond formation. They
PBr6– → steric repulsion never involve in ‘π’ bond formation.
● The concept of hybridisation was introduced to explain the
TRAIN YOUR BRAIN shapes of molecules, bond angles and bond lengths in the
Q. Which of the following pairs of species have identical molecules.
shapes?
● A half filled or completely filled or even vacant orbital can
(a) NO2+ and NO2– (b) PCl5 and BrF5 participate in Hybridization.
(c) XeF4 and ICl4– (d) TeCl4 and XeO4
● The hybrid orbitals are more effective in forming stronger
bonds that leads to the formation of more stable molecules.
Ans. (c) (a)
 KEY NOTE
Same atom undergoes different types of hybridisation under
different situations.
More the directional bond, greater is the bond strength
(b)
sp3 – sp3>sp2–sp2>sp–sp>p–p>s–p>s–s

sp-hybridisation
● This is also known as diagonal hybridisation or linear
hybridisation.
● One s- orbital combines with one p- orbital to give two
identical orbitals called sp - hybrid orbitals.
(c)
● The angle between the two sp-hybrid orbitals in an atom is 180°

1
● sp- hybrid orbital will have s- character or 50% s- character.
2
1
p- character or 50% p- character.
(d) 2
● The hybridisation of central atom in a linear molecule is
See-Saw Tetrahedral generally sp.

74 Dropper NEET
sp2-hybridisation sp3d2- hybridisation
● This is also known as trigonal hybridisation. ● One s-orbital , three p-orbitals and two d- orbitals combines
● One s- orbital combines with two p-orbitals to give three to give six identical orbitals called sp3d2 hybrid orbitals.
identical orbitals called sp2-hybrid orbitals. ● The angle between any two sp3d2 hybrid orbitals is 90° and
180°.
● The angle between any two sp2-hybrid orbitals in an atom is
120°. 1
● sp3d2 hybrid orbital will have th s-character or 16.7% s-
1 1 16
character, p − character & d − character
● sp2 hybrid orbital will have rd s-character or 33.3% 2 3
3
2 3 2
Example for sp d hybridisation is SF6
s-character and rd p- character or 66.7% p-character.
3
● 3dz and 3dx2–y2 orbitals are involved in sp3d2 hybridisation.
● The hybridisation of central atom in a molecule having 2

trigonal planar shape is sp2. Assigning the Type of Hybridisation


sp3- hybridisation (I) Total molecular valency method
Step 1: Total number of valence electron in the molecule is
● This is also known as tetrahedral hybridisation or tetragonal
calculated by adding the individual valency of all constituent atoms.
hybridisation.
Step 2: If total electrons are greater than eight, then divided by
● One s- orbital combines with three p-orbitals to give four
eight and quotient is equal to hybridised orbitals.
identical orbitals called sp3 hybrid orbitals.
e.g:.PCl5
● The angle between any two sp3-hybrid orbitals in an atom is
109°28'. Total valence electrons = 5+(5 × 7)=40
40 / 8=5=no.of hybridised orbitals → sp3d
1 3
● sp3- hybrid orbital will have th s-character or 25% s- character. th If remainder comes, that remainder should be divided by 2, initial
4 4
quotient comes then sum of quotient and final quotient is equal to
p-character or 75% p-character. number of hybridised orbitals, e.g., XeOF2
● The hybridisation of central atom in a molecule having Total valence electrons = 8 + 6 + (2 x 7) = 28
tetrahedral shape is sp3. 28 / 8 Quotient=3 ; Remainder = 4
dsp2 hybridisation Remainder is divided by 2
4 / 2 Quotient = 2
● The orbitals involved in hybridisation are dx2 – y2, s and
two p-orbitals having square planar geometry, bond angle Hence, sum of Quotient = 3+2 = 5 = no. of hybridised orbitals →
between any two dsp2 orbitals is 90°. sp3d
Step 3: If total valence electrons are either eight or less is divided
sp3d-hybridisation by 2,what Quotient appears,should be considered as number of
● One s-orbital, three p-orbitals and one d-orbital combines and hybridised orbitals, e.g., H2O
gives five identical orbitals called sp3d-hybrid orbitals. Total valence electrons = 2 + 6 = 8
● The angle between two sp3d hybrid orbitals in an atom is 90° 8 / 2 = 4 = sp3
and 120°. (II) Total number of Hybridised orbitals(For neutral molecule)
1
● sp3d hybrid orbital will have th s-character or 20% = 1/2(V + M)
5
3 1 V = no. of valence electrons
s-character, th p-character or 60% p-character, th M = no. of monovalent atom
5 5
d-character or 20% d-character.
Total number of Hybridised orbitals(For cationic species)
● The hybridisation of central atom in a molecule having = 1/2(V + M – C)
trigonal bipyramid shape is sp3d.
Total number of Hybridised orbitals(For anionic species)
Example for sp3d hybridisation is PCl5 = 1/2(V + M + A)
● 3dz orbital is involved in sp3d hybridisation.
2 C = +ve charge. A = -ve charge.
● The axial bonds (2.19Å) have been found to be larger than
equatorial bonds (2.04Å) since axial P-Cl bonds experience
Hybridisation in Carbon Compounds
greater repulsion from equatorial P-Cl bonds. ● The carbon atom in carbon compounds is tetravalent (i.e.,
No. of planar atoms in PCl5 is 4. carbon atoms form four covalent bonds).
PCl5 is highly reactive and in the solid state exist as [PCl4]+ ● The sp3 carbon atom involves in four single bonds, C
and [PCl6]– ions.

Chemical Bonding and Molecular Structure 75


● The sp2 carbon atom involves in a double bond and two single The molecular orbitals like atomic orbitals are filled in
accordance with the Aufbau principle obeying the Pauli's
bonds, =C exclusion principle and the Hunds rule.
● The sp carbon atom involves in two double bonds or one The molecular orbitals are named as σ, π, δ etc.
triple bond and one single bond = C = or ≡ C –
According to wave mechanics, the atomic orbitals can be
Types of expressed by wave functions (ψ), which represent the amplitude of
Steric number Geometry
Hybridisation waves. These are solutions of schrodinger wave equation. However
it is difficult to solve the above exactly for many electron systems.
2 sp Linear
To over come this problem scientists have used an approximate
3 sp2 Trigonal planar method known as LCAO (Linear Combination of Atomic Orbitals)
4 sp3 Tetrahedral method.
The atomic orbitals of two hydrogen atoms A and B may be
5 sp3d Trigonal
represented by wave functions ψA and ψB .
bipyramidal
Mathematically, formation of molecular orbitals may be
6 sp3d2 Octahedral
described as
7 sp3d3 Pentagonal ψMO = ψA ± ψB
bipyramidal
σ = ψA + ψB (Bonding molecular orbital)
σ* = ψA – ψB (Antibonding molecular orbital)
TRAIN YOUR BRAIN
Quantitatively, the formation of molecular orbitals can be
Q. In which of the following central atom is unhybridised? understood in terms of the constructive or destructive interference
(a) S(CH3)2 (b) SO2 of the electron waves of the combining atoms.
During the formation of BMO, the two electron waves
(c) SiH4 (d) PCl3
of the bonding atoms reinforce each other due to constructive
Ans. (a) As per Drago’s rule interference.
During the formation of ABMO, the electron waves cancel
MOLECULAR ORBITAL THEORY
each other due to destructive interference.
It was proposed by Hund and Mulliken to explain paramagnetic
As a result,the electron density in a bonding molecular
nature of oxygen molecule.
orbital is located between the nuclei of the bonded atoms but in
The electrons in a molecule are present in various molecular case of antibonding molecular orbital most of the electron density
orbitals as the electrons of atoms are present in the various atomic is located away from the space between nuclei( nodal plane).
orbitals.
Molecular orbitals are formed by linear combination (LCAO)
of atomic orbitals of comparable energies and proper symmetry.
This means that 1s orbital can combine with another 1s orbital
but not with 2s orbital because the energy of 2s orbitals is higher
than that of 1s orbital. This is true only for homonuclear diatomic
molecules.
By convention z-axis is taken as the molecular axis, 2pz
orbital of one atom can combine with 2pz orbital of the other atom
but not with the 2px or 2py orbitals because of their symmetry.
Molecular orbital is polycentric i.e. an electron in molecular
orbital is under the influence of two or more nuclei.
The number of molecular orbitals formed is equal to the
number of combining atomic orbitals. When two atomic orbitals
combine, two molecular orbitals are formed. One is known as
bonding molecular orbital while the other is called anti bonding
orbital.
It may be noted that the bonding molecular orbital is
stabilized to the same extent as the anti bonding molecular orbital
is destablised i.e. energy lost by BMO is equal to energy gained
by ABMO but total energy of two MO equal to sum of energy of
two atomic orbitals.
Order of energies of various orbitals is bonding orbitals < non
bonding orbitals < anti bonding oritals.

76 Dropper NEET
The increasing order of relative energies of M.O having less than Bond order ∝ Bond energy
or equal to 14 electrons. ∝ stability of a molecule
σ1s < σ*1s < σ2s < σ*2s < π2px = π2py < σ2pz < π*2px 1

= π*2py < σ*2pz bond length
for more than 14 electrons 10 − 4
N2
Bond order of= = 3
σ1s < σ*1s < σ2s < σ*2s < σ2pz < [π2px = π2py] <[π*2px 3
9−4
= π*2py] < σ*2pz Bond order of = N 2+ = 2.5
2
Difference between σ and π MO’s 2+ 8 −4
Bond order of N = 2 = 2.0
2
σ-molecular orbital π- molecualr orbital 10 − 5
Bond order of= N 2− = 2.5
1. Formed by the end 1. Formed by the sidewise 2
on overlap along the overlap perpendicular to 10 −6
internuclear axis internuclear axis Bond order of = N 22 − = 2
2
2. Overlapped region is very 2. Overlapped region is As the bond order increases the stability of molecule increase.
large small However, some of the bond orders are identical. In such case, a
3. Rotation about the 3. Rotation about the molecule or ion with more number of electrons in their antibonding
orbitals is less stable. Hence correct order is
internuclear axis is internuclear axis is
symmetrical unsymmetrial N22 – < N22 + < N2– < N2+ < N2.

4. Strong bonds are favoured 4. Weak bonds are The bond orders of 1,2 or 3 correspond to single, double or triple
favoured bond. But bond order may be fractional in some cases.
The magnetic properties of molecules can also be ascertained
 KEY NOTE Bonding in some diatomic molecules and ions
If all the electrons in the molecule or ion are paired it is Hydrogen molecule (H2)
diamagnetic in nature.If it is unpaired that is paramagnetic.
Total number of electrons = 2, filling in molecular orbitals we
Magnetic moment
= µ n ( n + 2 ) B.M have
σ12s < σ1*0s
Stability of Molecules
( Nb − N a )
2−0
If Nb is the number of electrons occupying bonding orbitals and Bond order = = = 1
2 2
Na the number of electrons occupying antibonding orbitals, then
Hence there is a single bond between two hydrogen atoms and
A positive bond order is Nb>Na means a stable molecule. While a due to absence of unpaired electrons it is diamagnetic.
negative (Nb<Na) or zero (Nb=Na) bond order means an unstable
molecule Helium molecule (He2)
Electronic configuration / Bond order of simple diatomic molecules The total number of electrons = 4 and filling in molecular orbitals
The electronic configuration and the bond order in case of simple we have
diatomic molecules can be obtained by filling the molecular σ12s < σ1*2s

orbitals by applying Aufbau principle and Hund's rule etc.
( Nb − N a )
2−2
Bond Order Bond order = = 0
=
2 2
The relative stability of a molecule can be determined on the basis Hence, He2 molecule can not exist.
of bond order. It is defined as the number of covalent bonds in
Nitrogen molecule (N2)
a molecule. It is equal to one half of the difference between the
The total number of electrons =14 and filling in molecular orbitals
number of electrons in the bonding and antibonding molecular
we have
orbitals.
π2 px2 
1 σ12s σ1*2s σ22 s σ*2 σ2
Bond order = [Number of bonding electrons – Number of 2s  2  2 pz
2  π 2 p y
antibonding electrons]
Bond order =
( Nb − N a ) 10 − 4
N − Na = = 3
or, = b 2 2
2 It is diamagnetic.
Chemical Bonding and Molecular Structure 77
Oxygen molecule (O2) Boron Molecule (B2)
Total number of electrons =16 and electronic configuration is No of electrons = 10
The electronic configuration is σ1s2 σ*1s2 σ2s2 σ*2s2 π2p1x = π2py1
π2 px2   π* 2 p1x 
σ12s σ1*2s σ22 s σ*2
2s σ 2
2 pz  2  *
σ*
1  2 pz It has 2 paired electrons. Hence, paramagnetic.
π2 p y   π 2 p y 
Basic Idea of Metallic Bonding
( Nb − N a )
10 − 6
In a metal crystal, all atoms are identical. So, these cannot be
Bond order = = = 2
2 2 linked by electrovalent bonds as all have same E.N..
As shown by electronic configuration the O2 molecule contains two Hence, in metal crystal, the atoms are bonded with each other
unpaired electrons, hence it is paramagnetic in nature. with a special type of bonding known as Metallic bonding.
Since the metals are electropositive, their ionisation evergies are low.
O+2 ion
So they readily lose their valence e–s and convert into positive ions.
Total number of electrons (16 - 1) = 15, These e–s move from place to place through the empty valence orbitals
Electronic configuration and are shared simultaneously among all of them.

π2 px2   π* 2 p1x  Essential Conditions for Metallic Bonding


σ12s σ1*2s σ22 s σ*2
2s σ 2
2 pz  2  *
*
 σ 2 pz
π2 p y   π 2 p y  The metal atoms should have low I.E.
There should be sufficient number of vacant orbitals in the valence
10 − 5 shell.
Bond order = = 2.5
2 Thus, the bonding which holds the metal atoms together on
It is paramagnetic. account of the force of attraction between metal ions & the mobile
e–s is called metallic bonding.
O2– Super oxide ion The strength of metallic bond is directly proportional to the no. of
valence e-s and the charge present on the nucleus. It provides an
Total number of electrons (16 +1) = 17. Electronic fonguration
answer to the fact that alkali metals are soft and have low M.P. &
π2p 2x  π* 2p 2x  B. P.. While Transition metals are hard and have high M.P. & B.P.
2 *2 2
σ1s σ1s σ2s σ*2
2s σ 2
2p z  2  * 1  σ2pz
*

 π2p y   π 2p y 
TRAIN YOUR BRAIN
Bond order =
( N b −=
Na ) 10 − 7
= 1.5 Q. How many number of molecules have bond dissociation
2 2 energy greater than that of I2 ?
It is Paramagnetic. F2, Cl2, Br2, N2, O2
Peroxide ion (O22 – ) (a) 2 (b) 3
(c) 4 (d) 5
Total number of electrons (16 + 2) = 18. The electronic
configuration is Ans. (d) Cl2 > Br2 > F2 > I2
N ≡ N , O = O there for B.D.E is higher than I2
 2

 π2 p x  π* 2 px2  DIPOLE MOMENT
σ12s σ1*2s σ22 s σ*2
2s σ22 pz  2  * 2  σ 2 pz
*

 π2 p y   π 2 p y  When a covalent bond is formed between two similar atoms (For


  ex H2, F2, O2) the shared pair of electrons situated exactly between
two atoms, the bond so formed is called non-polar covalent bond.
10 − 8 If a covalent bond is formed between two dissimilar atoms (ex
Bond order = =1
2 H-F) the shared pair gets displaced more towards Fluorine since
It is diamagnetic. Fluorine is more E.N. than Hydrogen, such bond is called polar
Bond dissociation energy order covalent bond.
Polarity of a polar molecule is expressed in terms of dipole
N2 > N2+ = N2– > N22–
moment (μ).
Bond length order
Dipole moment (μ)
N22– > N2– = N2+ > N2
= charge on the pole × distance between the poles.
Bond dissociation energy order Dipole moment is usually expressed in Debye units (D)
O2+ > O2 > O2– > O22– 1 D = 3.33564 × 10–30 Cm (S.I) where
Bond length order C = coulomb and m = meter
O22– > O2– > O2 > O2+ = 10–18 esu. Cm (CGS)

78 Dropper NEET
Dipole moment is vector quantity and is represented by crossed Net dipole moment of water = 1.85 D
arrow . The cross is on positive end and arrow head is on = 1.85 × 3.336 x 10–30 Cm
negative end. For ex
H–F = 6.17 × 10–30 Cm
In polyatomic molecules the dipole moment not only depends
upon the individual dipole moments of bonds known as bond
dipoles but also on the spatial arrangements of various bonds in
the molecule. In such case, the dipole moment of a molecule is the
vector sum of the bond dipoles of various bonds.

Calculation of Resultant Bond Moment


Let AB & AC are two polar bonds inclined at angle θ, their
In NF3 the bond pairs are moving opp. to lone pair and in NH3

:
dipole moments are μ1 & μ2. The resultant dipole moment may be
the bond pairs moving towards the lone pair.
calculated using µ R = µ12 + µ 22 + 2µ1µ 2 cos θ
Cis and trans isomers can be distinguished by dipole moments,
usually cis isomers have higher dipole moment and hence higher
polarity e.g.

Type of Dipole Mo-


Example Geometry
Molecules ment, μ(D)

HF 1.78 Linear Dipole moment is greatest for ortho isomer, zero for para isomer
which is less than that of ortho or meta isomer.
HCl 1.07 Linear
AB HBr 0.79 Linear
HI 0.38 Linear
H2 0 Linear

H2O 1.85 Bent


AB2 H2S 0.95 Bent
CO2 0 Linear μ = 2.53 μ = 1.48 μ=0
Hybridisation can be determined by dipole moment for e.g.
NH3 1.47 Trigonal-
pyramidal (i) If a molecule AB2 has μ = 0, the σ orbitals used by A (Z <
21) must be sp hybridised e.g. BeF2
NF3 0.23 Trigonal-
AB3
pyramidal F Be F
BF3 0 Trigonal-
(ii) If a molecule AB3 has µ= 0, the σ orbitals used by A (Z <
planar
21) must be sp2 hybridised e.g. BF3
CH3Cl 1.87 Tetrahedral F
AB4 CHCl3 1.04 Tetrahedral F B
CCl4 0 Tetrahedral F
Note:
Applications of Dipole Moment 1. Dipole moment ∝ electronegativity difference
Dipole moment is helpful in predicting the geometry of the HF > HCl > HBr > HI
molecule. 2. Dipole moment ∝ No. of lone pair of electrons.
Dipole moment helps in determining the polarity. HF > H2O > NH3
Dipole moment can distinguish between symmetrical and non 3. For homo atomic molecules and molecules having normal
symmetrical molecules e.g. CO2 has 0 diploe moment as it is shapes such as linear, trigonal, tetrahedral posses zero
symmetrical whereas H2O has a dipole moment of 1.85 D. dipole moment.
Ex: Cl2, N2, O2, CO2
4. Molecules having distorted shapes like angular, pyramidal
shows dipole moment
Ex: H2O, SO2

Chemical Bonding and Molecular Structure 79


Some important Points about Dipole Moment
 KEY NOTE
A polyatomic molecule having polar covalent bonds but zero
Š Resonance energy = Actual bond energy – Energy of most
dipole moment indicates the symmetrical structure of the
stable resonating structure.
molecule. e.g. B–F bonds are polar in BF3 but BF3 has µ = 0 due Š Stability of molecule ∝ resonance energy.
to its symmetrical geometry. Š More is the number of covalent bonds in molecule more will
If molecule have µ = 0, then it should be linear or having be its resonance energy.
Š Resonance energy ∝ number of resonating structures.
symmetrical geometry.
e.g. linear – CO2, CS2, BeCl2 (g) ; symmetrical geometry – BF3, BOND LENGTH
CH4 , PCl5 , SF6 , IF7 , XeF4.
The equilibrium distance between the nuclei of two bonded atoms
If molecule has µ ≠ 0 then it should be angular or having in a molecule is called bond length.
unsymmetrical geometry. Bond length is measured by spectroscopic, x-ray diffraction, and
e.g. SnCl2 , PbCl2, SO2 – angular molecular geometry. electron -diffraction techniques.
NH3, H2O, NF3, SF4, H2S – unsymmetrical molecular geometry. Covalent radius of an atom is equal to half the internuclear
distance between two identical atoms that are held together by
RESONANCE covalent bond.
Representation of molecule in more than one structure and none d A− A
Covalent radius of A =
of them explains all the properties of the molecule singly is called 2
Covalent radius of an atom in heterodiatomic molecule (A-B type)
Resonance.
when the E.N. difference between two atoms is large is calculated
Example: by using pauling empirical equation.
dA–B = rA + rB + C(XA – XB)
Where XA and XB= E.N. of atoms A and B the value of C depends
on the type of atoms involved in covalent bond.
For bonds involving atoms of II nd period, C = 0.09
For bonds between Si, P and S bonded to more E.N atoms not
belonging to first period, C = 0.06.
The bond length between a set of atoms in different molecules is
always same.
O-H bond length in H2O, H2O2, C2H5OH is same i.e. 0.97Å
As the p-character of hybrid orbital changes the bond length
changes.
C-H bond length in C2H6, C2H4 and C2H2 are 1.09Å, 1.08Å and
1.06 Å respectively.
As number of bonds between two atoms increases bond length
decreases.
C – C; C = C; C≡C
1.54 Å; 1.33 Å; 1.20 Å
Bond order = As the size of bonded atom increases bond length increases.
Total No. of bonds formed between two atoms in all structures HF HCl HBr HI
Total No. of resonating structures 92pm 127pm 141pm 160pm

 KEY NOTE
It is effected by resonance and hyperconjugation.

BOND ENTHALPY
2 +1+1 It is the amount of energy required to break one mole of bonds of
Bond order = = 1.33 a particular type between two atoms in a gaseous state.
3
80 Dropper NEET
The unit of bond enthalpy is kJ/mole TRAIN YOUR BRAIN
H2(g) → 2H(g), ΔH°d = 435.8 kJ/mole Q. The ONO angle is maximum in :
(a) HNO3 (b) NO+2
Heterolytic fission requires more bond energy than homolytic (c) HNO2 (d) NO2
fission. 102°

The number of bonds between two atoms increases as bond H 12


1 O
Ans. (b) (a) 141
enthalpy increases. O N 130°

C-C C=C C≡C


114°
O
+
341 610 828 kJ/mole
(b) O = N = O Bond angle is 180º because of sp
115 pm
As the number of lone pairs on a bonded atom increases, the bond hybridisation of nitrogen.
enthalpy decreases. 102°
H
•• •• •• •• (c) 143 (d)
−C − C − > − N − N − > − O − O − O N

11
•• ••

8
111° O
In case of H2O molecule, the enthalpy needed to break two O-H
bonds is not the same due to changed chemical environment. BOND ORDER
Therefore mean or average bond enthalpy is used. ● Bond order indicates number of bonds between two atoms in
–1
a molecule.
H2O(g) → H+(g) + –OH(g), ΔaH1Θ= 502 kJ mol ● Greater the bond order more is the strength of the bond.
–OH –1
→ H+(g) + O2–(g), ΔaH2Θ = 427 kJ mol ● With increase in bond order, bond enthalpy increases and
(g)
bond length decreases.
Average bond enthalpy of O-H bond in water ● Bond order 3 is being one of the highest for a diatomic molecule.

502 + 427 ● Isoelectronic species have same bond order.


= = 464.5 kJ
2 Eg,. N2, CO,NO+
● It is observed that with the increase in bond order, bond
 KEY NOTE energy increases and bond length decreases.
Š The magnitude of bond energy depends upon size of the
atoms forming the bond. HYDROGEN BOND
Š Shorter the bond length higher the bond energy. Bond energy Nitrogen, oxygen and fluorine are the highly electronegative
values are less for homolytic fission when compared to elements. When they are tied to a hydrogen atom to form covalent
heterolytic fission. bond, the electrons of the covalent bond are shifted towards
the more electronegative atom. This partially positive charged
hydrogen atom forms a bond with the other more electronegative
BOND ANGLE atom. This bond is called as hydrogen bond and is weaker than
● Angle between the orbitals containing bonding pair of covalent bond.
electrons around the central atom in a molecule or complex Hydrogen bonding is said to be formed when slightly acidic
ion is called bond angle. hydrogen attached to a strongly electronegative atom such as F,
● Bond angle is determined by spectroscopic methods. N and O is held with weak electrostatic forces by the lone pair of
electrons of the electronegative atom.
● As bond angle increases stability increases.
Hydrogen bond is represented by a dotted line (– – –)
Bond angle is effected by the presence of lone pair of electrons on
For example, in HF molecule, the hydrogen bond exists between
central atom
hydrogen atom of one molecule and fluorine atom of another
Ex: CH4 → 109°28' → [0 L.P] molecule as given below :
NH3 → 107° → [1 L.P]    – – – Hδ+ – Fδ–– – – – Hδ+ – Fδ–– – – – Hδ+ – Fδ–
H2O → 104.5° → [2 L.P]
Hydrogen bond is weaker than covalent bond and stronger than
Bond angle depends on state of hybridisation. van der waal force of attraction.
If the electronegativity of central atom decreases bond angle Hydrogen bond present in the same molecule is known as intra
decreases. molecular hydrogen bond.
In case, the central atom remains same bond angle increases with Due to intramolecular hydrogen bonding, ring formation or
chelation occurs.
the decrease in electronegativity of surrounding atoms.
Substances having intramolecular hydrogen bonds are less water
% s-character increases bond angle increases. soluble and steam volatile and have low boiling points.
Chemical Bonding and Molecular Structure 81
Hydrogen bond formed between different polar molecules is In water molecules: Due to polar nature of H2O there is
known as inter molecular hydrogen bond. association of water molecules giving a liquid state of abnormally
Order of H-bond strength high boiling point.

There is also similar H-bonding in alcohol (R—OH) ammonia


(NH3) and phenol (C6H5OH) molecules.
Types of Hydrogen Bonds The hydrogen bonds in HF link the F atom of one molecule with
the H-atom of another molecule, thus forming a zig-zag chain
(A) Intramolecular H-Bonding: This type of H-bonding
(HF)n in both the solid and also in the liquid.
occurs when polar H and electronegative atom are present
in the same molecule.

Carboxylic acid dimerises in gaseous state due to H-bonding

It has lower boiling point (i.e. more volatile) than its para-
derivative (where association of molecules takes place using
intermolecular H-bonding) because it exists as discrete molecules.
Alcohol is said to be highly soluble in water due to crossed
intermolecular H-bonding (between H2O and R—OH molecules).

However, isomeric ether is less soluble in water due to less polar


nature of ether.

Extra stability of the complex is because of intramolecular
hydrogen bonding in addition to the chelating effect. HCO3– ions exist as dimer in KHCO3
Necessary conditions for the formation of intramolecular
hydrogen-bonding In Na+HCO3–, the HCO3– ions are linked to an infinite chain through
(a) The ring formed as a result of hydrogen bonding should be intermolecular H-bonding.
planar.
(b) 5- or 6- membered ring should be formed.  KEY NOTE
(c) Interacting atoms should be placed in such a way that there Š The strongest H-bonds are formed by F atoms. Deuterium is
is minimum strain during the ring closure. more electropositive than H, therefore it also forms stronger
(B) Intermolecular H-Bonding: It is formed between two bonds.
different molecules of the same or different compounds.

82 Dropper NEET
Topicwise Questions

KOSSEL LEWIS THEORY & LEWIS DOT 14. In an ionic compound A+X–, the degree of covalent bonding
STRUCTURE is greater when
(a) A+ and X– ions are small
1. Metal ‘M’ forms a peroxide of the type MO2. Valency of the
(b) A+ is small and X– is large
metal with respect to oxygen
(c) A+ and X– ions are approximately of the same size
(a) 0 (b) 1 (c) 2 (d) 4
(d) X- is small and A+ is large
2. Valency of the metal atom with respect to oxygen is maximum in
15. Which of the following has highest ionic character?
(a) Mn2O7 (b) OsO4
(a) MgCl2 (b) CaCl2
(c) MnO2 (d) CrO3
(c) BaCl2 (d) BeCl2
3. Variable valency is a property of
16. In which of the following molecule, the central atom has
(a) Alkali metals (b) Transition metals
three lone pairs of electrons ?
(c) Alkaline earth metals (d) Inert gases
(a) Ammonia (b) Xenon difluoride
4. The molecule that deviates from octet rule is (c) Chlorine trifluoride (d) Hydrogen sulphide
(a) NaCl (b) BeCl2 17. Number of lone pairs of electrons in 9 gms. of water are
(c) MgO (d) NH3 [N = Avogadro Number]
5. Which of the following molecule deviates from octet rule (a) 2N (b) N / 2
with respect to central atom? (c) N (d) N / 4
(a) PCl3 (b) H2S (c) NH3 (d) XeF4 18. The number of electron pairs involved in the formation of
6. Expanded octet can be observed in the valency shell of the hydrogen cyanide molecule are
central atom in (a) Two (b) Eight
(a) NH3 (b) CH4 (c) PCl5 (d) BeCl2 (c) Three (d) Four
7. How many number of species are hypovalent ? 19. The number of π-bonds in the structure given below are:
BeCl2, BCl3, CCl4, PCl3, O2, H2 (NC)2 C = C (CN)2
(a) 2 (b) 3 (c) 4 (d) 5 (a) 9 (b) 7 (c) 5 (d) 2
8. Which of the following species will be the strongest Lewis 20. The molecule MLx is planar with six pair of electrons around
acid ? M in the valence shell. The value of x is
(a) Fe0 (b) Fe3+ (c) Fe2+ (d) Fe+ (a) 6 (b) 2 (c) 4 (d) 3
9. Which of following molecule/specie is having maximum 21. The electronegativities of two elements are 0.7 and 3.0. The
number of lone pairs in Lewis - dot structure ? bond formed between them would be
(a) BH −4 (b) BF4− (a) Ionic (b) Covalent
(c) Co-ordinate covalent (d) Metallic
(c) CN¯ (d) COCl2
22. Highly ionic compound is formed by the combination of
10. Which of the following species follows the octet rule for all
elements belonging to
of its atoms ?
(a) I A group and VII A gruop
(a) BeH2‑dimer (b) TeF6
(b) II A group VI A group
(c) XeF6 (d) C2F6
(c) III A group and V A group
11. What is the formal charge on O-atom in carbonate ion.
(d) ‘O’ group and VII A group
(a) –1 (b) 0
(c) +1 (d) – 1 & 0 both 23. An electrovalent compound is made up of
(a) Electrically charged particles
IONIC OR ELECTROVALENT/COVALENT BOND (b) Neutral molecules
12. Molecule having maximum number of covalent bonds is (c) Neutral atoms
(a) NH4OH (b) NH4Cl (d) Electrically charged atom or group of atoms
(c) CO(NH2)2 (d) CH3OH 24. Potassium forms a highly ionic compound when it combines
13. Number of bonded electrons in ethane molecule are with
(a) 7 (b) 12 (a) Chlorine (b) Fluorine
(c) 10 (d) 14 (c) Bromine (d) Iodine
Chemical Bonding and Molecular Structure 83
25. The number of sp2- sigma bonds in benzene are: 38. Which of the following species has bond angle less than
(a) 3 (b) 6 98°?
(c) 12 (d) None of these (a) H2O (b) NH3
(c) CH4 (d) PH3
26. The strength of the bonds by 2s-2s, 2p-2p and 2p-2s overlap
has the order: 39. Find out the relation between (adjacent angle)
(a) s-s > p-p > p-s (b) s-s > p-s > p-p  and ÐFBrF
ÐFClF  bond angle in ClF and BrF molecule
3 3
(c) p-p > p-s > s-s (d) p-p > s-s > p-s respectively.
 = FBrF
(a) FClF   > FBrF
(b) FClF 
27. Most ionic compound among the following is
(a) Sodium fluoride (b) Sodium Chloride  < FBrF
(c) FClF  (d) Can’t predicted
(c) Sodium bromide (d) Sodium iodide 40. The number of P–P–P angle in P4 structure is:
28. Which of the following has least polarity in bond? (a) 6 (b) 9
(a) H – F (b) H – Cl (c) 12 (d) 10
(c) H – O (d) H – S 41. The bond angle and hybridization in ether (CH3OCH3) is :
29. Which contains both polar and non-polar bonds ? (a) 106o51’, sp3 (b) 104o31’, sp3
(a) NH4Cl (b) HCN (c) > 109o 28’ sp3 (d) None of these
(c) H2O2 (d) CH4 42. The bond angle in H2S is:
30. Which of the following is non-polar? (a) > NH3 (b) Same as in BeCl2
(a) H2S (b) NaCl (c) > H2Se, < H2O (d) Same as in CH4
(c) Cl2 (d) H2SO4 43. The smallest bond angle around the central atom will be
31. Which of the following unit conversion of dipole moment is present in:
correct? (a) H2O (b) BeF2
(a) 1D = 3.3356 Cm (c) CH4 (d) NH3
(b) 1D= 3.3365 × 10–30 Cm 44. The bond length of the S–O bond is maximum in which of
(c) 1 C m = 3.3356 D the following compound ?
(d) 1 C m = 3.3356 × 10–30 D SOBr2, SOCl2, SOF2
32. The compound which does not contain ionic bond is: (a) SOCl2 (b) SOBr2
(a) NaOH (b) HCl (c) SOF2 (d) All have same length
(c) K2S (d) LiH 45. Which of the angle is most influenced by the presence of
33. Which of the following chloride has considerable covalent lone pair in SF4 ?
character? (a) Equatorial angle
(a) LiCl (b) NaCl (b) Axial angle
(c) KCl (d) CsCl (c) Angle between axial and equatorial
(d) None
34. Which of the following hydrocarbons has the lowest dipole
moment? 46. Correct order of bond angle in CH2F2:
CH3 H
ˆ H > HCF
(a) HC ˆ > FCF
ˆ ˆ > HCF
(b) FCF ˆ > HCH
ˆ
(a) C=C (b) CH3C ≡ CCH3 ˆ > FCFˆ > HCF ˆ ˆ > FCF ˆ > HCH ˆ
H CH3 (c) HCH (d) HCF
(c) CH3CH2C ≡ CH (d) CH2 = CH – C ≡ CH 47. Which of the following molecule has shortest C-H bond ?
(a) Ethene (b) Ethane
BOND PARAMETERS (c) Ethyne (d) Methane
35. Which of the following is correct order of bond length ?
48. The correct sequence of decrease in the bond angles of the
(a) BF4– < BF3 (b) NO2+ < NO2– following hydrides is:
(c) CCl4 < CF4 (d) +CH3 > CH4 (a) NH3 > PH3 > AsH3 > SbH3
36. Indicate the nature of bonding in diamond. (b) NH3 > AsH3 > PH3 > SbH3
(a) Ionic (b) Covalent (c) SbH3 > AsH3 > PH3 > NH3
­
(c) Molecular (d) Metallic (d) PH3 > NH3 > AsH3 > SbH3
37. In which of the following species, the angle around the 49. Find the maximum number of identical angle in CH2F2
central atom is exactly equal to 109°28ʹ? molecule.
(a) SF4 (b) NH3 (a) 2 (b) 4
(c) NH+4 (d) None of these (c) 6 (d) 8
84 Dropper NEET
50. In which of the following bond angle is maximum: 63. The species/molecule is having same shape and different
(a) NH3 (b) NH4+ hybridisation:
(c) PCl3 (d) SCl2 (a) XeF2, CO2 (b) I3–, H3O+

(c) OCl , CO (d) SO2, OCl2
51. Which of the following H-bonds is expected to have
maximum strength? 64. Which of the following tetra atomic species is non-polar and
(a) H-O----H (b) H-N----H planar?
(c) H-S-----H (d) All have same strength (a) BrF4– (b) NH4+
(c) ClF3 (d) BF3
52. Which one of the following compounds has the smallest
bond angle in its molecule? 65. Which of the possible molecule / species is having maximum
(a) OH2 (b) SH2 (c) NH3 (d) SO2 values for dipole moment. (where “A” is the central atom)?
(a) AX3 (having one lone pair on central atom)
53. In O3, there are:
(b) AX4 (Tetrahedral)
(a) 2σ, 1π bond (b) 1σ, 2π bonds
(c) AX4Y (having no lone pair on central atom)
(c) 2σ, 2π bonds (d) 2σ, 1π bond
(d) Can’t be predicted
54. Which one of the following has the smallest bond angle?
66. Which is the right structure of XeF4 ?
(a) NH3 (b) BeF2
(c) H2O (d) CH4
55. The correct order of bond angles (smallest first) in , H2S,
NH3, BF3 and SiH4 is:
(a) (b)
(a) H2S < NH3 < SiH4 < BF3
(b) NH3 < H2S < SiH4 < BF3
(c) H2S < SiH4 < NH3 < BF3
(d) H2S < NH3 < BF3 < SiH4
56. Which one of the following bonds has the highest average
bond energy? (c) (d)
(a) S = O (b) C ≡ C
(c) C ≡ N (d) N ≡ N
67. The shape of SO42 – ion is:
57. The correct decreasing order of bond angles is:
(a) Square planar (b) Square Pyramidal
(a) ClF3 > PF3 > NF3 > BF3
(c) Tetrahedral (d) None of these
(b) BF3 > PF3 > NF3 > ClF3
(c) BF3 > NF3 > PF3 > ClF3 68. Which of the following species given below have shape
similar to XeOF4 ?
(d) BF3 > ClF3 > PF3 > NF3
(a) XeO3 (b) ΙOF4+
58. Bond angle in water is:
(c) PCl5 (d) XeF5+
(a) 120° (b) 109.5°
(c) 107° (d) 104.5° 69. The total number of lone pairs of electrons in NO3– :
(a) 1 (b) 2
59. The correct order in which the O–O bond length increases in
(c) 6 (d) 8
the following :
(a) H2O2 < O2 < O3 (b) O3 < H2O2 < O2 70. The central atom in IF7 has seven pairs of valency electrons.
The shape of the molecule as per VSEPR theory is:
(c) O2 < H2O2 < O3 (d) O2 < O3 < H2O2
(a) Trigonal bipyramid
VSEPR THEORY (b) Hexagonal pyramid
60. Cyanogen, (CN)2, has a ___ shape/structure: (c) Pentagonal bipyramid
(a) Linear (b) Zig-zag (d) Square bipyramid
(c) V‑shape (d) Cyclic 71. The correct order of decreasing polarity is:
61. The shape of covalent molecule AX3 is: (a) HF > SO2 > H2O > NH3
(a) Triangular (b) T-shape (b) HF > H2O > SO2 > NH3
(c) Pyramidal (d) Any of the above (c) HF > NH3 > SO2 > H2O
62. Of the three molecules XeF4, SF4, SiF4, one which have (d) H2O > NH3 > SO2 > HF
tetrahedral structure is ? 72. Which of the following is a pyramidal species?
(a) All the three (b) Only SiF4 (a) CO2 (b) H3O+
(c) Both SF4 & XeF4 (d) Only SF4 & XeF4 (c) SnCl2 (d) NH4+
Chemical Bonding and Molecular Structure 85
73. In CCl4 the four valencies of carbon are directed towards the VALENCE BOND THEORY & HYBRIDISATION
corners of a
86. Which of the following molecule is having shortest bond
(a) Cube (b) Hexagon length of C–O bond ?
(c) Prism (d) Tetrahedron (a) CH3OH (b) H2CO
74. The shape of CO2 molecule is similar to: (c) CO (d) Na2CO3
(a) H2O (b) BeF2 87. The ratio of σ and π bonds in benzene is:
(c) SO2 (d) None of these (a) 2 (b) 6
75. Out of CHCl3, CH4 and SF4, the molecules having regular (c) 4 (d) 8
geometry are:
88. A hybrid orbital formed from s and p- orbital can contribute to
(a) CHCl3 only (b) CHCl3 and SF4
(a) σ- bond only (b) π-bond only
(c) CH4 only (d) CH4 and SF4
(c) Either σ or π-bond (d) Cannot be predicted
76. In OF2 molecule, the total number of bond pairs and lone
89. The hybrid state of C in CS2 should be:
pairs of electrons are respectively:
(a) sp2 (b) sp
(a) 2, 6 (b) 2, 8
(c) sp3 (d) Not specific
(c) 2, 10 (d) 2, 9
90. Indicate the wrong statement according to Valence bond theory:
77. Which one of the following has the regular tetrahedral structure?
(a) A sigma bond is stronger than π-bond
(a) XeF4 (b) [Ni(CN)4]2–
(b) p-orbitals always have only sidewise overlapping
(c) BF–4 (d) SF4
(c) s-orbitals never form π-bonds
78. The shape of formaldehyde molecule as per the VSEPR (d) There can be only one sigma bond between two atoms
theory is:
91. The strongest P–O bond is found in the molecule
(a) Linear (b) Planar triangle
(a) F3PO (b) Cl3PO
(c) Pyramid (d) Tetrahedron
(c) Br3PO (d) (CH3)3PO
79. Hybrid orbital with least s - character is:
92. In pent-3-en-1-yne the terminal carbon-atoms have following
(a) sp3d (b) sp2
hybridisation:
(c) sp 3 (d) sp
(a) sp & sp2 (b) sp2 & sp3
80. The number of hybrid orbitals in a molecule of decane are: (c) sp2 & sp (d) sp & sp3
(a) 36 (b) 40 93. During the complete combustion of methane CH4, what
(c) 38 (d) 8 change in hybridisation does the carbon atom undergo?
81. The molecule which contains σsp3–sp3 and σsp3–p bonds in it is: (a) sp3 to sp (b) sp3 to sp2
(a) CH3CH3 (c) sp2 to sp (d) sp2 to sp3
(b) CH3CHO 94. Which orbital is used by oxygen atom to form a sigma bond
(c) CH3CH2Cl with other oxygen atom in O2 molecule?
(d) CHCl3 (a) Pure p-orbital (b) sp2 - hybrid orbital
82. Bond angle between two hybrid orbitals is 107o, s - orbital
3
(c) sp - hybrid orbital (d) sp - hybrid orbital
character of hybrid orbital is nearly 95. Which of the following compounds have the same no. of
(a) 50 % (b) 33.33% lone pairs with their central atom ?
(c) 16.6 % (d) 25 %
[I] XeF5− [II] BrF3 [III] XeF2
83. Carbon atoms in C2(CN)4 are: [IV] H3 S + [V] Triplet Methylene
(a) sp hybridized (a) IV and V (b) I and III
(b) sp2 Hybridized (c) I and II (d) II, IV and V
(c) sp and sp2 hybridized 96. The correct order of hybridization of the central atom in the
(d) sp,sp2 and sp3 hybridized following species :
84. The interparticle forces in liquid hydrogen are. NH3, XeO2F2, SeF4, NO2+
(a) H-bonds (a) sp3, sp3, sp3d, sp (b) sp3, sp3d, sp3d, sp
(b) Vander waal's forces 3 3 2 3
(c) sp , sp d , sp d, sp 2 (d) sp2, sp3d, sp3d2, sp
(c) Covalent bonds 97. The hybridisation of the central atom will change when
(d) None of these (a) NH3 combines with H+
85. The number of sigma and Pi bonds in a molecule of cyanogen are: (b) H3BO3 combines with OH–
(a) 4, 3 (b) 3, 4 (c) NH3 forms NH −2
(c) 5, 2 (d) 3, 5 (d) H2O combines with H+
86 Dropper NEET
98. What is the state of hybridisation of anionic part of solid 110. pi bond is formed by the overlapping of
N2O5 ? (a) p - p orbitals along their axis
(a) sp (b) sp2 (b) s - p orbitals along the axis of p - orbital
(c) sp 3 (d) Not applicable (c) p - p orbitals perpendicular to their axis
99. Which one of the following pairs is isostructural? (d) s - s orbitals
(a) [BCl3 and BrCl3] (b) [NH3 and NO3– ] 111. Which of the following is not correct ?
(c) [ NF3 and BF3 ] (d) [BF4- and NH +4 ] (a) A sigma bond is weaker than pi bond
100. The hybridisation of C‑atoms in tetracyanomethane is (b) A sigma bond is stronger than pi bond
(a) sp, sp2 (b) sp3, sp (c) A double bond is stronger than a single bond
3
(c) sp , sp 3 (d) sp3 , sp2 (d) A double bond between two atoms is shorter than a
101. BF3 + F– → BF4– single bond between the same atoms.
What is the hybridiation state of B in BF3 and BF4– ? 112. The s - character in the hybrid orbital of the central atom,
(a) sp2, sp3 (b) sp3, sp3 present in a molecule having the shape of an octahedron is
2
(c) sp , sp 2 (d) sp3, sp3d (a) 25 % (b) 75 %
(c) 40 % (d) 16.66 %
102. Choose the molecules in which hybridisation occurs in the
ground state ? 113. The C-H bond in propane is
(i) BCl3 (ii) NH3 (a) σsp - s (b) σsp2–s (c) σp - s
(d) σsp3–s
(iii)PCl3 (iv) BeF2 114. The hybridisation of P in phosphate ion (PO43 –) is the same

The correct answer is as:
(a) i, ii, iv (b) i, ii, iii (a) Ι in ΙCl4– (b) S in SO3
(c) ii, iii (d) iii, iv (c) N in NO3– (d) S in SO32 –

103. The geometry of electron pairs around I in IF5 is: 115. Number of hybrid orbitals present in a molecule of propene
(a) Octahedral (b) Trigonal Bipyramidal are
(c) Square Pyramidal (d) Pentagonal Planar (a) 12 (b) 10 (c) 9 (d) 8
104. sp3d hybridization is considered to be a combination of two 116. Molecule obtained by sp3d2 hybridisation has bond angle of
hybridization. They are (a) 90o (b) 109o28'
(a) p3 + sd (b) sp2 + pd (c) 120 o
(d) 72o
(c) spd + p2 (d) None of these 117. Hybrid orbital having maximum p - character is
105. All the four sigma bonds in perchlorate ion are formed by (a) sp3d (b) sp3
which orbital? (c) sp (d) sp3d2
(a) sp3d3 (b) sp3d2 118. Hybridisation involves
(c) sp3d (d) sp3 (a) Addition of an electron pair
106. Which one of the following conversions involve change in (b) Combination and redistribution of atomic orbitals
both hybridisation and shape?
(c) Removal of an electron pair
(a) CH4 → C2H6 (b) NH3 → NH+4
(d) Separation of orbitals
(c) BF3 → BF4 – (d) H2O → H3O+
119. On hybridisation of one ‘s’ and one ‘p’ orbitals we get
107. Which of the following penta atomic species is tetrahedral
(a) Two mutually perpendicular orbitals
and non polar ?
+
(b) Two orbitals at 180o
(a) CH3F (b) NH 2 F2 (c) Four orbitals directed tetrahedrally
(c) [Ni(CO)4] (d) BF4− (d) Three orbitals in a plane
108. The orbital overlapping is maximum in 120. As the s - character in a hybrid orbital increases, the bond
(a) Cl2 (b) HI angle
(c) HCl (d) HBr (a) Increases (b) Decreases
(c) Does not change (d) Becomes zero
109. Linear combination of two hybridised orbitals belonging to
two atoms and each having one electron leads to a 121. sp3d hybridisation results in
(a) Sigma bond (a) A square planar molecule
(b) Double bond (b) An octahedron molecule
(c) Co-ordinate covalent bond (c) A trigonal bipyramid molecule
(d) pi bond (d) A tetrahedron molecule

Chemical Bonding and Molecular Structure 87


122. s - p overlapping is present in 134. On catalytic hydrogenation, ethylene gives ethane during
(a) Br2 (b) H2 this reaction
(c) O2 (d) HF (a) Hybridization of carbon atoms changes from sp2 to sp3
123. Valence bond theory of Pauling and Slater accounts for the (b) bond angle decreases from1200 to 109.50
following characteristic of covalent bond (c) C-C bond length increases from1.34 A0 to 1.54A0
(a) Directional (b) ionic (d) All of these.
(c) Strength (d) hybrid 135. The ratio of pure orbitals to hybridized orbitals in ethylene is
124. Shape of molecule is decided by (a) 2 : 3 (b) 3 : 1
(a) Sigma bonds (b) pi bonds (c) 1 : 1 (d) 1 : 3
(c) Both sigma and pi bonds (d) Neither sigma nor pi bonds
MOLECULAR ORBITAL THEORY
125. The strength of bonds by 2s-2s, 2p-2p, 2s-2p, 1s-1s overlap
has the order: 136. Which of the following species has lowest ionization
(a) 1s-1s > 2p-2p > 2s-2p > 2s-2s potential?
2−
(b) 2p-2p > 2s-2p > 2s-2s > 1s-1s (a) O2 (b) O2
(c) 2s-2s > 1s-1s > 2s-2p > 2p-2p (c) O2+ (d) O2–
(d) 2s-2p > 2s-2s > 2p-2p > 1s-1s 137. Among KO2, AlO2– , BaO2 & NO2+ unpaired electron is
126. The hybridization of atomic orbitals of nitrogen in NO2+ , present in:
NO3– and NH4+ are (a) NO2+ and BaO2 (b) KO2 and AlO2–
(a) sp2, sp3 and sp2 respectively (c) KO2 only (d) BaO2 only
(b) sp, sp2 and sp3 respectively 138. What is correct sequence of bond order ?
(c) sp2, sp and sp3 respectively (a) O2+ > O2– > O2 (b) O2+ > O2 > O2–
(d) sp2, sp3 and sp respectively (c) O2 > O2– > O2+ (d) O2– > O2+ > O2
127. According to valence bond theory the predicted bond angle 139. Which statement is correct about O2+ ?
for H2O
(a) Paramagnetic and bond order < O2
(a) 90° (b) 109°28′
(c) 107°18′ (d) 104°28′ (b) Paramagnetic and bond order > O2

128. Which one of the following is a correct set? (c) Diamagnetic and bond order < O2
(a) H2O,sp3,angular (b) H2O,sp2, linear (d) Diamagnetic and bond order > O2
(c) NH4 ,dsp , square planar (d) CH4,dsp2, tetrahedral
+ 2 140. In which of the following ionization processes, the bond
order has increased and the magnetic behaviour has changed?
129. Which one of the following is the correct set with reference (a) C2 → C2+ (b) NO → NO+
to molecular formula, hybridisation of central atom and (c) O2 → O2 + (d) N2 → N2+
shape of the molecule?
141. The molecular electronic configuration of H2– ion is
(a) CO2,sp2, bent (b) H2O, sp2, bent
(a) (σ1s)2 (b) (σ1s)2(σ*1s)2
(c) BeCl2, sp, linear (d) H2O, sp3, linear 2 1
(c) (σ1s) (σ*1s) (d) (σ1s)3
130. What is the hybridisation state of the central atom in the
142. Which one of the following molecules contains no π-bond?
conjugate base of NH4+ ion?
(a) CO2 (b) H2O
(a) sp (b) sp3 (c) SO2 (d) NO2
(c) sp 2 (d) dsp2
143. Which of the following molecules/ions does not contain
131. For which hybridization, there are two unequal bond angles unpaired electrons?
(a) sp3 (b) sp2 (a) O 22 − (b) B2
(c) sp (d) sp3d
(c) N-2 (d) O2
132. The shape of CH4; SO4-2; PO4-3 is
144. Anti- bonding molecular orbital is formed by
(a) Trigonal planar
(a) Addition of wave functions of atomic orbitals
(c) Angular
(b) Substraction of wave functions of atomic orbitals
(c) Tetrahedral
(c) Multiplication of wave functions of atomic orbitals
(d) Trigonal Bipyramidal
(d) Finding the arithmetic mean
133. The compound in which carbon uses sp3 hybridisation for
145. Which one of the following species does not exist under
bond formation
normal conditions?
(a) H-COOH (b) (NH2)2C=O
(a) Li2 (b) Be +2
(c) H-CHO (d) CH3CH2 OH (c) Be2 (d) B2
88 Dropper NEET
146. Oxygen molecule is HYDROGEN BONDING
(a) Diamagnetic with no unpaired electrons 160. Which compound has electrovalent, covalent, coordinate as
(b) Diamagnetic with two unpaired electrons well as hydrogen bond ?
(c) Paramagnetic with two unpaired electrons
(a) NH4Cl (b) HNO3
(d) Paramagnetic with no unpaired electrons
(c) CuSO4.5H2O (d) BeCl2.2H2O
147. The species having bond order different from that in CO is
(a) NO– (b) NO+ 161. Which of the following is the correct order of strength of
– H‑bonding in the given compound ?
(c) CN (d) N2
(a) HF < NH3 (b) H2O > H2O2
148. Which one of the following has the strong O-O bond?
(c) H2O2 > H2O (d) NH3 > H2O
(a) O +2 (b) O2 162. Strongest hydrogen bond is present in
(c) O-2 (d) O 22- (a) Ammonia (b) Water
(c) Hydrogen fluoride (d) Ethyl alcohol
149. In which of the following pairs, the two species have identical
bond order? 163. Intermolecular hydrogen bond is not present in
(a) N-2 , O 22- (b) N-2 , O-2 (a) Ammonia (b) Water
(c) Hydrofluoric acid (d) Salicylaldehyde
(c) N-2 , O+2 (d) O +2 , N 22-
164. Which of the following exists as a liquid at room temperature
due to the formation of associated molecules only?
150. How may bonds does B2 have?
(a) Benzene (b) Ammonia
(a) 0 (b) 1
(c) Bromine (d) Carbon disulphide
(c) 2 (d) 3
165. Which of the following species has intramolecular H-bonds?
151. According to MOT, C2 molecule has
(a) Phenol (b) o-Nitrophenol
(a) 1σ & 1π bond (b) Only 2π-bond
(c) p-Nitrophenol (d) Nitroethane
(c) Only 2σ-bond (d) 1σ and 2π bond
166. Bond energy of covalent O - H bond in water is
152. Which of the following is diamagnetic?
(a) Greater than bond energy of hydrogen bond
(a) H + (b) O2
2 (b) Equal to bond energy of hydrogen bond
(c) Li2 (d) He +2
(c) Less than bond energy of hydrogen bond
153. Molecular orbital theory was given by: (d) Half of the bond energy of hydrogen bond
(a) Kossel (b) Moseley
167. Water is a liquid while hydrogen sulphide is a gas because
(c) Mulliken (d) Werner
(a) Water has higher molecular weight
154. Molecular orbital electronic configuration for ʻXʼ anion is- (b) Hydrogen sulphide is a weak acid
KK σ*2s2 σ*2s2 π 2px2 π 2py2 σ2pz2 π* 2px1, the anion ʻXʼ is
(c) Sulphur has high electronegativity than oxygen
(a) N-2 (b) O-2 (d) Water molecules associate through hydrogen bonding
(c) N 22- (d) O 22 − 168. Which of the following is a normal liquid ?
155. The number of nodal planes present in σ*2s anti- bonding (a) NH3 (b) H2O
orbital is (c) HF (d) Br2
(a) 1 (b) 2 169. Among the following, the boiling point is high for
(c) 0 (d) 3 (a) Ethyl alcohol (b) Dimethyl ether
156. Which of the following is paramagnetic with bond order 0.5? (c) Acetone (d) Chloroform
(a) F2 (b) H +2 170. Strongest hydrogen bond is present in
(a) CH3OH (b) CHCl3
(c) N2 (d) O-2
(c) (CH3)2CHOH (d) (CH3)3COH
157. Which of the following pairs have identical bond order?
171. Which of the following is miscible with water ?
(a) N 2 , O 22 + (b) N 2 , O 2−
(a) CS2 (b) C2H5OH
(c) N −2 , O 2 (d) O 2 + , N 2 (c) CCl4 (d) CHCl3

158. The bond order of O2 is: 172. The coupling between base units of DNA is through
(a) 0.5 (b) 1.5 (a) Hydrogen bonding (b) Electrostatic bonding
(c) 3.5 (d) 2.5 (c) Covalent bonding (d) Vander Waal’s forces
159. The molecule having bond order 3 is 173. Which of the following is steam volatile ?
(a) H2 (b) N2 (a) Phenol (b) o - Nitrophenol
(c) O2 (d) He2+ (c) m - Nitrophenol (d) p - Nitrophenol
Chemical Bonding and Molecular Structure 89
174. Which of the following compounds would show evidence of 184. H-bond is not present in:
the strongest hydrogen bonding ? (a) Water
(a) Propan-l-ol (b) Propan-2-ol (b) Glycerol
(c) Propan-l,2-diol (d) Propan-l,2,3-triol (c) Hydrogen fluoride
175. The compound having hydrogen bonds in it is (d) Hydrogen sulphide
(a) NH3 (b) H2S 185. Which of the following has strongest hydrogen bonding?
(c) HCl (d) PH3 (a) Ethyl amine (b) Ammonia
176. Strength of hydrogen bond is intermediate between (c) Ethyl alcohol (d) Diethyl Ether
(a) Vander Waal forces and covalent bond 186. The high boiling point of water is due to-
(b) Ionic bond and covalent bond (a) Co-ordinate bonding
(c) Ionic bond and metallic bond (b) Covalent bonding
(d) Resonance (c) Electrostatic force of attraction
(d) Hydrogen Bonding
177. NH3 has a much higher boiling point than PH3 because
(a) NH3 has a larger molecular weight 187. When two ice cubes are pressed over each other, they unite to
form one cube. Which of the following forces is responsible
(b) NH3 undergoes umbrella inversion
to hold them together?
(c) NH3 contains hydrogen bonds
(a) Dipole forces
(d) NH3 contains ionic bonds where as PH3 contains covalent
(b) Vander waal's forces
bonds
(c) Covalent forces
178. The maximum number of molecules that one water molecule
(d) Hydrogen bond forces
can hold through hydrogen bonding is
(a) 2 (b) 4 188. Hydrogen bonding is maximum in:
(c) 6 (d) 8 (a) Ethyl chloride (b) Triethyl amine
(c) Ethanol (d) Diethyl ether
179. The high density of water compared to ice is due to
(a) H-bonding interactions 189. What is the dominant intermolecular force or bond that must
(b) Dipole-dipole interactions be overcome in converting liquid CH3OH to a gas?
(c) Dipole-induced dipole interactions (a) Hydrogen bonding
(d) Induced dipole-induced dipole interactions (b) Dipole-dipole force
(c) Covalent bond
180. The force responsible for the union of two ice blocks as a
single block is (d) London dispersion force
(a) Vanderwaals force 190. Which of the following is least volatile?
(b) Hydrogen bonds (a) H2O (b) H2S
(c) dipole interaction (c) H2Se (d) H2Te
(d) vanderwaals repulsion 191. The hydrogen bonding is maximum strong in:
181. The Electronegative elements between which the H-bond is (a) S-H-----S (b) N-H-----O
formed principally in aqueous ammonia (c) S-H-----O (d) F-H------O
(a) O, O (b) N,N 192. The comparatively high boiling point of HF is due to
(c) O, N (d) Both O,O & N,N (a) High reactivity of fluorine
182. Which of the following form hydrogen bond ? (b) Small size of hydrogen atom
(a) CH3F (b) CH3Cl (c) Formation of hydrogen bonds and consequent association
(c) CH3Br (d) None (d) High IE of fluorine
183. Chelation is observed in 193. Intramolecular H-bonding is present in:
(a) P-nitrophenol (b) O-nitrophenol (a) Meta- nitrophenol (b) Salicyaldehyde
(c) Benzaldehyde (d) HF (c) HCl (d) Benzophenone

90 Dropper NEET
Learning Plus
1. The correct order of hybridization of the central atom in the 8. Among the triatomic molecules/ions BeCl2 , XeF2, ICl2+,
following species NH3, PCl5 and BCl3 is I3+, I(CN)2–
(a) sp3d, sp2 and sp3 (b) sp3 , sp3d, sp2 How many are sp3d hybridised and linear choose the
correct pairs ?
(c) sp2, sp3, sp3d (d) sp3, sp2, sp3d
(a) BeCl2 & XeF2 (b) XeF2 & I(CN)2–
2. How many molecule which has sp3d hybridisation but planar
as well as non-polar in structure? (c) XeF2 & ICl2+ (d) BeCl2 & I3+

[ICl2]–, BrF5, SOF4, SF6, XeOF4 , ClF3, I3 , [I(CN)2]–, PCl5 9. In PO43– ion the average formal charge on the oxygen
(a) 2 (b) 4 atom of P–O bond is:
(c) 6 (d) 8 (a) + 1 (b) –1
(c) – 0.75 (d) + 0.75
3. Polarity in a molecule and hence the dipole moment –
depends primarily on electronegativity of the constituent 10. In NO3 ion, the number of bond pairs and lone pairs of
atoms and shape of a molecule. Which of the following electrons on nitrogen atom are:
has the highest dipole moment? (a) 2, 2 (b) 3, 1
(c) 1, 3 (d) 4, 0
(a) CO2 (b) HI
(c) H2O (d) SO2 11. If the equatorial plane is x–y plane in sp3d hybridisation
then the orbital used in pd hybridisation are -
4. Choose the INCORRECT options.
(a) pz and dz2 (b) px and dxy
(a) All dN-O bond lengths are equal in nitrate ion.
(c) py and dyz (d) None of these
(b) All dC-O bond lengths are equal in sodium carbonate.
12. How many maximum number of atoms are present in one
(c) All dC-O bond lengths are equal in acetic acid. plane in PF2(CH3)3 molecule, only consider those geometry
(d) All dB-O bond length are equal in boric acid. which is non-polar in nature?
5. In a compound

The number of sigma and pi bonds respectively are


(a) 19, 11 (b) 19, 5 (a) 2 (b) 4 (c) 7 (d) 8
(c) 13, 11 (d) 7, 3 13. Which of the following statement is true for IO2F2– ?
6. Hydrogen bonds are formed in many compounds e.g., H2O, (a) The electrons are located at the corners of a trigonal
HF, NH3. The boiling point of such compounds depends to a bipyramidal but one of the equatorial pairs is unshared.
extent on the strength of hydrogen bond and the number of (b) It has sp3d hybridisation and is T–shaped.
hydrogen bonds. The correct decreasing order of the boiling (c) Its structure is analogous to SF4.
points above compounds is: (d) (a) and (c) both
(a) HF > H2O > NH3 (b) H2O > HF > NH3 14. Which of the following species has tetrahedral geometry?
– –
(c) NH3 > HF > H2O (d) NH3 > H2O > HF (a) BH4 (b) NH2
7. Choose the correct statement: (c) CO32 – (d) H3O+
(a) NH3 is having bond angle of 109°28’. 15. Number of π bonds and σ bonds in the following structure
(b) The direction of the dipole moment of NF3 is as shown is:
in the diagram

(c) sp2 hybrid orbital is consisting of 66.67 % ‘p’ character.


(d) None of these (a) 6, 19 (b) 4, 20 (c) 5, 19 (d) 5, 20

Chemical Bonding and Molecular Structure 91


16. In NO2+, N undergoes 27. Which of the following angle corresponds to sp2
(a) sp3 hybridisation (b) sp2 hybridisation hybridization?
(c) sp hybridisation (d) sp2d hybridisation (a) 90° (b) 120°
17. Which of the following should have pyramidal shape ? (c) 180° (d) 109°
(a) [ClOF2]+ (b) ICl3 28. The bond angle in PH3 is :
(c) [BrΙCl]– (d) All of these (a) Much lesser than NH3
18. Among the triatomic molecules/ion BeCl2 , NO2+, ICl2–, XeF2 (b) Equal to that in NH3
, I3–, OCl2 total number of linear molecule(s)/ion (s) are : (c) Much greater than in NH3
(a) 2 (b) 3 (d) Slightly more than in NH3
(c) 4 (d) 5 29. In which of the following sets, all the species are diamagnetic
19. Which molecule/ion out of the following does not contain in nature?
unpaired electrons? (a) [O22–, O22+ , N22– ] (b) [C2, O22–, N22+]
(a) N2+ (b) O2 (c) O22 – (d) B2 (c) [O22–, O2+, O22–] (d) [N2+, O22+, B2]
20. In which of the following molecule/ion all the bonds are not 30. Which of the following order of energies of molecular
equal? orbitals of N2 is correct?

(a) XeF4 (b) BF4 (c) C2H4 (d) SiF4
(a) ( π2p y ) < ( σ2p z ) < ( π* 2p x ) ≈ ( π* 2p y )
21. The geometry of ammonia molecule can be best described
as: (b) ( π2p y ) > ( σ2p z ) > ( π* 2p x ) ≈ ( π* 2p y )
(a) Nitrogen at one vertex of a regular tetrahedron, the other
(c) ( π2p y ) < ( σ2p z ) > ( π 2p x ) ≈ ( π 2p y )
* *
three vertices being occupied by the three hydrogens
(b) Nitrogen at the centre of the tetrahedron, three of the
(d) ( π2p y ) > ( σ2p z ) < ( π* 2p x ) ≈ ( π* 2p y )
vertices being occupied by three hydrogens
(c) Nitrogen at the centre of an equilateral triangle, three 31. Which of the following statement is not correct from the
corners being occupied by three hydrogens view point molecular orbital theory?
(d) Nitrogen at the junction of a T, three open ends being (a) Be2 is not a stable molecule
occupied by three hydrogens (b) He2 is not stable but He2+ is expected to exist.
22. Which molecular geometry is least likely to result from a (c) Bond strength of N2 is maximum amongst the
trigonal bypyramidal electron geometry? homonuclear diatomic molecules belonging to the
(a) Trigonal planar (b) see–saw second period.
(c) Linear (d) T–shaped (d) The order of energies of molecular orbitals in N2
2 2 2 2 2 2
molecule is σ1s σ *1s , σ2s , σ * 2s , σ2p z , π2p x =π2p 2y
23. Which of the following has bond angle greater than 120°?
32. Among KO2, Al2O3 and BaO2, the unpaired electron is
(a) OCl2 (b) O(CH3)2
present in:
(c) O(SiH3)2 (d) NO2–
(a) KO2 only (b) KO2, BaO2
24. The correct order of bond angle is :
(c) Only Al2O3 (d) Only BaO2
(a) H2S < NH3 < BF3 < CH4
33. The fluoride whose value of dipole moment is not equal to
(b) NH3 < H2S < CH4 < BF3
zero, is:
(c) H2S < NH3 < CH4 < BF3
(a) XeF4 (b) CF4
(d) H2S < CH4 < NH3 < BF3
(c) SF4 (d) PF5
25. Consider the following molecules ;
34. In which of the following, d-orbitals are not used by central
H2O H2S H2Se H2 Te atom in hybridisation?
I ΙΙ ΙΙΙ ΙV (a) PF5 (b) PCl5
Arrange these molecules in increasing order of bond angles. (c) PBr5 (gaseous) (d) None of these
(a) Ι < ΙΙ < ΙΙΙ < ΙV (b) IV < III < II < I 35. BrO3− is isostructural with which of the following ?
(c) Ι < ΙΙ < ΙV < ΙΙΙ (d) ΙΙ < ΙV < ΙΙΙ < Ι
(a) XeO3 (b) XeF3
26. If the electronic configuration of an element is 1s2 2s2 2p6
3s2 3p6 3d2 4s2, the four electrons involved in chemical bond (c) XeF4 (d) XeO2
formation will be: 36. In which of the following compounds B–F length is shortest ?
(a) 3p6 (b) 3p6, 4s2 (a) BF4– (b) BF3 → NH3
(c) 3p6, 3d2 (d) 3d2, 4s2 (c) BF3 (d) BF3 → N(CH3)3

92 Dropper NEET
Multiconcept MCQs
1. Assuming the bond direction to be z‑axis, which of the 6. Choose the correct order of bond angle.
overlapping of atomic orbitals of two atom (A) and (B) will
Column-I Column-II
result in bonding?
A NO2+ 1. 180°
(I) s‑orbital of A and px orbital of B
B NO2 2. 134°
(II) s‑orbital of A and pz orbital of B
C NO2– 3. 120°
(III) py‑orbital of A and pz orbital of B
D NO3– 4. 115°
(IV) s‑orbitals of both (A) and (B)
(a) I and IV (b) I and II (c) III and IV (d) II and IV 5. 109°

2. Choose the INCORRECT property for the following molecules: A B C D A B C D


(I) CH2F2 ; (II) CHF3 ; (III) CH3F (a) 5 4 3 2 (b) 5 2 4 3
(a) C–F bond length order : CH3F > CH2F2 > CHF3 (c) 1 2 4 3 (d) 1 4 3 2

(b) C–H bond length order : CH3F > CH2F2 > CHF3 7. N2 and O2 are converted to monocations N2+ and O2+respectively,
which is wrong statement ?
(c) Shape of the all species is perfectly tetrahedral
(a) In N2+ , the N–N bond weakens
(d) Dipole moment is non zero for given compounds. (b) In O2+, the O–O bond order increases
(c) In O2+, the paramagnetism decreases
3. Identify the correct match.
(d) N2+ becomes diamagnetic
(i) XeF2 (A) Central atom has sp3 hybridisation
and bent geometry. 8. Give the correct order of initials T or F for following
statements. Use T if statement is true and F if it is false.
(ii) N3– (B) Central atom has sp3d2 hybridisation (i) The order of repulsion between different pair of electrons
and octahedral. is lp – lp > lp – bp > bp – bp
(iii) PCl6–(PCl5 (s) (C) Central atom has sp hybridisation (ii) In general, as the number of lone pair of electrons on
anion) and linear geometry. central atom increases, value of bond angle from normal
3 bond angle also increases.
(iv) ICl2+ (I2Cl6 (l) (D) Central atom has sp d hybridisation
cation) and linear geometry. (iii)The number of lone pair on O in H2O is 2 while on N in
NH3 is 1.
(a) (i – A), (ii – B), (iii – C), (iv – D) (iv) The structures of Xenon‑fluorides and Xenon‑oxyfluorides
could not be explained on the basis of VSEPR theory.
(b) (i – D), (ii – B), (iii – D), (iv – C)
(a) TTTF (b) TFTF
(c) (i – B), (ii – C), (iii – A), (iv – D)
(c) TFTT (d) TFFF
(d) (i – D), (ii – C), (iii – B), (iv – A)
9. The H bond in solid HF can be best represented as:
4. Which is not correctly matched?
(a) H–F...H–F...H–F
(a) XeO3 ; Trigonal bipyramidal
(b)
(b) ClF3 ; bent T-shape
(c) XeOF4 ; Square pyramidal
(c)
(d) XeF2 ; Linear shape
5. Which of the following compounds have the same no. of
lone pairs with their central atom ? (d)
[I] XeF5– ; [II] BrF3 ; [III] XeF2 ; [IV] H3S+ [V] : CH2
10. The experimental value of the dipole moment of HCl is 1.03
(a) IV and III D. The length of the H – Cl bond is 1.278Å. The percentage
(b) I and III of ionic character in HCl is:
(c) I and II (a) 43 (b) 21
(d) II, IV and V (c) 17 (d) 7

Chemical Bonding and Molecular Structure 93


11. Which of the following pair of molecules have the same 19. Match List I (Molecules)with List II (Bond order) and select
shape but difference in polarity (polar or non-polar)? the correct answer using the codes.
(a) H2O and NH3 (b) SnCl2 and SO2 List – I List – II
(c) CO2 and N2O (d) SO2 and SO3 I. Li2 A. 3
II. N2 B. 1.5
12. Hydrogen bonding plays a central role in the following
phenomenon : III. Be2 C. 0
(a) Ice floats in water IV. O2 D. 0
(b) Higher Lewis basicity of primary amines than tertiary E. 2
amines in aqueous solutions. Codes
(c) Formic acid is more acidic than acetic acid (a) I - B, II - C, III - A, IV - E (b) I - C, II - A, III - D, IV - E
(d) Dimerisation of acetic acid in benzene (c) I - D, II - A, III - E, IV - C (d) I - C, II - B, III - E, IV - A
13. Among the following pairs, which two species are not 20. The Atomic numbers of three elements A,B and C are a, a+1
isostructural? and a+2. C is an alkali metal. In a compound of A and C, the
nature of bonding is
(a) PF6– and SF6 (b) SiF4 and SF4
(a) Coordinate
(c) IO3– and XeO3 (d) BH4– , NH4+ (b) Covalent
14. Which compound exhibits maximum dipole moment among (c) Ionic
the following? (d) Metallic
21. The electronegativity difference between N and F is greater
than N and H, yet the dipole moment of NH3 (1.5D) is
(a) (b) greater than that of NF3 (0.2D). This is because:
(a) In NH3 as well as NF3, the orbital dipole and bond dipole
are in opposite direction
(b) In NH3 , the orbital dipole and bond dipole are in the opposite
(c) (d) direction, where as in NF3 these are in same direction
(c) In NH3 , as well as in NF3 the orbital dipole and bond
dipole are in same direction.
(d) In NH3, the orbital dipole and bond dipole are in same
15. Which of the following is a wrong order with respect to the direction where as in NF3 these are in opposite direction.
property mentioned against each ?
22. Which of the following statements are correct?
(a) O22– > O2 > O2+ [Paramagnetic moment] (I) The hybridisation found in cation of solid PCl5 is sp3.
(b) Bond Angle : H2O > H2S > H2Se (II) In AB2L2 type, the BAB bond angle is always greater than
(c) H2 > H2+ > He2+ [stability] the normal tetrahedral bond angle.
(d) NO2+ > NO2 > NO2– [bond angle] (III) In ClO3–, NH3 and XeO3, the hybridisation and the
number of lone pairs on the central atoms are same.
16. O2F2 is an unstable yellow orange solid and H2O2 is a
(IV) In discrete P4 molecule, there are six P—P bonds and
colourless liquid, both have O–O bond and O–O bond length
four lone pairs of electrons.
in H2O2 and O2F2 respectively is -
(a) I, II and III only
(a) 1.22 Å, 1.48 Å
(b) I, III and IV only
(b) 1.48 Å, 1.22 Å (c) III and IV only
(c) 1.22 Å, 1.22 Å (d) All are correct
(d) 1.48 Å, 1.48 Å 23. Which of the following statements are correct? (As the
17. Which bond angle θ would result in maximum dipole s-character of a hybrid orbital decreases).
moment for the triatomic molecule XY2 shown below ? (I) The bond angle decreases
(a) θ = 90° (b) θ = 120° (II) The bond strength increases
(c) θ = 150° (d) θ = 180° (III) The bond length increases.
18. Among the following select the correct statements: (a) (I) and (III) (b) (II) and (III)
(c) (I) and (II) (d) All are correct
I. PH6 does not exist
24. The correct order of increasing C - O bond length of CO,
II. pπ-dπ bond is present in SO2
CO32 – , CO2 is
III. SeF4 and CH4 have the same shape (a) CO32 – < CO2 < CO
IV. I3+ has a bent shape (b) CO2 < CO32 – < CO
(a) II, III (b) I, II (c) CO < CO32 – < CO2
(c) I, IV (d) I, II, IV (d) CO < CO2 < CO32 –
94 Dropper NEET
NEET Past 10 Years Questions

1. BF3 is planar and electron deficient compound. Hybridization The bond energy of H2 is  (2020 Covid Re-NEET)
and number of electrons around the central atom, (c− a) (b − a)
respectively are: (2021)
(a) (b)
2 2
(a) sp3 and 6 (b) sp2 and 6 (c) (c – a) (d) (b – a)
(c) sp2 and 8 (d) sp3 and 4
8. Identify the wrongly matched pair. (2020 Covid Re-NEET)
2. Match List-I with List-II. (2021)
Molecule Shape or geometry of molecule
List-I List-II
(a) SF6 Octahedral
(A) PCl5 (i) Square pyramidal
(B) SF6 (ii) Trigonal planar (b) BeCl2 Linear
(C) BrF5 (iii) Octahedral (c) NH3 Trigonal pyramidal
(D) BF3 (iv) Trigonal bipyramidal (d) PCl5 Trigonal planar
Choose the correct answer from the options given below.
9. Which of the following diatomic molecular species has only
(a) A-ii B-iii C-iv D-i
p bonds according to Molecular Orbital Theory?  (2019)
(b) A-iii B-i C-iv D-ii
(a) O2 (b) N2
(c) A-iv B-iii C-ii D-i
(c) C2 (d) Be2
(d) A-iv B-iii C-i D-ii
10. Identify the incorrect statement related to PCl5 from the
3. Which of the following molecules is non-polar in nature?(2021) following:  (2019)

(a) CH2O (b) SbCl5 (a) Three equatorial P-Cl bonds make an angle of 120° with
each other
(c) NO2 (d) POCl3
(b) Two axial P-Cl bonds make an angle of 180° with each
4. Which of the following set of molecules will have zero other
dipole moment?  (2020)
(c) Axial P-Cl bonds are longer than equatorial P-Cl bonds
(a) Boron trifluoride, hydrogen fluoride, carbon dioxide, (d) PCl5 molecule is non-reactive
1,3-dichlorobenzene
11. Consider the following species: (2018)
(b) Nitrogen trifluoride, beryllium difluoride, water 1,
CN+, CN–, NO and CN
3-dichlorobenzzene
Which one of these will have the highest bond order?
(c) Boron trifluoride, berylium difluoride, carbon dioxide,
(a) NO (b) CN– (c) CN (d) CN+
1,4-dichlorobenzene
(d) Ammonia, beryllium difluoride, water, 12. Which one of the following pair of species have the same
1,4-dichlorobenzene bond order? (2017-Delhi)
(a) N 2 ,O 2

(b) CO, NO
5. Identify a molecule which does not exist.  (2020)
(a) Li2 (b) C2 (c) O2 (d) He2 (c) O2, NO+ (d) CN–, CO
6. How many (i) sp2 hybridised carbon atoms and (ii) π bonds 13. The species, having bond angles of 120° is (2017-Delhi)
are present in the following compound? (2020 Covid Re-NEET) (a) BCl3 (b) PH3
(c) ClF3 (d) NCl3
14. Which one of the following ions is not tetrahedral in shape?
(a) 8, 6 (b) 7, 6 (c) 8, 5 (d) 7, 5  (2017-Gujarat)
(a) [NiCl4] 2– (b) NH 4+
7. The potential energy (y) curve for H2 formation as a function
of internuclear distance (x) of the H atoms is shown below. (c) BF4− (d) [Cu(NH3)4]2+
15. Which of the following pair of species is not iso-structural?
 (2017-Gujarat)
(a) BrO3 ,XeO3

(b) ICl4 ,XeF4

(c) ClO3− ,CO32 − (d) IBr2− ,XeF2

Chemical Bonding and Molecular Structure 95



16. Which of the following hydrides has the largest bond angle? 24. Decreasing order of stability of O2 , O2 , O2+ and O22– is:
 (2017-Gujarat)  (2015 RE)
(a) H2Se (b) H2S (c) H2Te (d) H2O –
(a) O2– > O2– > O2+ > O2 (b) O2+ > O2 > O2 > O22–
2
17. Which one of the following compounds shows the presence – –
(c) O2– > O2 > O2 > O2+ (d) O2 > O2+ > O22– > O2
of intramolecular hydrogen bond? (2016 - II) 2

(a) Cellulose (b) Concentrated acetic acid 25. The total number of π-bond electrons in the following structure
is: (2015)
(c) H2O2 (d) HCN
18. In which of the following molecules, all atoms are coplanar?
 (2016 - II)

(a) (b)
(a) 8 (b) 12 (c) 16 (d) 4
26. Which of the following options represents the correct bond
order? (2015)
(a) O2– < O2 < O+ (b) O2– > O2 < O +
(c) (d) 2 2
(c) O2– < O2 > O2+ (d) O2– > O2 > O2+
27. The correct bond order in the following species is: (2015)
– –
(a) O2+ < O2 < O2+ (b) O2+ < O2 < O2+
2 2
19. Among the following which one is a wrong statement? (2016 - II) – –
(c) O2 < +
O2 < 2+
O2 (d) 2+
O2 < O2+< O2
(a) SeF4 and CH4 have same shape
(b) I3+ has bent geometry 28. Which of the following pairs of ions are isoelectronic and
isostructural? (2015)
(c) PH5 and BiCl5 do not exist
(a) SO3 2– , NO3 – –
(b) ClO3 , SO3 2–
(d) p� - d� bonds are present in SO2
2–
(c) CO3 , SO3 2– (d) ClO3– , CO32–
20. The hybridisations of atomic orbitals of nitrogen in NO2+,
NO–3 and NH+4 respectively are: (2016 - II) 29. Maximum bond angle at nitrogen is present in which of the
3
(a) sp, sp and sp 2 2 3
(b) sp , sp and sp following? (2015)

(c) sp, sp2 and sp3 (d) sp2, sp and sp3 (a) NO2+ (b) NO3– (c) NO2 (d) NO2–

21. Which of the following pairs of ions is isoelectronic and 30. Which of the following molecules has the maximum dipole
isostructural? (2016 - II)
moment? (2014)
(a) CH4 (b) NH3 (c) NF3 (d) CO2
(a) CO32– , NO3– (b) ClO3– , CO32–
(c) SO32– , NO3– (d) ClO3– , SO32– 31. Which one of the following species has planar triangular shape?
 (2014)
22. Consider the molecules CH4, NH3 and H2O. Which of the (a) NO3– (b) NO2–
given statement is false? (2016 - I)
(c) CO2 (d) N3
(a) The H─C─H bond angle in CH4 is larger than the
H─N─H bond angle in NH3 32. Which of the following organic compounds has same
hybridisation as its combustion product (CO2)? (2014)
(b) The H─C─H bond angle in CH4, the H─N─H bond (a) Ethyne (b) Ethene
angle in NH3, and the H─O─H bond angle in H2O are all
(c) Ethanol (d) Ethane
greater than 90º.
33. Which of the following is a polar molecule? (2013)
(c) Then H─O─H bond angle in H2O is larger than the
(a) BF3 (b) SF4
H─C─H bond angle in CH4
(c) SiF4 (d) XeF4
(d) The H─O─H bond angle in H2O is smaller than the
34. Which of the following is electron-deficient? (2013)
H─N─H bond angle in NH3
(a) (CH3)2 (b) (SiH3)2
23. Predict the correct order among the following: (2016 - I)
(c) (BH3)2 (d) PH3
(a) Lone pair ─ bond pair > bond pair ─ bond pair > lone
35. Which of the following is paramagnetic? (2013)
pair ─ lone pair
(b) Lone pair ─ lone pair > lone pair ─ bond pair > bond pair (a) CO (b) O −2 (c) CN– (d) NO+
─ bond pair 36. Four diatomic species are listed below. Identify the correct
(c) Lone pair ─ lone pair > bond pair ─ bond pair > lone pair order in which the bond order is increasing in them:(2012 Mains)
─ bond pair (a) He2+ < O2– < NO < C22 – (b) NO < O2– < C22 – < He2+
(d) Bond pair ─ bond pair > lone pair ─ bond pair > lone pair (c) O2– < NO < C22 – < He2+ (d) C22 – < He2+ < O2– < NO
─ lone pair

96 Dropper NEET
37. In the replacement reaction 39. Bond order of 1.5 is shown by:  (2012 Pre)
(a) O2 (b) O2+

(c) O 2 (d) O22 –
40. Which one of the following pairs is iso-structural (i.e., having
the same shape and hybridisation)?  (2012 Pre)
The reaction will be most favorable if M happens to be: – +
(a) [BF4 and NH4 ] (b) [BCl3 and BaCl3]
(2012 Mains)
(c) [NH3 and NO3–] (d) [NF3 and BF3]
(a) Li (b) Na (c) K (d) Rb
41. During change of O2 to O −2 ion, the electron adds on which one
38. The pair of species with the same bond order is: (2012 Pre)
of the following orbitals? (2012 Mains)
(a) N2 , O2 (b) O22 –, B2 (a) π* orbital (b) π orbital
+
(c) O2 , NO + (d) NO , CO (c) σ* orbital (d) σ orbital

Chemical Bonding and Molecular Structure 97


ANSWER KEY
Topicwise Questions

1. (c) 2. (b) 3. (b) 4. (b) 5. (d) 6. (c) 7. (b) 8. (b) 9. (b) 10. (d)
11. (d) 12. (c) 13. (d) 14. (b) 15. (c) 16. (b) 17. (c) 18. (d) 19. (a) 20. (c)
21. (a) 22. (a) 23. (d) 24. (b) 25. (c) 26. (c) 27. (a) 28. (d) 29. (c) 30. (c)
31. (b) 32. (b) 33. (a) 34. (b) 35. (b) 36. (b) 37. (c) 38. (d) 39. (b) 40. (c)
41. (c) 42. (c) 43. (a) 44. (b) 45. (a) 46. (a) 47. (c) 48. (a) 49. (b) 50. (b)
51. (a) 52. (b) 53. (d) 54. (c) 55. (a) 56. (d) 57. (c) 58. (d) 59. (d) 60. (a)
61. (d) 62. (b) 63. (b) 64. (d) 65. (d) 66. (c) 67. (c) 68. (d) 69. (d) 70. (c)
71. (b) 72. (b) 73. (d) 74. (b) 75. (c) 76. (b) 77. (c) 78. (b) 79. (a) 80. (b)
81. (c) 82. (d) 83. (c) 84. (b) 85. (b) 86. (c) 87. (c) 88. (a) 89. (b) 90. (b)
91. (a) 92. (d) 93. (a) 94. (b) 95. (c) 96. (b) 97. (b) 98. (b) 99. (d) 100. (b)
101. (a) 102. (c) 103. (c) 104. (b) 105. (d) 106. (c) 107. (d) 108. (a) 109. (a) 110. (c)
111. (a) 112. (d) 113. (d) 114. (d) 115. (b) 116. (a) 117. (b) 118. (b) 119. (b) 120. (a)
121. (c) 122. (d) 123. (a) 124. (a) 125. (a) 126. (b) 127. (d) 128. (a) 129. (c) 130. (b)
131. (d) 132. (c) 133. (d) 134. (d) 135. (c) 136. (d) 137. (c) 138. (b) 139. (b) 140. (b)
141. (c) 142. (b) 143. (a) 144. (b) 145. (c) 146. (c) 147. (a) 148. (a) 149. (c) 150. (b)
151. (b) 152. (c) 153. (c) 154. (a) 155. (a) 156. (b) 157. (a) 158. (b) 159. (b) 160. (c)
161. (b) 162. (c) 163. (d) 164. (b) 165. (b) 166. (a) 167. (d) 168. (d) 169. (a) 170. (a)
171. (b) 172. (a) 173. (b) 174. (d) 175. (a) 176. (a) 177. (c) 178. (b) 179. (a) 180. (b)
181. (c) 182. (d) 183. (b) 184. (d) 185. (c) 186. (d) 187. (d) 188. (c) 189. (a) 190. (a)
191. (d) 192. (c) 193. (b)

Learning Plus
1. (b) 2. (a) 3. (c) 4. (c) 5. (a) 6. (b) 7. (c) 8. (b) 9. (c) 10. (d)
11. (a) 12. (c) 13. (d) 14. (a) 15. (c) 16. (c) 17. (a) 18. (d) 19. (c) 20. (c)
21. (b) 22. (a) 23. (c) 24. (c) 25. (b) 26. (d) 27. (b) 28. (a) 29. (b) 30. (a)
31. (d) 32. (a) 33. (c) 34. (d) 35. (a) 36. (c)

Multiconcept MCQs
1. (d) 2. (c) 3. (d) 4. (a) 5. (c) 6. (c) 7. (d) 8. (b) 9. (c) 10. (c)
11. (c) 12. (a) 13. (b) 14. (c) 15. (a) 16. (b) 17. (a) 18. (d) 19. (b) 20. (c)
21. (d) 22. (b) 23. (a) 24. (d)

NEET Past 10 Years Questions


1. (b) 2. (d) 3. (b) 4. (c) 5. (d) 6. (b) 7. (d) 8. (d) 9. (c) 10. (d)
11. (b) 12. (d) 13. (a) 14. (d) 15. (c) 16. (d) 17. (a) 18. (c) 19. (a) 20. (c)
21. (a,d) 22. (c) 23. (b) 24. (b) 25. (a) 26. (a) 27. (c) 28. (b) 29. (a) 30. (b)
31. (a) 32. (a) 33. (b) 34. (c) 35. (b) 36. (a) 37. (d) 38. (b) 39. (c) 40. (a)
41. (a)

98 Dropper NEET
4 Chemical Bonding and
Molecular Structure

Topicwise Questions
1. (c) Q The valency of O is 2, by cross method the compound 14. (b) Due to Fajan's rule, according to which the cation distorts
becomes M2O4 which is same as MO2. the electron cloud of the anion and pulls the electron
2. (b) Valency is double to the number of oxygen atoms density towards itself. Thus, it introduces some covalent
reacting. character in the ionic bond. Now, the smaller the cation,
greater will be its polarizability.
3. (b) Differentiating electrons enters into (n-1)d electrons
participate in bonding. It is property of transition metal. 15. (c) Ionic character increases as: BeCl2 < MgCl2 < CaCl2 <
4. (b) The central atom of Be contains 4 electrons BaCl2 because larger the size of the cation, higher is the
ionic character.
5. (d) XeF4 has 10 electrons in valence shell of Xe
16. (b) 17. (c) 18. (d)
6. (c) PCl5 has 10 electrons in valence shell of P.
19. (a)

7. (b) (I) Cl–Be–Cl (II)


(III) H–H
20. (c) Since the molecule possess 6 pairs of electrons, it should
8. (b) Strongest Lewis Acid is one with maximum O.S. exhibit octahedral geometry. But as its structure is planar,
it will have 2 lone pairs of electrons, and 4 sigma bonds.
21. (a)
9. (b) a. b.
22. (a) More electropositive and more electronegative combination
gives more ionic compound.
lp = 0 lp = 12 23. (d) Ionic bond is formed by ions.
24. (b) Small anion forms more ionic compound.

c. : C ≡ N : d. 25. (c) All the C–atoms in benzene are sp2 hybridised.
26. (c) Greater the strength of bond, more is the directional
character.
lp = 2 lp = 8
27. (a) Sodium Fluoride
28. (d) The electronegativity of S is least among others. Hence
10. (d) all have complete octet H-S bond is least polar in nature.

29. (c)

11. (d)

12. (c) CO(NH2)2 13. (d) 14 30. (c) ΔEN. = 0 ∴ non-polar.


31. (b) 4.8 e.s.u Cm = 1.6 ×10–29 col.m 46. (a) The hybrid orbitals used for forming C–H bonds contain
1.6 × 10−29 more s–character than hybrid
1 e.s.u. Cm =
4.8
= 0.33 × 10–29 col.m
= 3.33 × 10–30 col.m
32. (b) In HCl, H–atom and Cl–atom shares one electron pair to
form covalent bond. orbitals used for forming C–F bonds. As more the
s–character in hybrid orbital larger will be bond angle.
33. (a) Li+ being smaller, according to Fajan rule, Li+ has high
polarising power. Hence LiCl should have maximum 47. (c) In H–C≡C–H, the C–H bond is made of sp-s overlap
ionic character. which imparts shorter bond length.
34. (b) Symmetrical with linear structure will have zero dipole 48. (a) As the electro-negativity of the central atom decreases
moment. and the size increases, the bond angle decrease in similar
⊕ molecules.
35. (b) O = N = O Bond order = 2
36. (b) In diamond, one carbon atom is attached to more carbon
atoms by means of covalent bonds. 49. (b)
37. (c) NH +4 is sp3 hybridised
 1  Total number of angles are six.
 H= 2 [5 + 4 − 1 + 0]
= 4
  , HC
HCH  F, FC
 F, HC
 F, HC
 F, HC
F
38. (d) PH3, θ = 94 o
 (, HC
Maximum number of identical angles are HCH
four  F,) FC
 F, HC
 F,
 , FBrF
39. (b) FClF  as electronegativity of central atom increases
50. (b) Due to absence of lone pair on N in NH4+, lp–lp, or lp –bp
angle of molecule increases. repulsion is not there.
40. (c) Thus for each P–3 P–P–P angle 51. (a) O is more electronegative than N & S take greater strength
of H-bonds as the strength of H-bonds is attributed to the
electro-negativity of the atom bonded to hydrogen.
52. (b) In H2S, due to low electronegativity of sulphur the
l.p.-l.p. repulsion is more than b.p. -b.p. repulsion and
hence the bond angle is 92°.
∴ for 4 it is 12
53. (d) is one of the resonating structure of
O ozone.
41. (c) CH3 111.7° 54. (c) Hint: Due to maximum lp-bp repulsion present in H2O.
CH3
42. (c) Bond angle of hydrides of group 16 elements are 55. (a) The order of bond angles
H2O > H2S > H2Se > H2Te BF3 > SiH4 > NH3 > H2S
120° 109°28’ 107° 92.5°
43. (a) H2O (104.50) ; BeF2 (1800)
CH4 (1090) ; NH3 (1070) 56. (d)
57. (c) Bond angles are:
44. (b) BF3 – 120°, NF3 – 106°, PF3 – 101°, ClF3 – 90°
58. (d) Bond angle of water is 104.5. In water, oxygen is sp3 –
hydridised with two lone pairs and two bond pairs and
In SOBr2, S–O bond has maximum bond length in lp - lp repulsions is more than l.p - b.p. repulsion, which
comparision to S–O bond lengths in SOF2 and SOCl2, in turn is more than bp - bp repulsion.
because in SOBr2, S–O bond has been formed by hybrid
59. (d) O–O bond length in O2 is 120.7 pm, O3 is 127.8 pm,
orbital containing less s-character. According to Bent H2O2 is 147.5 pm
Rule.
60. (a)
61. (d) Shape of the molecule depends upon the total number of
45. (a)
electron pairs around the central atom which include l.p
as well as b.p.
Equatorial bond angle has been reduced from 120° to 62. (b) SF4 has see-saw shape due to 1 lp and 4 bp and XeF4 has
103° square planar shape due to 2 lp and 4 bp.

Chemical Bonding and Molecular Structure 33


71. (b) Polarity of molecule decreases with decrease in dipole
moment. The dipole moment of HF, H2O, SO2 and NH3
63. (b)
are 1.92 D, 1.84 D, 1.60 D, and 1.4 D respectively.
72. (b) 3 B.P and 1 L.P
73. (d) sp3- Tetrahedral.
74. (b) CO2 and BeF2 both are linear.
75. (c) Regular geometry of CH4 is attributed to zero dipole
moment (µ = 0) and absence of lone pair on carbon atom.
76. (b) OF2 has 2 bond pairs and 8 lone pairs.
:F O F:

: :

: :
: :
77. (c) Both XeF4 and [Ni(CN)4]2– are square planar. In XeF4,
Xe is sp3d2 hydridised with 2 lp and Ni in [Ni (CN)4]2– is
dsp2 hybridised.
78. (b) In aldehydes hybridisation of carbon = sp2
79. (a) s-character = 20%.
80. (b) In decane 4 × 10 = 40 hybrid orbitals present.
81. (c) C-C bond is sp3 – sp3 and C-Cl is sp3 – p

82. (d) sp3 hybridization, hence s - character is 25%


83. (c) C2(CN)4 is

64. (d) N≡ C C C≡ N
N≡ C C C≡ N
Tetra atomic C = C is sp2 hybridised and C ≡ N is sp hybridised.
µd ≠ 0 µ d =0 84. (b)
85. (b) N ≡ C –C ≡ N
Planar & Polar Planar & non-polar
65. (d) Values of dipole moment can’t be predicted as we don’t
know about the electronegativities of the atoms. 86. (c) a.
66. (c) To have minimum repulsions, the two lone pair occupy
the trans positions in octahedral geometry.
67. (c) Hybridisation of SO 24−
b.
1
H= [6 + 0 − 0 + 2]= 4
2
68. (d) a. XeO3 is trigonal pyramid. c.
b. IOF4+ is see-saw.
c. PCl5 is trigonal bipyramidal.
d.

d.
c. → triple bond therefore bond length shortest.

square pyramidal. 87. (c) s = 12, p = 3

69. (d) 88. (a) s - p bond involves head-on overlapping. Therefore; it is


a sigma (σ) bond.
89. (b) S = C = S (linear) is best explained by sp hybrid state of
70. (c) sp3d3
C.

34 Dropper NEET
90. (b) a σ bond is formed by axial overlapping. (b) NH 3 and NO3−
b. p-orbital have both axial and side ways overlapping

91. (a)

In OPF3, molecule % s‑character in O – P is maximum (c) NF3 and BF3


according to Bent’s Rule, hence it is strongest among all.

92. (d)

93. (a) a. CH4 + 2O2 → CO2 + 2H2O


sp3     sp
94. (b) Do it yourself
(d) BF4− and NH +4
95. (c) XeF5– : 2 lone pairs ; pentagonal planar
BrF3 : 2 lone pairs ; Bent-T‑shape
XeF2 : 3 lone pairs ; Linear
H3S+ : 1 lone pairs ; Pyramidal
↑↑
CH 2 : 2 unpaired ; Linear
(No lone pair)

96. (b)
100. (b)

101. (a)

97. (b) NH 3 + H + → NH +4 2 NH 3 → NH 2− + NH +4
102. (c) (a) Electronic configuration of Beryllium in ground state
sp3 sp3 sp3 sp3 sp3 is 1s22s22p1.
H3BO3 + OH– → [B(OH)4]– H2O + H+ → H3O+
sp2 sp3 sp3 sp3 sp3

98. (b) N2O5 in solid state → NO2+ NO3–


(b) Electronic configuration of nitrogen in ground state
NO3– → hybridisation →sp2 is 1s22s22p3.
99. (d) If number of B.P. and L.P are same for the given pairs,
they are isostructural.
(a)
(c) Electronic configuration of phosphorus in ground
state is 1s22s22p63s23p3.

Chemical Bonding and Molecular Structure 35


(d) Electronic configuration of boron in ground state is 120. (a) bond angle ∝ s- character.
1s22s2. 121. (c) sp3d hybridization gives TBP.
122. (d) s-orbital of hydrogen and p-orbital of fluorine overlaps.
123. (a) Overlapping of atomic orbitals determines direction of
the bond.

42
124. (a) sigma bond is directional.
103. (c) IF5 = 7 + 5 × 7 = = 5 +1
8 125. (a) Order of overlapping is
sp3d2 hybridisation, hence it should be octahedral. σp – p > σs – p > σs – s, 1s-1s bond is stronger because
Due to the presence of one lone pair of electrons, its 1s orbital is close to the nucleus.
actual geometry is square pyramidal. 126. (b) Do it yourself
127. (d) 104° 28
128. (a) In H2O, hybridization is sp3 and the shape is angular due
to 2 bp & 2 lp
129. (c) In BeCl2, hybridization sp kind the shape is linear due to
104. (b) A fact, to be remembered. 2 bp & 0 lp
5 + 4 −1
105. (d) In perchlorate ions,
Central Cl atom is sp3 - hybridised. So; all
+
130. (b) NH=
4 T .P = 4 sp 3
2
( )
the four sigma bonds are sp3 - p bonds. 131. (d) sp3d has bond angles 90° & 120°
132. (c) All of them have total four electron pairs with 0 lp .
106. (c) a. CH4 → CH3––CH3 Hence tetrahedral in shape.
4 b.p 3 b.p 4 b.p
133. (d) CH3CH2OH is saturated hence all carbon atoms are sp3
Hybrisation: Tetrahedral Tetrahedral hybridized.
b. NH3 → NH+4
134. (d) Ethylene on hydrogenation into ethane : hybradization ,
3 b.p +1 l.p 4 b.p bond angle & C - C bond length change
Hybrisation: Pyramidal Tetrahedral 135. (c) Do it yourself
c. BF3 → BF4− 136. (d) π*2px2.π*2py1 antibonding molecular orbitals contain 3
Hybridisation: Tetrahedral trigonal planar electrons in O2– . There energy being more , ionisation
d. H2O → H3O+ potential is less.
2b.p.+2 l.p 3b.p. + 1 l.p 137. (c) KO2 has K+ and O2– structure having one unpaired
Hybridisation: Angular Pyramidal electron.
Thus, conversion of BF3 into BF4− involves changes in 138. (b) B.O. of O2 = 2
both hybridisation and shape. O2 + = 2.5
O2 − = 1.5
107. (d) Perfectly tetrahedral µD = 0 non polar sp3-
139. (b) Both O2+ and O2 are paramagnetic bond order of O2+= 2.5
hybridised and O2 = 2
108. (a) Order of overlapping is 140. (b) (a) C2 → C2+
σ p – p > σs – p > σs – s B.O.= 2 B.O. = 1.5
109. (a) hybrid orbitals form sigma bonds only. µ = 0, µ ≠ 0,
i.e. diamagnetic i.e. paramagnetic
110. (c) pi bond is formed by the lateral overlaping.
(b) NO → NO+
111. (a) sigma bond is stronger than pi bond.
B.O.= 2.5 B.O. = 3.0
112. (d) sp3d2 µ ≠ 0, µ = 0,
113. (d) In propane carbon undergoes sp3 hybridisation. i.e. paramagnetic i.e. diamagnetic
114. (d) Both PO43– & SO32– have hybridisation sp3. (c) O2 → O+2
115. (b) In propene 10 hybrid orbitals (3+3+4) are present. B.O.= 2 B.O. = 2.5
µ ≠ 0, µ ≠ 0,
116. (a) In sp3d2 hybridization, angle between two hybrid orbital is 90°.
i.e. paramagnetic i.e. paramagnetic
117. (b) p-character = 75% (c) N2 → N+2
118. (b) In hybridisation combination and redistribution of B.O.= 3 B.O. = 2.5
atomic orbitals takes place. µ = 0, µ ≠ 0,
119. (b) sp hybridisation is formed. i.e. diamagnetic i.e. paramagnetic
36 Dropper NEET
141. (c) σ1s2 σ*1s1 157. (a) Isostructural species will have same bond order
142. (b) It does not have any π - bond because all the molecules 158. (b) 1.5 159. (b) N2 160. (c)
have O - atom with lone pairs but in H2O, the H - atom 161. (b) More is the partial charges generated more is the strength
has no vacant orbital for π - bonding. of H-bonding.
143. (a) O 22 − (Number of electron = 18 does not contain unpaired
electron)
O 22− = σ1s2 , σ*1s2 , σ2s2 , σ*2s2 , σ2pz2,
π2p2x ≈ π2p2y π*2px2 = π*2py2
144. (b) Since anti-bonding orbitals are formed by substraction of The strength of H-bonding is more in H2O.
orbitals.
162. (c) F has highest EN.
145. (c) Bond order of Be2 is zero, it does not exist. 163. (d) o-Salicylaldehyde has intramolecular H-bonding.
146. (c) According to MOT oxygen is paramagnetic with two
164. (b) Ammonia has hydrogen bonds
unpaired electrons.
147. (a) NO– has 16 electron system bond order = 2 165. (b) Ortho-nitrophenol
NO–, CN–, N2 are isoelectronic with CO.
∴ All have same bond order = 3
148. (a) Bond order ∝ bond length.
+
B.O of O 2 = 2.5 and rest has B.O. less than 2.5
Thus, it has the strongest O – O bond among the given.
149. (c) Total number of electrons in N −2 = 7 + 7 + 1 = 15 166. (a) H-bond by simply vander waal’s attraction.
2− 167. (d) Because of high EN of ‘O’ forms intermolecular
Total number of electrons in N = 7 + 7 + 2 = 16
2
2−
H-bonding.
Total number of electrons in O = 8 + 8 + 2 = 18
2
168. (d) Bromine has no hydrogen bonds.
Total number of electrons in O = 8 + 8 + 1 = 17

2
Total number of electrons in O = 8 + 8 - 1 = 15
+
2
169. (a) Ethyl alcohol has hydrogen bonds.
170. (a) Due to +I effect. An electron-releasing group increases
Configuration will be same as N −2 and O +2 contain the electron density on oxygen tending to decrease the
same total number of electrons. Thus N −2 and O +2 has the polarity of O—H bone. This decreases the Hydrogen
same bond order. bond strength.
150. (b) B2 = KK * (σ2s)2 (σ*2s)2 (πpx)1 ≈ (πpy)1 171. (b) C2H5OH can form intermolecular H-bonding with H2O.
1 172. (a) Because base units in DNA has H,N & O as component
Bond order = (4 − 2) = 1
2 elements.
151. (b) C (12) = σ1s2 , σ*1s2 , σ2s2 , σ*2s2 , π2p2x ≈ π2py2 173. (b) Because o-nitrophenol has intramolecular H-bonding.
C2 has two pairs of electrons in π2px and π 2py , therefore; 174. (d) More number of H-bonds
it has only 2π - bonds. 175. (a) Only O,N,F (because of high EN) can form H-bonding.
152. (c) No unpaired electrons, therefore diamagnetic. 176. (a) Ionic bond > covalent bond > Hydrogen bond > vander
eg: Li2 = 6e- = σ1s2 ,σ*1s2 , σ2s2 waals forces > metallic bond.
153. (c) Mulliken gave the molecular orbital theory. 177. (c) NH3 has intermolecular H-bonding.
154. (a) Given electronic configuration of anion X is- 178. (b) 2 lone pairs of ‘O’ and 2’H’ atoms in H2O molecule
σ1s2 , σ*1s2 , σ2s2 , σ*2s2 , π2p2x ≈ π2py2, σ2pz2 , π*2px1 involve in H-bonding

∴ Total number of electron of anion X = 15 179. (a) Ice occupies more volume than water
Hence; the anion X is N2– 180. (b) Intermolecular H-bonding is formed between H2O
molecules.
155. (a) In the anti-bonding molecular orbital, most of the
electron density is located away from the space between 181. (c) In aqueous ammonia hydrogen bonding is present
the nuclei, as a result of which there is a nodal plane (i.e., between NH3 & H2O .
a plane at which the electron density is zero) between the 182. (d) None
nuclei.
183. (b)
+
156. (b) According to Molecular Orbital Theory, H 2 = σ1s1 184. (d) Hydrogen bond is formed between molecules of
N − Na 1 − 0 compounds having O, F and N with hydrogen.
Bond order b = = 0.5
2 2
Chemical Bonding and Molecular Structure 37
185. (c) Ethyl alcohol forms stronger H - bonds than ethylamine
or ammonia due to greater electronegativity of oxygen
than nitrogen atom. Diethyl ether, however, does not
form H-bonds, since it does not have a H-atom attached 190. (a) Water (H2O) is least volatile due to the association of H2O
to any highly electronegative atom. molecules through H-bonding.
186. (d) The higher boiling point of water is due to hydrogen 191. (d) Strength of hydrogen bond between H----F is the strongest
bonding. O-atom of each H2O molecule is covalently and hence; it has the strongest bond.
linked with two H-atoms of its own molecule and with 192. (c) Intermolecular H - bonding is found in (HF)n due to higher
another H-atom of adjacent H2O molecule by H-bonding. electro-negativity of fluorine atoms. Hydrogen bonding
187. (d) Ice has H-bonding therefore; when two ice-cubes are is helpful in the association of HF molecule. So, HF form
pressed over each other, they unite to form one cube. in liquid form.
188. (c) In ethanol, hydrogen bonding is maximum. 193. (b)
189. (a) The molecule of CH3OH are associated by hydrogen
bonding.

Learning Plus
1. (b) 2. (a) with the 3rd O atom & due to the -ve charge present on O
3. (c) CO2 being symmetrical has zero dipole moment atom octet is completed.
Among HI, SO2 and H2O dipole moment is highest for Thus, 3 O-atoms shared with 8 electrons of N-atom.
H2O as in it the central atom contains 2 lone pairs. ∴ Number of bond pairs (or shared pairs) = 4
4. (c) 5. (a) Number of lone pairs = 0
6. (b) Strength of H-bond is in the order H....F > H…O > 11. (a) A fact, to be remembered.
H…..N. 12. (c)
But each H2O molecule is linked to four other H2O
molecules through H-bonds whereas each HF molecule 13. (d)
is linked only to two other HF molecules. Hence, b.p of
H2O > b.p of HF > b.p. of NH3

7. (c) (a) NH3 bond angle is 107.5°


(b) 14. (a) (BH4 ⇒ 4 bond pairs + 0 lone pair ⇒ sp3 hybridised =
tetrahedral geometry)
(CO32 – = triangular planar
1
(c) Percent ‘s’ character = × 100 = 33.33% –
NH2 = V – shape
3
H3O+ = pyramidal)
Percent ‘p’ character = 66.67 %
8. (b) 15. (c) The given compound will have the correct structure as
19 σ-bonds are present in the given molecule i.e., there
9. (c) In PO43 – ion, formal charge on each O-atom of P–O bond
are 5 π-bonds and 8 C–H + 11 C–C σ-bonds.
total charge 3 16. (c) NO+2 involves an sp type of hybridization.
= =− =−0.75
Number of O − atom 4

10. (d) In N-atom, number of valence electrons = 5 17. (a)


2e– of N atom are covalently bonded with one O atom,
other 2e– of N atom are involved in coordinate bond with
other O atom, one e– lest of N atom is covalently bonded

38 Dropper NEET
+ 28. (a) As electronegativity of P is much lower than N, so in
18. (d) Cl – Be – Cl O = N = O NH3
Nitrogen attracts all bond electron towards the centre and
electron-electron repulsion is higher so it has a higher
bond angle. Bond angle in ammonia is 107º & that in
phosphene is 93º.
19. (c) The electronic configuration of the given molecules are
(2py) 
2
(2py)
* *
29. (b) C2 : (σ1s) 2 < (σ1s) 2 < (σ2s) 2 < (σ 2s) 2 < (p2z px =p2z py)
N 2+ =σ1s 2 , σ *1s 2 , σ2s 2 , σ * 2s 2 , pp 2x =pp 2y , σ2p1z
It has one unpaired electron. µeff = 0 ; i.e. diamagnetic
* *
2 2 2 2 1
2 2 2 2 2 2 1 1 1 B2 : (σ1s) < (σ1s) < (σ2s) < (σ 2s) < (p2px )
O 2 = σ1s , σ *1s , σ2s , σ * 2s , σ2p , p2p ≈ p2p p * 2p = p * 2p
z x y z y
(p2py )1 (p2px ) =
= (p2py )
O2 has two unpaired electrons.
µeff ≠ 0 ; i.e. paramagnetic
2− 2 2 2 2 2 2
O 2 σ1s , σ *1s , σ2s , σ * 2s , σ2p , p2p ≈
= z x
* *
O2 : (σ1s) 2 < (σ1s) 2 < (σ2s) 2 < (σ 2s) 2 < (σ2pz) 2
p2p 2y , p * 2p 2x ≈ p * 2p 2y * *

Thus, O22 – has no unpaired electrons. < (p2px ) 2 = (p2py ) 2 < (p 2px)1 = (p 2py)1

B2 = σ1s 2 , σ *1s 2 , σ2s 2 , σ * 2s 2 p2p1x ≈ p2p1y Thus, B has O2 : µeff ≠ 0


2
two unpaired electrons. O22 + : µeff = 0

20. (c) XeF4 ⇒ 4bp + 2p square planar ⇒ all bonds are equal O22 – : µeff = 0

BF4 ⇒ 4bp + 0p ⇒ tetrahedral (all bonds are equal) N2 :
SiF ⇒ 4bp + 0p ⇒ tetrahedral (all bonds are equal) * *
(σ1s) 2 < (σ1s) 2 < (σ2s) 2 < (σ 2s) 2 < (p2px ) 2
= (p2py ) 2 < (σ2pz) 2
N2 : µeff = 0
Thus, in C2H4 all the bonds are not equal.
•• N2+ : µeff ≠ 0 ,
21. (b) In NH 3 central atom is nitrogen which is sp3 hybridized
N22 – : µeff ≠ 0
hence it will be at the centre of tetrahedron with H–atoms
at three vertices. N22 + : µeff = 0
22. (a) In trigonal bypyramidal geometry lone pair electrons 30. (a) The correct increasing order of energies of molecular
cannot occupy axial positions – (VSEPR theory) orbitals of N2 is given below

23. (c) O(SiH3)2 σ1s 2 < σ*1s < σ2s < σ* 2s < ( p2p x ≈ p2p y ) < σ2p z
(p* 2p x ≈ p* 2p y ) < σ* 2p z

24. (c) H2S– No hybridisation bond angle ≈ 93º 31. (d) Existence of molecule, bonding nature and energy order
NH3 – Pyramidal 104.5º of molecular orbitals can be explained on the basis of
molecular orbital theory as follows
CH4 – Tetrahedral 109º28’
(i) Molecules having zero bond order never exists while
BF3 – Trigonal plannar120º
molecular having non-zero bond order either exists or
25. (b) As the electronegativity of central atom increases the expected to exist.
bond angle increases due to repulsion between bond pair (ii) Higher the value of bond order, higher will be its
and bond pair as bond pairs are more close to the central bond strength.
atom.
Electrons present in bonding molecular orbital are known
26. (d) The given belongs to d-block of the periodic table. In as bonding electrons (Nb) and electrons present on
transition elements i.e., d-block elements, electrons of anti-bonding molecular orbital are known as anti-bonding
ns and (n – 1) d/ subshell take part in bond formation
electrons (Na) and half of their difference is known as
electronic configuration shows that the element is
bond order.
titanium (Z = 22).
(d) is incorrect. The correct order of energies of molecular
27. (b) For sp2 hybridization, the geometry is generally triangular orbitals in N2 molecule is
planar.
Thus, bond angle is 120°. σ2s < σ* 2s < ( p2p x ≈ p2py ) < σ2p z < p* 2p x ≈ p* 2p y < σ* 2p z

Chemical Bonding and Molecular Structure 39


32. (a) In KO2 , O2– (superoxide ion) has one unpaired electron. 34. (d) PF5 exists as PF4+ PF6−
In BaO2, O22– (peroxide ion) has paired electrons.
(s) sp3 sp3 d 2
In AlO2– , [Al3+ O22– ]– , O22– (peroxide ion) has paired
electrons PCl5 exists as PCl4+ PCl6−
(s) sp3 sp3 d 2
33. (c) XeF4 is square planar in shape.
PBr5 exists as PBr4+ PBr6−
(s) sp3 sp3 d

PBr5
(g) sp3 d

35. (a)

36. (c) Only in BF3 pπ-pπ back bonding is possible due to which
B–F bond length is shortest. In other compounds, p-orbital
at boron is not vacant hence, pπ-pπ back bonding is not
possible.

Multiconcept MCQs
1. (d) 8. (b) (i) (T)
(ii) (F) In general as the number of lone pair of electrons
2. (c) on central atom increases, value of bond angle from
2−
normal bond angle decreases due to lp – lp > lp – bp
 + 
3. (d) (i) (ii)  N= N= N  (iii) (T)
  (iv) (F) Structures of Xenon fluorides and xenon oxy
sp fluoride are explained on the basis of VSEPR theory.
9. (c)
(iii) PCl5 (s) exists as [PCl4]+ and [PCl6]– , 10. (c)
11. (c) Both CO2 and N2O are linear in shape. CO2 is non-polar
but N2O is polar

(iv) I2Cl6()  + –
  [ICl2] + [ICl4]

self ionisation 12. (a)
4. (a)
13. (b)
5. (c) XeF5– : 2 lone pairs ; pentagonal planar
BrF3 : 2 lone pairs ; Bent-T‑shape
XeF2 : 3 lone pairs ; Linear
H3S+ : 1 lone pairs ; Pyramidal Si is sp3 hybridised in SiF4 and it has tetrahedral shape
↑↑ S is sp3d hybridised and SF4 has bent see-saw shape. So,
CH 2 : 2 unpaired ; Linear not isostructural.
(No lone pair) 14. (c) Linear dipole is created in p-nitroaniline, dipole moment
6. (c) in opposite direction cancel out.

7. (d) N2+ has one unpaired electron therefore it is paramagnetic 15. (a) O2 have two unpaired electron, O2+ have one unpaired
and not diamagnetic electron, O22 – have zero unpaired electron. As number
of unpaired electrons increases paramagnetic character
increases.
40 Dropper NEET
16. (b) 21. (d)

According to Bent’s Rule in O2F2, there is more


p-character in O–F bond in comparison to O–H bond in 22. (b) (I) [PCl4]+ → sp3
H2O2 ; hence s-character in O–O bond is greater in O2F2.

17. (a) R = P 2 + Q 2 + 2PQ cos θ ; as θ increases from 90°, (II) has 104.5° bond angle due to lp-lp repulsion.
(III) All have sp3 hybridisation and one lone pair.
cosθ becomes –ve. So R ↓.

18. (d)
(IV)
SeF4 has sp3d hydridisation and see-saw shape I3+ has sp3
hybridisation and bent shape.
23. (a) % s-character ∝ bond angle
1
19. (b) Bond order = ( Nb − Na ) 1
2 ∝ ∝ bond strength
bond length
Bond order of Li2 = 1, N2 = 3, Be2 = 0, O2 = 2
20. (c) A is a VII A group and C is a IA group element. ∴ The 24. (d)
compound is ionic.

NEET Past 10 Years Questions


1. (b) F F 4. (c)
B B The given set of molecules have dipole moment zero.
F F F F Because dipoles of the bond cancel each other.

sp2, Trigonal planar 6e– around central atom.



2. (d)

Hybridisation L.P. Shape


(A) PCl5 sp3d 0 Trigonal bipyramidal
(B) SF6 sp3d2 0 Octahedral
(C) BrF5 sp3d2 1 Square pyramidal
(D) BF3 sp2 0 Trigonal planar
3. (b) Cl
Cl
Sb Cl
Cl 5. (d)
Cl
For He2 molecule
sp3d
Dipole moment (µ) = 0
Electronic configuration is σ1s2, σ*1s2

Trigonal bipyramidal Non-polar

Chemical Bonding and Molecular Structure 41


1 (d) False
B.O
= (N b − N a )
2 Due to longer and thus weaker axial bonds, PCl5 is a
reactive molecule.
1
= (2 − 2) 11. (b)
2
NO : (σ1s)2, (σ*1s)2, (σ2s)2, (σ*2s)2, (σ2pz)2, (π2px)2 =
=0
The bond order comes out to be zero. This indicates that (π2py)2, (π*2px)1 = (π*2py)0
there is no bond formation between 2 He atoms and hence 10 - 5
the He2 molecule does not exist. BO = = 2.5
2
6. (b)
CN– : (σ1s)2, (σ*1s)2, (σ2s)2, (σ*2s)2,
(i) Number of sp2 hybridised carbon atoms is 7
(π2px)2 = (π2py)2, (σ2pz)2
(ii) Number of pi bonds is 6
10 - 4
BO = =3
2

CN : (σ1s)2, (σ*1s)2, (σ2s)2, (σ*2s)2,

(π2px)2 = ( π2py)2, (σ2pz)1


7. (d) 9-4
BO = = 2.5
2
Potential energy of two H atoms at infinite distance = a
CN+ : (σ1s)2, (σ*1s)2, (σ2s)2, (σ*2s)2,
Potential energy of two H atoms at distance equal to bond
length = b (π2px)2 = (π2py)2

So, The bond energy of H2 = (b – a) 8- 4


BO = =2
2
8. (d)
Hence, option (b) should be the right answer.

12. (d)
CN– = 6 + 7 + 1 = 14
CO = 6 + 8 = 14
These two species are isoelectronic and iso structural in
nature. Therefore, they both have same bond order.

13. (a)

BCl3 having bond angles of 1200. VESPR


Theory
14. (d)
[Cu(NH3)4]2+ is not tetrahedral. It is a square planar complex.
9. (c)

σ1s 2 , σ*1s 2 , σ2s 2 , σ* 2s 2 , p2p 2x =


p2p 2y

Double bond in C2 consists of both p bonds because of the


sp2d
presence of four electrons in two p molecular orbital. In other
15. (c)
molecule a double bond is made up of a sigma bond and a
pi bond.
10. (d)

Hence, they both are not isostructural.


16. (d)
O, Se, Se, Te belong to Group 16.
On moving down the group, size of atom increases.

42 Dropper NEET
’O’ is most electronegative and lone pairs 21. (a, d)
lie close to the atom electron cloud. This
causes repulsion in lone pairs of oxygen
and bond pairs of hydrogen.

∴ Angle maximum due to l.p - b.p repulsion.

Te has maximum size: lone pair lie far away


from the atom electron cloud. Lone pair - bond 22. (c)
H
pair repulsion is the least.
CH4 – sp3 hybridised, tetrahedral C
17. (a)
H
Intramolecular hydrogen bonding takes place within the bond angle 109o28ʹ H
H
same molecule i.e., between the atoms of the same molecule.
NH3 – sp3 hybridised, bond angle – 107o
Since, cellulose is a complex structure containing oxygen
and hydrogen the bonding occurs between them easily.
Whereas, in HCN, H2O2 and concentrated acetic acid inter- H2O – sp3 – bent shaped
molecular hydrogen bonding occurs.
Eg.: bond angle - 104o5ʹ
So, bond angle of H2O is less than that of NH3 & CH4
23. (b)
18. (c) Order of repulsing force according to VSEPR theory is lone
H H H H pair – lone pair > lone pair – bond pair > bond pair – bond pair.
H H 24. (b)
H H H H O2 (atomic number) = 16
Coplanar are in a plane or where all C atoms are sp2 Molecular orbital Diagram:
hybridised. Bond order = 1/2 (No. of bonding orbital – no. of anti bonding
orbitals)
19. (a)
= 1/2 (10 – 6) = 2
SeF4 and CH4 do not have same shape. SeF4 is AB4L type
Similarly, For O–2 = 1.5
molecule with 4 bond pair and 1 lone pair with shape
O2–2 = 1
see–saw. CH4 is AB4 type molecule with no lone pair and
1
tetrahedral shape. and Bond order ∝ ∝ stability
Bond length
I3+ have 2 lone pairs with bent/angular shape. BiCl5 does not
∴ Order of Bond length = O2–2 > O–2 > O2 > O+2
exists because of inert pair effect. SO2 type molecule have
both pπ - pπ & dπ - pπ bonds.
20. (c)
Hybridisation state = Number of σ bond + number of lone
pair or
Hybridisation state → from steric number rule
1
Hybridisation state = (V.E + MA – C + a)
2
1
For, NO +2= ( 5 + 0 − 1)
2
= 2 → sp
1
NO3= [5 + 0 + 1]

2
= 3 → sp 2
1
NH +4= [5 + 4 − 1]
2
= 4 → sp3

Chemical Bonding and Molecular Structure 43


25. (a) 34. (c)
Each pi bond have 2e – involved so 4 pi bonds = 8e –. Among the 4 options (CH3)2 , (SiH3)2 and PH3 all have
complete octets with 8e─ . But BH3 has only 6e─ and act as
26. (a)
a Lewis acid.
Bond order:
= O −2 1.5=O 2 2=O +2 2.5
35. (b)
O +2 > O 2 > O −2 In paramagnetic species there is a presence of unpaired
27. (c) electrons.
Indentation Bond order: Except O2– all contains paired electrons.
O 22 + 3,=
= O +2 2.5 → O 22 + > O +2 > O 2−
O −2 1.5,= 36. (a)
28. (b) By MOT for filling of e– in molecular orbitals
29. (a) He +2 0.5,
Bond order of= = NO 2.5,
= O −2 1.5

Species → NO3– NO2 NO2 NO2+
C 22 − = 3
Hybd. → sp2
sp2 sp2 sp
o
Bond angle → 120 134 o 115o 180o So, C22 − > NO > O −2 > He +2
↓ ↓
B.P.– B.P. L.P. – B.P. repulsion 37. (d)
repulsion dominant So, For the reaction
dominant Bond angle ↓
So bond angle↑
30. (b) The reaction will be faster with Rb because lattice energy of
RbF is less than LiF, NaF, KF due to large size of Rb+

38. (b)
O 22 − : 1 B2 : 1
O2+ : 2.5 NO+ : 3
NO : 2.5 CO : 3
N2 : 3 O2 : 2
39. (c)
= 0.23D O 2 ⇒ σ1s 2σ*1s 2σ2s 2σ* 2s 2σ2p z2 ( p2p 2x = p2p 2y )( p* 2p1x = p* 2p1y )
31. (a)

NO3 resonating structure 1
O 2 :Bond order = (8 − 4 ) = 2
2

1 1
O +2 :Bond order = ( 8 − 3 )= 2
2 2

1 1
Planar structure O −2 :Bond order = ( 8 − 5 )= 1
2 2
32. (a)
Hybridisation of CO2 is sp O = C = O 1
O 22− :Bond order = (8 − 6 ) = 1
and of ethyne C2H2 is sp H – C ≡ C – H 2
33. (b) 40. (a)
– +
BF4 & NH4 have sp3 hybridised central atom and exhibit
F tetrahedral geometry.
m=0
F
41. (a)
F
m=0 Electronic configuration of O2

σ (1s)2, σ* (1s)2, σ (2s)2, σ* (2s)2, σ (2Pz)2, π (2Px)2, π (2Py)2,


π* (2Px)1 = π*(2Py)1
m=0 m≠0
So, incoming e- will enter in π* 2Px / π*2Py

44 Dropper NEET

You might also like